Vous êtes sur la page 1sur 370

!

PART I:
REPRESENTATIVE PROBLEMS

PART II:
TOPIC WISE PROBLEMS
[ liT- JEE 1972 ONWARDS]
I

i'
I'
"

..:1 I'
,t
,

"

.11 '"

;i
:" 1
11 .,

Ii,i ,I

i,
. I'" "
(i
.. :ll c.
o ' .~.~ -".
'", .

~. ': I

..
.. .~

I
I
j

,
"
I

)
I
:1
Ii
"
PROBL-
"IN
PHYSICS
(EXCLUSIVELY FOR IIT-JEE MAINS)

~ G. K. PUBLISHERS (PVT.) LTD.


Published by :
RAKESH MITTAL
for G.K. PUBLISHERS (P) LTD.
H-205, SECTOR-53
NOIDA - 201307 (U.P.)

@AUTHOR

No part of this book may be reproduced in a retrieval system


or transmitted, in any form or by any means, electronics,
mechanical photo copying, recording and or without
permission of th~ publisher .

.E.dition : 2005

• ~ ".~ "- - -- - - -0;:-- --


..

CONTENTS IN BRIEF

PART. I
Mechanics ; ..........................•.••........................... 1 - 36
Waves •.•..................................................................... 37 - 40
Heat and Thermodynamics 41 - 46
Electro Magnetics 46 - 69
Optics ......•....................................•........................... 69 - 77
Modern Physics : 77 - 80
Measurements 80 - 84
• Answers .....•..•••..•..••..••.•........•........•.....•..•••••....... 85 - 106
• Explanations ......•.•.•..........•••••••••••.•................... 107 - 182

PART - II
Kinematics ..........................•.......•......................... 183 - 185
Projectile .............••.•............ 185 - 18 7
laws of Motion .....................••.••........................... 187 - 190
Friction ............................................................•..•• 190 - 194
Circular Motion ...................................................• 194 - 195
Work, Power & Ene. If,Y ••••••••••••••••••••••••••••••••••••••••• 196 - 199
System of Particles 199 - 208
Rotation 208 - 21 5
Gravitation .............................................••............ 2 I 5 - 216
Properties of Matter .•............................................ 2 I 7 .. 217
Hydrostatics 2 I 8 - 22 I
Fluid Dynamics 221 - 223
Viscosity ....•.......................................... ,..........•..... 223 - 223
Surface Tension 223 - 224
SHM 224 - 226
Waves 226 - 233
Doppler's Effect. 233 - 234
Thermal Expansion 235 - 238
Calorimetry 238 - 239
Heat Transfer. 239 - 243
Kinetic Theory of Gases 243 - 245
Thermodynamics 245 - 252
Electrostatics 252 - 261
Capacitance 261 - 263
CurrenL 263 - 273
Magnetic Field 273 - 281
Electromagnetic Induction 281 - 288
Alternating Current 288 - 288
Ray Optics 288 - 299
Wave Optics 300 - 306
Photoelectric EffecL..•............................................ 306 - 308
Atomic Physics 308 - 3 I 2
Radio Activity 3 I3 - 3 I4
Nuclear Physics 314 - 316
• Answers ..........••............••••.••••.••........................ 31 7 - 350

••
PREFACE•••
The book is in two parts. The first pan of the book has new
arid representative problems. Answers have been given to all the
problems but solutions are given for odd numbered problems to
help the students in developing their problem solVing aptitutde~.

However I have given the solution but students are advised to


refer these solutions only after attempting the questions.

Part II is having previous years problems from lIT - J EE


(1972 onwards). In part II of the book students will get an idea
about the patterns and level of the questions being asked in IIT- JEE
for different topics. Part II is specially designed! for last hour
preparation.

Last but not the least, I wish to' e>onvey my tllanks, to the
publisher of this book Mr. Rakesh Mittal, Mrs, Roanam Mittal &
Mr. Nitin Bhargava for bringing out this book in an excellent Ibnn.

All efforts have been made to keep the' book fj;ee'from errors.
Inspite of my best possible efforts, some printing. errors might have
occured. I shall be grateful to the readers if the same are'f:jrought to
my notice.

Suggestions for the improvement of. illis' lillOk, ar:e?welcome.

April 2005
New Delhi AIt4AtJ'i'~. ~
B. Tech
PRACTICE PROBLEMS ' 1

(PRACTICE PROBLEMS)
IPART.II

~,,""<e?,},1
L!ill].,?<::'::"'<"'«<e, ••u,:,',:'.',,':"""""
On a cricket field, a rabbit is at the origin of coordinate system
and a dog at the position (46 m, 28 m), The rabbit runs on the

ground with a constant velocity (7,51+ lOJ) mls. The dog can run
with a speed 5 mls. If the dog starts to run immediately the rabbit
starts, what is the minimum time in which the dog could catch the
rabbit?

A particle A has speed of5 mfs. At time t; 0 its position relative to


origin of coordinate system is (-11 m, 16 m) with the aim getting
as close as possible to another particle B. At time t; 0, the particle
B is at a point (4 m, 36 m) and is moving with constant velocity

(101-5J) mls. Find the velocity vector of particle A and the


minimum distance of approach.
~ •.•...•.. , ••................... - -.,..... .,.,., , ,., ' .
W )?""""""::"<><:"'\"?:?? ue.>" ,
A swimmer can swim relative to water
with a velocity v and the river flow
velocity is u. The swimmer starts from
B
a point A on one bank to reach a point , t
r.C
C on the other bank which is at a : /
,, '
'
distance 1 from the point B directly d:, ,,/ -+u
opposite to A on the other bank, as ,, ''
shown in the figure. The width of the
river is d. Find the time taken by the
.':
A
swimmer to go from A to C.
2 . PRACTICE PROBLEMS

A boat is moving iill a triangular course


in a river. The flow velocity of the river

6
is u and the velocity of boat relative to t
~U
water is u,f3. The length of the each
A C B
side of the triangle ABC shown in the t
figure is I. Find the time taken to
complete the full course.
E[]]•...••.•..•••
U> ••.•.•..........
A man can swim with a velocity v
relative to water and can run on the B
ground with a velocity w. The flow ,,
,,
velocity of the river is u. The width of the
river is d. The man is at a point A on one d:, ~u
,,
bank and wants to reach point B lying ,
right across on the other bank of the
,
A
river as shown in the figure. Find the
minimum time in which the man can
reach the destination.

lill!illlu.yn'.nn.uy' .
Two particles A and B'move with constant
velocities v1 and v2 along two straight lines ,A

<
v,
inclined at angle e towards the intersection
point O. At the moment t = 0 the particles a e
were located at the distance 11 and 12 from .' (J2
I, B
the point 0 as shown in the figure. Find the
shortest distance between the particles.

In a still water lake a steamer is observed by two persons


travelling at speed v into two boats in opposite directions on a
straight track AB. To an observer in one boat the steamer appears
to cross the track AB at right angles while to the observer in the
other boat the angle appears to be 45': At what angle does the
boat actually cross the track and what is its speed?
'I • 'II: •
A B
PRACTICEPROBLEMS 3

C!J _
A man running on a horizontal road finds the rain falling at angle
(J. with the vertical. Ifhe doubles his speed angle becomes p. What

will be the angle when he triples his speed?


w _
A -particle starts from rest with zero initial acceleration. The
acceleration increases uniformly with time. Find the time
average and distance average of velocity upto a certain instant
when the velocity becomes v.

~------------------
A particle starts from rest and moves on a straight line. The
acceleration of the particle varies with time as shown in the
figure. Find the velocity and displacement of the particle at t = 4s.
n(m/s2)

3 4 tis)

lliJ _
A particle moves along a straight line such that its displacement x
from a fixed point on the line at time t is given by
x' = at' + 2bt + c.
Find acceleration as a function of displacement x.

~-- ------------------
A particle moves in a straight line under an attraction towards a
fixed point 0 on the line from rest at a distance 'a' from O. If its

acceleration when at a distance x from 0 varies as ~, find the


x
time period of the oscillatory motion about O.
[ill ~
A particles moves along the x-axis. The velocity ofthe particle as a
function of x is given by
v' = 12:< - 3x3.
If (be the instantaneous acceleration, express velocity as a
function of(.
4 PRACTICE PROBLEMS

~------_-'---.:.~--'-'------
A body of mass m is thrown straight up with initial velocity vo.
The air drag equals kv', where k is a constant and v is the velocity
of the body. The acceleration due to gravity isg. Find the velocity
with which the body comes down.
[ill _
A particle moves along the x-axis. The accleration of the particle
is given by
a = (kl _w2x), where k and ware positive constants.

At I = 0, x = 0 and ~~ = O.Find the displacement x as a function of


time I.
[!i] ....:....., _
A particle moves along the x-axis. The acceleration of the particle

(
is given by, f = x3
).
-
~) '.
x2 ,where). and ~ are positive constants.

The particle starts from rest from x = a. Find the time period of
the oscillatory motion performed by the particle.
~ .

. A particle is projected in X-Y plane y

from origin at I = 0 with initial


velocity 5 mls. A constant : /' S mis'
acceleration of 5 mls2 making an ~30° x
angle 30° with x-axis, as shown in Smls
figure, acts on the particle. Find the
displacement and distance travelled
by the particle at I = 5 seconds.
Iill --_
A particle is projected from the ground at
an angle 45° with a initial velocity 10 mis, 10 mls
as shown in the figure. The acceleration !------ ",
;;:is'! ",
due to gravity is g = 10 mls'. Find the -~"-"--_
average velocity and average speed upto g
the instant when particle comes back to
the ground.
PRACTICE PROBLEMS S

~------------~--
'!\voparticles are projected from the same
point with velocities 0 and 20 making v . 2v
equal angle e = 30° with the horizontal in
oppositedirections as shown in the figure. )c.,.4
Find the separation between them when
their velocity vectors become mutually
perpendicular. The acceleration due to
gravity isg.
1201 _
Aparticle is thrown from a height h
horizontally towards a vertical wall
moving away with a speed 0 as
shown in the figure. If the particle
returns to the point of projection
1].
It
v

x
I-VI4
aftllier. suffering. tthwo alellaStidc
nm;~ :mnmm:ry;mm
co Slons, one WIth e w an
another with the ground, find the
initial separation x between the
particle and the wall.

~-----------------
A projectile is fired with velocity u
at an angle e so as to strike a point
on the inclined plane inclined at an
angle a. with the horizontal. The
point ofprojection is at a distance d
from the inclined plane on the
ground as shown in the figure. The
angle e is adjusted in such a way
that the projectile can strike the
inclined plane in minimum time,
find that mlnimum time.
1221 -'-- _
A particle is projected with initial
velocity 0 at angle e with the ~
horizontal. There is an inclined
plane at an angle a. through the : 9 a
point of projection as shown in the _-
figure. The angle e is adjusted so ~~~------
that the particle strikes that plane
at an angle 45°.Find the height h of
the point struck above the point of
projection.
6 I PRACTICE PROBLEMS

~r;;;;-, "'F.~.",.",.,.",..,~""r,:
~'1"::,~h:;'~'lqlf.:'
:'ittd'. -""'''''-',,", 11 ~w,,""''''''""
rl:~~~1)h:;,j,->!i~:-~.'~~::I}~~~~i~: ""'-'~"', "'~
~~ll'ttZ~';~~~i;o1~i)~~"';;h
n
A particle is projected with an initial
speed it from a point at height h. above
the horizontal plane as shown in the
figure. Find the maximum range on the
horizontal
.,
plane .
r.>Al""-I'f'''''
~ .'tfl:l,~,w"~~_~!fil'~)l'
';;~'1~1),
!~~;,J:.'i.J~~-~i/~~ '~'
::ir<h~ki-~~l'ki'l'i:~:.t.~:~:~i
{!••. "i~;:~~I}i "~,,.,,."
,~-t~'.:i~fJ
A particle is projected from the
ground such that it touches the
four corners of a regular hexagon
of side length a, as shown in the
figure. Find the corresponding
horizontal range on the ground
and the time of flight.
125'll~~'~B:~~~h'7l~~~'JJ]
Two planes are inclined at angles lX and P
with the horizontal and a particle is
projected at right angle to the one plane
from a point at a distance 'a' from the
point of intersection of the planes as
, shown in the figure. If the particle strikes
to the other plane at right angle, find the
time of flight.
1261 J7i~t~;f4!;'~~.:~~-~~f\'t~?i~iA~i'm
A particle is projected from the
. point 0 on the ground to hit a
target P at a height h above the
ground as shown in the figure.
Find the least value of the velocity
of projection u and the time taken
by the particle to reach from 0 to P.
127- I ~~"~'~~W~S11ln~~;~~~~~6?J~~J
A particle is projected from the ground I
with initial velocity u at angle lX to the • ~ ~
horizontal and strikes elastically a II ' '.
vertical wall moving towards it with a . /. •
horizonThtal
figure:
veartiloc.itlY
e pee
~ absshdowdnbinkthtoe
IS oun e ac
,an,,~':
. I--;- % •

the point from where it was projected. ,.


Find the initial distance x of the wall
from the point of projection ..
PRACTICE PROBLEMS ( 7

1281 _
A particle is projected with initial
velocity v as shown in the figure.
After elastic collision with the
inclined plane the particle
rebounds vertically. Then the
particle retraces its path and
comes back to the point of
projection. Find the time it takes
to return to the point of projection.
1291 _
A stone is projected with initial
velocity u at angle a with the
horizontal. Find the angular
velocity of the stone with respect
to the point of projection, when it
is at its maximum height.
1301_. 0_" _-_0. _

A particle is projected from the ground att = 0 with initial velocity


u at an angle a with the horizontal. Find the radius of curvature
of the path at time t.
1311 _

1\vo particles A and B are moving with


uniform velocities v, and V2 along two
perpendicular lines as shown in the
figure. At t = 0 their distances from 0 are
/1 and /2 respectively. Find angular
velocity of the line joining them at time t
and its maximum value.
rill _
1\vo particles A and B describe coplanar
concentric circles of radii RI and ~ with
angular velocities "'I and"", as shown in the
figure. Find the angular velocity of A with
respect to B.
8 I PRACTICE PROBLEMS

A string, attached with a block passing


over two fixed pulleys A and B, is being B
pulled as shown in the figure. The radii of v
pulley A and B are R, = 10 em and ~ = 5
em respectively. If the normal component
of acceleration of a point P, on the pulley
A is 40 m1s2 and the tangential component
of acceleration of a point P2 on the pulley
B is 30 m1s2 at the instant, calculate the A
corresponding speed u of the tape, the
magnitude of total acceleration of P" and Block
the magnitude of the total acceleration of P2'
I!m :::':::n:::,,:::::::::::::::n::::::::::::::::::::;::::::::::::':':':::::::::::::::::::::::;::::::::n:n:::::n:::'::::
Consider an arrangement shown in the figure. 'A' is a bobbin,
which can roll on the horizontal plane, B is a fixed pulley and C is
a movable pulley. The string connecting the bobbin 'A' to the
pulley 'B' makes an angle '<1.' with the vertical at any instant and
the block attached with the pulley C is being pulled down with an
instantaneous speed u. Find the velocity of the centre of the
bobbin at this instant if it rolls without slipping.

A rod is bent into the "L" shape and


hinged at 0 so that it can be rotated
about z-aXis in x-y plane as shown in
the figure. At the position shown in
the figure, the angular velocity is y
2 rad/sec and angular velocity is
decreasing at the rate of 4 rad/s2.
Find the acceleration (in vector form)
x
of the end A of the rod?
PRACTICEPROBLEMS 9

~---------------
A sphere 'A: of radius R is rotating
1
about its centre 0 with a constant
angular velocity (i) I' Another sphere B
of radius R:!rolls without slipping with
angular velocity (i)2 over the sphere A
A
as shown in the figure. Find the
acceleration of the point of contact of
the rolling sphere.

~,----------------
A rod OA is rotating about 0 with
counter clockwise angular velocity y'
4 rad/s about z-axis which is decreas- ';'.... '--t, -- ..
ing at the rate 10 rad/sec2. The motion of , ,, ,
,, , ',
a slider's' on the rod is separately
controlled. At a particular instant, as
shown in the figure, the slider's' is at a
,
:,,
,,
~-'
Ci
,
fllIj
s
'
f-:-x
,A
,
distance 6 cm from 0 and it is moving , ,,
away from 0 with an instantaneous
velocity 5 cm/s and acceleration 81 cm/s2
\', ..•....
-._---'"
,";
"
with respect to the rod. Determine the
absolute velocity and acceleration of's'
for this position in vector form.
[ill ~ _
A wheel of radius R rolls on a
horizontal ground with a speed v.
An insect crawls at a constant speed
u along a spoke of the wheel as
shown in the figure. Find the
acceleration of the insect with
respect to the ground in terms of
the distance r of insect from the
centre of the wheel?
lill _
A rigid rod OA is freely attached with another rigid rod OB at 0 as
shown in the figure. The lengths of the rods OA and OB are 3 m
and 4 m respectively. The entire assembly rotates about Z-axis
with a constant angular velocity 3 rad/s. Simultaneously, Lherod
OA is being rotated (raised) about an axis l.hrough 0 at the
con.Lant angular velocity 4 rad/s. At a parLicular instant the rod
OAis along Y-axis as shown in the figure. Find the velocity of the
end A at this instant in vector form.
10 . PRACTICE PROBLEMS

3m
A Y

140 I.
A disc of radius r, attached with a
horizontal axle OA, rolls without
sliding on a circular track of radius R
as shown in the figure. In the process,
the centre of the disc moves with a
constant speed u. Find the angular
acceleration of the disc.
[ill.
A rod AB of length 4 m slides
with its ends in contact with
the floor and the inclined
plane. At a particular instant
the position of the rod is shown
in the figure. At his instant the
end A is moving towards right A
with velocity 4 mls and
acceleration 5 m/s'. Find:
(a) angular velocity of the rod
(b) velocity of the end B
(c) angillar acceleration of the rod
(d) acceleration of the end B.

1421
A rod OA of length 4 m is hinged at O.
Another rod AB is attached freely at A
with the rod OA. At a particular instant
the positions of the rods are shown in the
figure and the rod OA is rotating
clockwise with angular velocity 1 radiauls. 1J
Find:
(a) angular velocity and angular accelera-
tion of the rod AB
(b) velocity and acceleration of the end B.
PRACTICE PROBLEMS, 11

@]---------------
1\vo rods of equal length AC and BC c
are freely joined at C. 1\vo ends A
and B are pulled with speeds 2 mls
and 3 mls respectively, as shown in
the figure, at a particular instant. ._A
Find the speed of the point C at this 3m!.
instant. .
1441 _

D
1\vo rods AB and CD rotate about ends A
and C respectively in plane are
immediately above the other, as shown
in the figure. Find the velocity and
acceleration of the point of crossing for
A. C
the orientation given in the figure. ~50t:m -l

~---------------
A rod AB of length 1 m is attached to two sliders at A and B
moving on the guide bars as shown in the figure. Find the velocity
of the slider B for the situation described in the figure when
VA = 0.2 mls. Z
"A!

1\vo rods AB and CD are placed in D


a plane intersecting each other at
an angle El. The rods move
perp~ndicular to themselves in
the same plane with velocities v, A B
and "2 as shown in the figure.
Find the velocity of the point of
the intersection oftwo rods.
12 I PRACTICE PROBLEMS

1471'__ ~ ~_~~~
Consider an arrangement shown in the figure. The block of mass
m" is constrained to more in the vertical direction only. The wedge
of mass m. moves in the horizontal direction. The slider '8' of mass
m2 moves on a fixed horizontal rod. The friction between all the
contact surfaces is negligible. At a particular instant the string
connecting the slider '8' to tbe block of mass In! is making angle 8
with the rod. Find the acceleration ofthe block of mass In! at this
instant? The accE>!eration due to gravity isg.

In the arrangement shown in the figure, the masses of all the four
blocks A, B, C and D is equal to In. The masses of pulley and the
thread are negligible. The friction in the pulleys is absent. If the
system is released only one block moves up and the other three
blocks come down. Which block starts moving up and with what
acceleration? The acceleration due to gravity isg.

mr ilWl\k
C D
A
PRACTICE PROBLEMS 13

@]---------------
In the arrangement shown in the
figure, friction at all the contact
surfaces is absent. The mass of
pulleys and thread is negligible.If
the system is released from rest
in the position shown in the
figure, find the acceleration ofthe
wedge?
[ill _
In the arrangement shown in the figure
pulleys are massless and frictionless. The
masses of the blocksA, B, C is equal to m
and the mass of the block D is 2m. If the
system is released from rest, find the
acceleration of the block A

[ill _
Two blocks of masses m, and m2
are connected with a string and
are placed on a wedge of mass m3
as shown in the figure. The
friction between all contact
surfaces is negligible and the
masses of the pulley and thread
are also negligible.If the system is
released find the acceleration of
the wedge?
fill_' _
In an arrangement shown in the
figure, there is no friction at any
contact surface. The masses of
pulley and thread are negligible.
If the system is released, find the
acceleration ofthe "'edge.
777

~---------------
A block of mass is placed over a plank of mass m. as
In)
shown in the figure. The coefficientoffriction between plank and
14 " ' PRACTICE PROBLEMS

the ground is ~2 and between


block and plank is ~j' A force of
magnitude F is applied on the
block at an angie a with the
horizontaL It is found that the
plank moves on the ground but
the block does not move relative to
the plank, find the range of F
(Assume cot a > ~j > ~2)'

IMI»'
In the arrangement of the
blocks shown in the figure, mj = 2
kg, m2 = 3 kg and ms = 7 kg, The
coefficient of friction between
mj and m2 is ~j = 0,2 and between
m2 and ms is ~2 = 0,3, There is no
friction between ms and ground,
Find the acceleration aj, a2 and as
of the blocks mj, m2 and m3 if the
magnitude of the force F applied
on ms equals to
(aj 12 N, (bj 30 N, (cj 48 N,

I~KI»'
In the arrangement shown in the
figure, a cylinder of mass mj is placed
in the groove formed by two identical
prisms each of mass mj' The
coefficient of friction for all the
contact surfaces is ~, Find the initial
acceleration of the cylinder, just
after the release of the system from
rest,
15~1>
In the arrangement shown in the
figure, the coefficient of friction
for all the contact surfaces is ~, "';'_.- ..•• ,..••• r-;

i.
The masses of pulleys and threads
and negligible,The blockofmass ms , m,
always remains in contact with the
block of mass emj' Find the
aoceleration of the block of mass mj
just after the release' of the "'ysteHl
from rest.' .
PRACTICE PROBLEMS' 15

lill_. _
1\vo particles of masses ml and m2 are A
connected with the two strings of lengths
II and 12 with the end B of a rod AB as (,)

shown in the figure. The rod AB is rotated


about the vertical axis so that the rod AB B
I,
remains vertical. Find the angular
velocity'" of rotation? The acceleration m,
due to gravity is g.
!ill_' _
In the arrangement shown in the
figure, initially the block of mass ml is
In,
in limiting equilibrium. The coefficient
offriction between the block of mass m1
and floor is fJ.. Now the particle of mass
m2 is given a horizontal velocity vo. The .
initial distance of the block of mass m2
from the pulley is I. The masses vf
pulley and threads and negligible. Find
the initial acceleration a of the block of
mass ml and initial radius 'R' followed
by the block of mass m2'

'~-' ---------------
A thin rod is bent to form a horizontal
circular track of radius R as shown in
the figure. A ring of mass m is given an
initial velocity of magnitude vo. If the
coefficient of kinetic friction is fJ.,
determine the distance travelled
before the ring comes to rest? The
acceleration d\Je to gravity isg.

~_.. ---------------
A rough vertical circle of radius R,
carrying a 1:Jeadof mass m, rotates in its
own plane about its centre with uniform
angular velocity "'. The coefficient of
friction is fJ.. Find the minimum value of
'" so that the bead will never slip? The
acceleration due to gravity isg.
16 I. PRACTICE PROBLEMS

161 I;. ".",,-" .


_ .~. _ _ .• (""_"r ~ : __ ~'-'- ~"'~'~~""' •• ', ' __ • '."'-- h

,
.. ,
"

A particle ofmass m is tied to one end of a ;"


,,'
------ ..-.•. "": .•.
light inextensible string of lengthl the ,, ',
,,
other end ofwhich is fixed as shown in the ,,
, ,
figure. The particle is projected with a , '
: .~ :
velocity v from the lowest position. Find '. :.).. :
the net force acting on the particle, when " ; 0 ..".... I
.... I: 1/
the string makes an angle 0 with the .............
~L.
1 r .
vertical?Theacceleratioildue togravity isg. m u
62

Aparticle suspended by a thread is projected with


a velocity v = zJii
from the lowest position as
I-----: :
shown in the figure. Find the time taken by the I ,/
particle to reach the horizontal position. 'ii,
~i:" -.

A particle slides down the surface


of a smooth fixed sphere ofradius
R starting from rest at the highest
point B. The particle leaves the ,,
sphere at some point and then ,,
strikes the horizontal plane ,,
,,
t!:lroughthe lowest point A of the ,,
sphere, at a point P as show:!.in
A p
the figure. The acceleration due to
gravity is g. Find the distance AP.

A particle is attacbed to a fixed point by a


fine string of length 1 and is projected
horizontally flom the lowest point with :,;,... -... -.. .•.....
"":

, ,
/
,, ,,
'~\

velocity vo = 19 (2+ ~) as shown in


: ..~ :
the figure. lIn the subsequent motion the ,
\

, 'I
I
'
.f

I
,
string first :becomes slack and again , I: .;.'
becomes,tight and ,then 'passes through
••••••

.. .•..•.••
_~.I'
.
I

-~'
••.

lowest point 'alongcircular path. }o'ind'the 'm (10

speed v with which the string will pass


through'the;lowest'point first time.
PRACTICE PROBLEMS 17

~---------------
A slider can slide on a smooth vertical elliptical guide as shown in
the figure. The fixed point 0 is located at one focus of the elliptical
guide. One end of spring is connected at 0 and the other end is
connected to the slider. The spring is unstretched when the slider
is at A. The speed Vo given to the slider atA is such that the speed
approaches to zero at C. The semi-major and semi-minor axes are
'a' and 'b' respectively. The acceleration due to gravity is g.
Determine the speed of the slider at B.

~---------------
In the arrangement shown in the
figure m, = 2kg, m2 = 1 kg,
K, = 15 Nlcm and K2 = 5 N/cm.
The acceleration due to gravity is
10 mls2• Initially the system is in
equilibrium. Find the work done
by external agent in slowly pulling
down m2 by a distance of 8 em.
1671 _
A uniform chain of length I rests on a
{l-ll)l
rough table so that one end hangs over the A
edge. The chain slides off the table all by
itself when 11 fraction of the length of the
chain hangs, as shown in the figure. The Smooth
chain starts sliding from rest. Find the
speed of the end A of the chain when it
completely slides off the table.
[ill ~ _
Find the work done by a force, y B (2m.4m,0)
F = [(y2 - x2 + Z2) i + (3xy - 5Z»
+ (4Z) k 1 in taking particle from
origin 0 to the point B (2m, 4m, 0)
along the path OAB as shown in --;:-l-~~--l_-x
o A (2m.0.O)
the figure.
18 PRACTICEPROBLEMS

@!]--------------
In the arrangement shown in the figure, all the contact surfaces
are frictionless. Find the distance covered by the wedge of mass
'm' on the horizontal plane till the mass m, is lowered by a
distance l along the surface of wedge.

A uniform plate having a circular hole is shown in the figure. Find


the co-ordinates ofthe centre of mass.
y 3cm

Bern

E
u
Gem '"
x
J- 20cm -I
[ill
In the arrangement shown in the figure, m] = 3kg, m2 = 2 kg,
M = 5 kg and there is no friction at any contact surface. The
acceleration due to gravity is 10 mls2. The system is released from
rest. After 1 second, the velocity of m] is 3 mls. Find the kinetic
energy of the system at this instant.

In, In,

M
PRACTICE PROBLEMS 19
lill _
Consider an arrangement shown in the figure. There is no friction
at any contact surface. The system is released from rest and the
string connecting the blocks A and B is also cut. Find the
maximum extension in the spring connecting the blocks A and B
in the subsequent motion. The acceleration due to gravity isg.
A Skin' B

m,

[ZTI _
A heavy particle of mass m is
placed on the top of a smooth
hemisphere also of mass m
which is placed on a smooth
horizontal plane as shown in
the figure. The system is
released from rest. Find the
angle e with the vertical where
the particle will loose contact
with the hemisphere.
lill _
A particle of mass m is placed on
the wedge shaped block of mass
M as shown in the figure. The
system is released from rest.
Neglect the friction at all the
contact surfaces and also neglect
the impact at the centre O. Find
(a) maximum velocity acquired
by the block .
(b) average speed of the block
over one period of oscilla-
tion.
20 PRACTICE PROBLEMS

Two men each ofmassM stand on


two platforms each .of mass m
hanging as shown in the figure.
One of the man leaps vertically
upward with a velocity Vo relative
to the platform. Find the distance
through which that platform will
descend. m

Three particles A, Band e of mass m,


2m and 3m respectively lie on a smooth m
horizontal table. A and B as well as B A
and e are connected by light
inextensible strings each of equal
length Z. The string connecting A and B C
is tight. The initial distance between B 3m

ande' IS zJ3
2 and partIe
. Ie' .
e ISgIven a
velocity Vo parallel to AB as shown in the figure. Find the velocity
of the particle A, just after its starts moving.
17~IH>.
A particle is projected from a point on a smooth horizontal ground
with velocity v at angle a to the horizontal. The coefficient of
restitution for the collision of the particle with the ground is e.
The acceleration due to gravity is g. Find
(a) the total time for which the particle rebounds on the ground.
(b) the distance on the ground from the starting point when the
particle ceases to rebound. ~
\,,\,~,

A smooth circular table is surrounded by a


smooth rim whose interior surface is
vertical. A particle whose coefficient of
restitution is e is projected along table
from a point on the rim in a direction
making angle 6 with the radius through
the point and returns to the point of
projection after three impacts on the rim.
Find the angle 6 ?
PRACTICE PROBLEMS 21

A block A of mass ml is resting on a long plank B of mass m2 as


shown in the figure. There is no friction between plank .B and
the ground. The coefficient offriction between block A and plank
is !t. A bullet of mass m is fired with velocity Vo into the block A
and gets embedded in it. Find the dislllacementofblock A on the
plankB.

---"
.' ..

A particle strikes the floor with a velocity ih. The coefficient of


restitution betw~en the particle and floor is e. The unit vector
normal to the floor is fi. Find the velocity (vector) ;;2 of the
particle with which it rebounds.

A moving sphere A of mass ml experience a perfectly elastic


collision with a stationary sphere B of mass m2 as shown in the
figure. At the instant of collision the velocity vectors .of.Amakes
an angle of 30 with the line joining the centres of A and B. Mter
0

collision the spheres fly apart symmetrically relative to ,theinitial


0
motion ,direction ofthe. sphere A with 'the ,angle of divergence ,60 •

'Find.the ratio mi ..
'11iz

AV
'.~

~ • ..J:!
::,,:,::~::,:;::;:;':"
...........
......... :...
;.:.:.:.:.:
;.

A sphere of mass m is moving with a velocity VI = + mls (I 2J)


and another sphere of mass 2m is moving with a velocity
v2 = (-I + 3J) mls. At the instant of thier collision the line joining

their centres is parallel to a vector r = (I - J) m. The coefficient of


1
restitution, e = "2' Find the velocities v~and v~of the spheres
just after collision.

1.8,... ,.3....."...'1 ,'.•,'.•,'.•,'.•,r"...


..."•., .•.•.'.•.•.'.•.'" ...r.'."•.,,'.•...'.•,'...',•.',•.•.•.•..'. .;.•,!.' .•,','.:,..:,:.:,' ..',.'..'.:, '.'.'.'.'.".",'.", .,':.'.'.:•.•'.".:.: ••.•.•.•., "" .,::,,::::,:;;-:':'.:.:::;::::.:::':}';;
;.:;:.:::::;:;::,::::::,::::::,:::'::.:.:.; :} : ••
;
? ",'
:;::

'!\vo discs whose masses are ml and m2 .


are moving in the directions, making
angles a and fl with the line joining
their centres, collide as shown in the
figure. The directions of motion after
the impact is found to be perpendicular
to their direction before impact. Find
the coefficient of restitution.
I,$~T.:•••••
:::!:::••••••!.:.",'.U ••'.!••'••••••'•••••,'•••U'.!' •••••'•••••••m:::!•••••••••••'••,••••!.!••m•••••••,•••••••••••;•.•'.;';:,.,.:::::;;
A smooth sphere impinges obliquely on an
identical sphere at rest. Before impact the
first sphere was moving in a direction
making an angle e with line joining
centres at the moment of impact as shown
in the figure. The direction of motion of
the first sphere is turned through an
anlge 0 by the impact. The angle e can be
. changed to get the maximum angle of
deviation O.The coefficient of restitution
is e. Find the maximum value of angle of
deviation 0 and the corresponding value of
the angle e.
1,~~::I
••••
Um••••••'.m:•••um ••••,•••••••••••:.:.,' •••,.;•••••':.:•.,.., ••••;,';, •••• ::: •• : ".:', ••,•••:'.,'.,',; ••••':•••,', ••': '.'
A smooth sphere of mass m, is tied to a fixed
point by an extensible string and another sphere
mass m2 impinges directly on it with velocity u
in a direction making an acute angle a with the
thread as shown in the figure. The coefficient of
restitution is e. Find the velocity of the
suspended sphere of mass m2 just after the
impact.
PRACTICE PROBLEMS 23

~_. ---------------
'!Wo identical spheres, of mass mare
suspended by vertical string so that they
are in contact at the same leveL A third
sphere of same radius but of different
mass M falls vertically and strikes other
two simultaneously so that their centres
at the instant of impact form an
equilateral triangle in a vertical plane, as
shown in the figure. If u be the speed of
centre ofthe sphere Mjust before impact,
find the velocities of the suspended
spheres just after impact. The coefficient
of restitution is e.
[ill_' _
A hemisphere of mass M rests
on a smooth horizontal floor
and may move freely in the
horizontal direction. A sphere u
of mass m moving horizontally
with a velocity v strikes the e M
hemisphere at a point where
the common normal makes an -7-/-/-~-- """7 7
T T T
~ /

angle e with the vertical, as


shown in the figure. The
coefficient of restitution is e.
Find the velocity of the
hemisphere just after the
impact.

~---~-----------
'!Wowedge shaped blocksA and Beach
of equal mass M are placed on the
frictionless horizontal floor. A smooth A B
sphere of mass m 1l!0vingvertically
down with the velocity v strikes the o e
wedges symmetrically as shown in the 77~

figure. The coefficient of restitution is


e. Find the velocities of wedges just
after the c011isjon?
24 I . PRACTICE PROBLEMS

~rp~. i). f' ~:'~P


..~~ .It~l.. r,.
~i.,~.,~ _~ '._J-:~~~:_~:.~"TIt~
Consider an arrangement shown in the figure in which all the
contact surfaces are frictionless. The container 'N contains water
and its initial mass is mo. A trolly 'B' which can store water has
also mass mowhen it is empty. There is a hole at the bottom in the
container, which is when opened water leaks out at a constant
rate of fl kg/so The water falls on the trolly and comes to rest
relative to the trolly. Initially the system is at rest. The syst.em is
released and hole is opened at t = O. Find t.he velocit.y of trolly
after time t.

19Q If:~'..
"-, p -- -:-. ~:?"'* 41-.':::7-~_:~5~~
A spherical rain drop of radius ro starts falling freely receives
water particles during its fall, so that. its radius increases at a
dr .
constant rate -d = fl. The acceleration due to gravity isg Find the
t .
distance that t.he drop has fallen in time t1
191" .,1 t~,-~~~t-~~;J~' ~~ 'i"1..~.5rJ:~~~~%'i~~~
A disc of mass mo and cross-sectional area A
is being pulled by a constant force F in a
region where uniform cloud of dust particles
are moving towards the disc with a velocity u
as shown in the figure. All the particles
meeting the disc attach to it. The density of
the dust cloud is p. The disc starts moving
from rest at t = O.Find the velocity of the disc
as a function of time. .,
19.2":1
r.~V""t~i.~. ~~i~"
;'J,.t:-l.Ct~ .c: ~ Y.:7t:'t:
..~'?~i!~.1 !..•, l' ' ':f£~~.'
v, '.,,»,..s;:i?',-~1"".y...
I'.:.J.••, , .•.,.~.••.•.••••
."Ir'.~''';:'l'!.\tG''''
r-':;.At:•• .,:t~!t~_-;~"4
•....•.
:I'"". ..i1~~

A heavY particle of mass m is attached to one end of a chain of


length I and Same mass'm. The chain is placed on a horizontal
floor at rest. The particle is projected vertically upward with an
initial velocity va, as shown in the figure. The acceleration due to
gravity isg. Find the least.value of va so that the whole chain may.
leave the floor. "' I
mJJ>HhJf>O::nh
k-m
.
PRACTICE PROBLEMS . 25

Find the moment of inertia of a hemisphere of mass M and radius


R shown in the figure, about an axis AA: tangential to the
hemisphere. 'A

{Q:);
• M •
,,
,
!A'

The distances of the vertices of triangle of mass M, from an axis


AA: in its plane are a, band c as shown in the figure. Find the
moment of inertia ofthe triangle about this axis.
A

A C

A particle is projected horizontally along the interior of a smooth


hemispherical bowl of radius R. The particle moves on the helical
path of increasing radius as shown in the figure. The initial
position ofthe particle is at an angle Bwith the vertical. Find the
initial velocity Vo required for the particle to just reach to the top
ofthe bowl.
26 I PRACTICE PROBLEMS

[EJ_: _
'!\vo particles each of mass m, rest on a smooth horizontal floor
and are connected by an ideal non deformed spring of natural
length l and spring constant k. If one of the particle is given a
velocity Vo perpendicular to the line joining the particles as shown
in the figure, find maximum elnogation x of the spring in process
of motion assuming x«l.
~ to,
@-""'"'' ~ m
m

~-' ---------------
A vertical hollow cylinder is fixed on the ground. A uniform rod
can be balanced partly in and partly out of the cylinder with the
lower end of the rod resting against the vertical wall of the
cylinder, as shown in the figure. The angle marie by rod with the
vertical in equilibrium is e. If the maximum and minimum values
of e are ex and p respectively, then find the coefficient of friction
between rod and cylinder.

Eill _
'!\vo wheels are connected by an axle and are placed over a plank
as shown in the figure. The angle of inclination e of the plank can
be varied. It is found that the system just slips down when upper
wheel is locked at angle e = ex and when lower wheel alone is
locked slips down at angle e = p. Find the coefficient of friction
'between wheels and the plank.
PRACTICE PROBLEMS 27

9
19 1_" --------------

Consider an arrangement shown in the figure. The pulley P is


frictionless and the threads are massless. The mass or the spool is
1
m and moment of inertia ofthe spool is ZmR2• The mass of the
disc of radius R is also m. The surface below the spool is rough to
ensure pure rolling of spool. The mass of the block is m and the
surface below the block is smooth. Find the initial acceleration of
the block when the system is released from rest.

Spool

B
III

SmooUl

11001 _
Consider an arrangement shown in the figure. The surface below
the plank and block are frictionless. The pulleys attached with
the block are frictionless and the thread are massless. The conwct
surface between cylinder and plank is rough enough to ensure
pure rolling. A horizontal foice F is applied at the centre of mass
of the cylinder towards left. Find the initial acceleration of the
plank.
B1oc.k

III

11Ol1 ~
A wedge of mass M is placed on a smooth
horizontal plane and a cylinder of mass
m is placed on the wedge as shown in the
figure. The contact surface ootween
cylinder and wedge is rough. Find the
acceleration of the wedge when the
cylinder begins to roll down.
28 PRACTICE PROBLEMS
". !
11021' I

A light rope of length / passes over a pulley' of


mass M and radius R which can rotate abou t a
horizontal axis. The moment of inertia of the

pulley is iMR2 . Two monkeys A and B of equal


mass m are initially at rest at the lower ends of
the string as shown in the figure. The monkey A
starts moving up with speed v and simultaneously m

monkey B starts moving up with 2"v both relative


to the rope. Which monkey will reach the top first

h
and how much time it will take? Assume R« /.
11031 i-'.--- ~u_ -, . '1
;

A uniform rod oflength / is released


from rest from the position shown
in the figure. The acceleration due
to gravity is g. There is no friction l

at any surface. Find the initial ngg: ~>nn


angular acceleration of the rod.
1 4
1 0 1_:' ~------------~~----'
A uniform rod AB oflength / and mass o
m is suspended by two identical strings
OA and OB from a fixed point 0 as
shown in the figure. The rod is in the
horizontal position and each string
makes an angle 8 with the rod. If the Af:::::'=====~.
string OB is cut what will be the B
tension in the string OAjust after the
cut?
11051'
Au niform rod of mass m and length /
is standing vertically on a smooth
horizontal floor. A slight disturbance
causes the lower end A slip on the
floor. Find the velocities ofthe ends of
the rod at the instant it makes angle 8
with the horizontal as shown in the
figure.
PRACTICE PROBLEMS . 29

11061 _

A uniform rod of length I is placed in horizontal position on a nail


as shown in the figure. The rod is released from rest from this
position. Find the coefficient of friction between the rod and the
nail, if the rod starts sliding after turning through an angle G.
t-1/4
1-- 31/4 ==:I
11071 _

A rigid rod of length I and mass m joins


two particles each of mass m. The rod lies In
on a frictionless table and is struck at one
end by a particle of mass m moving with
velocity Vo as shown in the figure. After
the collision, the particle moves straight 0-
back. Assume that the collision is mvo m,
perfectly elastic. Find the angular velocity
of the rod after the collision.
11081 _

A disc of mass M is lying on a frictionless


horizontal surface. A small particle of
mass m strikes the edge of the disc with a
velocity Voin a tangential direction of disc
and gets embedded in it, as shown in the
figure. Find the angular velocity of the
disc just after the collision?
11091 _

A solid hemisphere of mass M = 8 kg and


radius R = 5 m is supported at the ends of
the diameter of the circular cross-section.
The supports are frictionless. A particle of
mass m = 1 kg is dropped freely from a
height II and strikes the end of the
diameter of the circular section
perpendicular to the diameter passing
through the supports and sticks, as shown
in the figure. The hemisphere just reaches
a position where the flat face of it is
vertical. Find h. Also calculate the
reactive impulse 'J' of the supports during
collision.
30 . PRACTICE PROBLEMS

11101 -'- _
A solid sphere of mass m is rolling on a rough horizontal surface
with velocity vo. It collides elastically with a cubical block of same
mass m at rest. The height of centre of mass of both the bodies is
same. Assume that there is no friction between the sphere and
the block. The coefficient offriction at all the surfaces is fl. Find at
what time and at what distance from first collision the second
collision will take place.
lt~ ~

~» ••
;;;;'~.n7

11111 _

A circular ring of mass M and radius R lies


on a smooth horizontal surface. An insect
of mass m starts moving round the ring III

with uniform velocity v relative to the


ring. Find angular velocity of insect with
respect to the ground. M

11121_' ----------------
A solid spherical ball spinning with an
angular velocity "'0 strikes a horizontal
floor with a velocity Vo at angle e with the
normal to the floor and rebounds as shown
in the figure. The coefficient of restitution
is e and the coefficient of friction is fl. Find
after ti,e rebound the angle $, the angular
velocity'" and the velocity v.
11131,_. ~ _
Two cylinders ofradii R, and Roare of equal
length and are made up of same material.
Initially cylinders are rotating about their
axes with angular velocities "'] and ",'2 as

00
g;~~~g'~i~!;
ceases due to the friction between them.
Find the angular velocties "'; and "'•. of the
cylinders after the slipping ceases.
PRACTICE PROBLEMS 31

11141 _

A satellite is projected into space from a


point P at a distance ro from the centre of
the earth at an angle C1. with the ~,,;
.•.
,,
horizontal as shown in the figure. The ,, ,,
velocity of projection equals to the velocity ,,, ,,
of the satellite in the circular orbit of ,, ,,,
radius ro around the'earth. It is found that ,, ,
the satellite is propelled into an elliptic , /
orbit. Find the length of semi-major axis -~-_..- --
'a' and eccentricity e of the elliptic orbit.

11151
A body is projected vertically upward from the surface of earth
with a velocity equal to the escape velocity. The radius of earth is
R and the acceleration due to gravity on the surface of the earth is
g. Find the time taken by the body to reach to the height h.
11161 _
The density p inside a solid sphere ofradius R and mass M varies as
p ex: .!, where r is the distance from the centre.
r
Find the gravitational potential V at the centre of the sphere and
also find the internal potential energy (self energy) u of the
matter forming sphere.
11171 _

A uniform rod of length 'I', mass 'm' cross


sectional area 'A' and made up of a
, '"
material of young's modulus Y is rotated <.1.s
about one end with an angular velocity'" I
,,
as shown in the figure. Find the ,
elongation 6/ in the length of the rod,
assuming III « /.
11181 . _
A cone of circular cross-section having
base radius R, height / and mass m is
suspended from its base as shown in the
figure. The material of cone has Young's
modulus Y. The acceleration due to
gravity is g. Find elastic potential energy
stored in the cone.
32 PRACTlt:E PROBLEMS

11l9!....i _-_. ~ _
A tank of rectangular cross-section is
filled with a liquid of density p upto height 'I
H. There is a semicircular gate ofradius R
hinged along the base line of the container
as shown in the figure. Find the torque
acting on the gate about base line. The
I
H

acceleration due to gravity isg.


11201_i _
A right circular cone of semi-vertical angle (J. can float vertex
down in water with generator vertical and the base just clear of
the water as shown in the figure. Find the relative density of the
material of the cone.

1\\'0 cylindrical containers of cross-sectional area Al and Az are


connected together at their bottoms by a thin pipe of uniform
cross-sectional area a as shown in the figure. Initially the
container of cross sectional area Al is filled upto height H with a
non-viscous incompressible liquid. Now the tap is opened and
liquid flows from one container to the other container. Find the
time taken to equalize the level of liquid in both the container.
The acceleration due to gravity isg.
PRACTICE PROBLEMS 33

11221_. --_._ .... _.... _..... _.. -----------

A cylindrical container of radius Rand


height H is filled with a non-viscous
incompressible liquid upto a height h as
shown in the figure. Now the cylinder is
rotated about its axis with a constant
angular velocity <D. Find the maximum
value of <D for which the water will not
spill over.

A mercury drop shaped as a round tablet


of radius R and thickness h is placed
between two horizontal glass plates. Now
a mass m is placed on the upper plate and
due to its weight the distance betwoen the
plates is diminished n times. The surface
tension is T and acceleration due to
gravity is g. Find the angle of contact 8.
(Assume is h «R)
1 1' -- ._ ..._- .... _.~. '~~'-.- . --_._- .__ .,- - - '--"-
124
The surface tension of the soap water solution is S and the
atmospheric pressure is Po. Find the work to be done in order to
blow a soap bubble of radius a under the isothermal condition.

11251_' -_ ... _--_._- ---=--=---'--"-'--=--'-"-- -----" ---"'----' '-'-" -' '-''-"---'--'


The space between two coaxial cylinders of radii a and b is filled
with a liquid of coefficient of viscosity '1. The length of cylinder is
l. The outer cylinder can be rotated about its axis. Find the torque
required to rotate the outer cylinder with a constant angular
velocity Ill.
34 . ,
PRACTICE PROBLEMS

A viscousliquid is fbwing through a pipe ofcross-sectional radius


R The volume of liquid flowing per unit time is Vo' Find the
volume of the liquid flowing per unit time through the section
R
upto radius '2 from the axis of the pipe.

A solid cone of semi-vertical angle ex. and height h is suspended


from its vertex as shown in the figure. The acceleration due to
gravity is g. Find the time period of small oscillation if the cone is
slightly displaced from its vertical position.

A block of mass m is dropped from a height h on the free end of a


vertical spring of force constant K as shown in the figure. The
block first compresses t.l1espring and it leaves the spring when
spring stretches to its natural length. Then the block rises to its
original height. Find the time period of oscillations. The
acceleration due to gravity isg.
PRACTICE PROBLEMS 35

I~~I
••••.••.•.....
\ .•.................•.•.•.•.•.•.•...
A thin uniform rod is hinged at one end A and the other end B is
supported by a vertical massless inextensible thread oflength l as
shown in the figure. Now the end B is slightly displaced. Find the
time period of small angular oscillations of this system? The
acceleration due to gravity is g.

11 1..:.:.................................
.••.•••...•
"'
..•.•
0011.
..........•......
':':':',:,',':::::::':::':':,:,:,:,:,:,:,':';':':::':=::":'
.............•.. •.•.• ...............•....
-.-.-..... :.;:=:::::::::::::;::':'::;: :':::,:,:,:,:,=,:,'::,:;::,'=, ....
•..
A symmetrical block of mass M having a notch of hemispherical
shape of radius R is placed on a horizontal surface. A small
. particle of mass m is placed inside the hemispherical notch and
can slide without friction. Find the time period of small
oscillations of the particle and the block. The acceleration due to
gravity is g.

M ",

mm
....•...........•..•.•.•.•.•....•.•..•..•••••••••••
~ 13 :.••••••••••..•....................••.....•.•.•.•
..
:::::::::::::::'::::-: :
:•.••.••••.•••••••••.•••••••••••••••••.•.•.•.••
::.:.:
....
:.:.,.:.;.:.:.: :.;.:-:.:.:-:,:::,:::::::::.;.-.-"

A cylindrical rod of length (l, + l2) and made up of a material of


density P2 is in equilibrium with the upper end at the surface of
the upper liquid. The densities of upper and lower liquids are p,
and P3 respectively. The densities p" P2and P3 are in geometric
progression. The acceleration due to gravity is g. Find the time
period of small vertical oscillations of the rod.
p•

........
........
36 PRACTICE PROBLEMS

/1321_'._-_0._-_"_- __ ._0_._------_._ .. .0. --'-'_.---"-


Consider an arrangement shown in the figure. There is no friction
between wedge and the horizontal surface but there is sufficient
friction between cylinder and the wedge so that the cylinder can
roll without slipping. Find the time period of small oscillations if
the cylinder is slightly displaced downward from equilibrium
position.

11331:-.- -- ~----,,",," ....-=-- ------------.- •• '"-;

Two SHMS x = a cos 3rot and y = b sin rot along x and y-axis
respectively are supposed on a particle. Find the equation ofpath
traced by the particle in x-y plane and also plot the graph?
1134Ii~-- '1 ••.• -1
Consider a system shown in the figure. Find the angular
frequencies of possible oscillations along the vertical direction.
PRACTICEPROBLEMS 37

_un .....•.•.•.••••.•..•
H .•.• [.1
ItMI?'................
A simple harmonic transverse wave of amplitude 8 mm travels in
the direction of positive x-axis. At t = 0.1 second, for a particle at a
distance of 10 cm from the origin the displacement is 6 mm, and
for a particle at a distance of25 cm from the origin the displacement
is 4 mm. Find the equation of the wave. (Given sin 49° - 0.75)
I~*~I?).............
A snap shot, of a vibrating string at t = 0, is shown in the figure. A
particle at point P is observed moving up with velocity (20.,[2 ,,) cmls

and the tangent at the point P makes an angle a = ton.1 (2.,[2,,)


with the positive x-axis: Find the equation of the wave.
y (em)
4

The equation of a transverse wave is given by


'1'= (1Q-') sin ,,(lOt- .{3x-y)
where x, y and'!' are expressed in metre and t in second.
Find (a) the direction of propagation of the wave
(b) wavelength and
(c) phase difference between two points A (.{3 m, 1m) and
B (2.{3 m, 2m).
~ ..
~ •.•
??"' ••'."" , ,..".,.,.".,.,.,.".".,.,.,.,"""",',',',"',',.,.,
•••••
"•••••••••••
- -.-
>' ',','..
;.-

The equation of a plane longitudinal wave travelling through a


homogeneous gaseous medium of density 1 kg/rna is given by
'I' = (10-9) (lOOOt- 5x)
where x and'!' are in metre and t in second. Find (a) intensity of
the wave and
(b) the amplitude of pressure variation.
38 PRACTICE PROBLEMS
I
11391.
The shape ofpulse of a plane progressive wave is given by an equa-
b3
tion y= 2
b2 +(x-ct)
Find:
(a) the maximum value of particle velocity up att = 0 and also plot
the graph showing variation of up with respect to x.
(b) the equation of this wave in a reference frame moving in posi-
tive x-direction with a constant velocity u.

A source of sonnd is placed in air at some


height above the air and water interface. :4: Source
The temperature of air and water is 200 C. Air

c ::::::::::::::::::: :Wt\t&r::::
The mean molar mass of air is 28.8 and ...J!..
c.
= 1.4. The bulk modulus ofwater is 2.088 x
10' Pa and density of water is
1gm/c.c. Consider only refraction ofsound
at the interface. Find the percentage of
power transmitted from air to water?
11411
A string of mass 0.8 kg/m is stretched to a tension 500 N. Find the
mean power required to maintain a travelling wave of amplitude
of 10 mm and wavelength 0.5 m.
11421'
Two strings of mass per unit length
~, = 0.2kg/m and ~2 = 0.8kg/m are joined
together and stretched between two
supports as shown in the figure. A
sinosudal wave of amplitude A is
incident from the side AO and ~ 0 ~B
transmission and reflection takes place I.
at the interface O. Find --"-, - --,- ••--
(a) the amplitude of the transmitted
wave.
(b) the percentage of incident power
transmitted to the second string.
PRACTICE PROBLEMS • 39

1
1431 _

A stretched string of length 1 clamped at


both the ends has a tension T. Now the ~
string is pulled aside a distance h (<< I) at 1------]1;-----
its centre and released, as shown in the :)
figure. Assume that the tension remains
unchanged by small transverse
displacement. Find the energy of
subsequent oscillations.
1144 1 _

Two sources separated by 12 m vibrate


according to the equations
y 1 = (3 sin 41tt)cm and y 2 = (4 sin 41tt)cm.
)( 8m 4m
They send out the waves with speed 32 m/ 8, I
p
82
)(

s. What is equation of motion of a particle


P, 8 m from first source and 4m from second
source as shown in the figure?
11451 _
Consider a closed pipe of length
1 located along x-axis with closedend
at the origin as shown in the figure.
The density of air is p and the
velocity of sound is v. The pipe x _0
resonates in nIh overtone with
-:-(---------------1jIr;K
-"'
,.
displacement amplitu de A. Find the
expression showing pressure
variation at point (x, 0).

11461"_"_" -----------------
A metal rod AB of length 1m is
clamped at two points P and Q as
shown in the figure. Find the
A, .
P
.1

B
Q
I
I

maximum possible wavelength for 1 5cm 1 BOem 15c1ll


the natural longitudinal oscillations
of the rod.
40 PRACTICE PROBLEMS

11471 _

Consider a situation shown in the


figure. Source and observer both are
moving towards the wall with
velocities v. and Vo respectively. The
wall is moving away with velocity u. -OJ
The wind is blowing with a velocity OJ v, 0, u
towards the wall. The frequency of the
sound emitted by the source is no and
the velocity of sound with respect to
air is v. Find the frequency and
wavelength of the reflected waves
received by the observer.
148
1 1_: -----~------------
At a particularinstant a source of sound offrequency 1000 Hz is at
(2m, 1m) an.d an observeris at(5m, 5m). The velocity of source and

observer aniaO (i + J) mls and 15 (i + .J3J)


mls respectively at
this instant. The velocity of sound in air is 330 mls. Find the
frequency a!1d wavelength of the sound received by the observer.
11491_.' _
, The edges of a square plate of side
length I are clamped and transverse y
waves are set up in it. The coordinates
of the corners of the square plate are
shown in the figure. Take z-axis along (0,<):jLL.==="';«'.I)
the direction of displacement. Take
t = 0 at the instant the centre of the
plate passes through its mean position.
The maximum amplitude of oscillation hz77n'777:77n-d<---X
is A and the plate is vibrating in its (0,0) (1,0)
fundamental mode. Write the
equation describing the standing wave
in the plate ?
150
1 1_' ,----------------
A uniform rectangular membrane of length 20 em and bredth 10
em is clamped at its edges. The mass of the membrane is 20 gm.
The surface tension in the membrane is 5 kg/em. Find the
fundamental frequency of oscillation.
PRACTICE PROBLEMS 41

_WI
I~~~IHH.
Tworods ABand BC ofequal cross-sectional area are joined together
and clamped between two fixed supports as shown in the figure.
For the rod AB length is l ,coefficient of thermal linear expansion
is lXI' Young's modulus is VI and density is Pl. For the rod BC, length
is l" coefficient of thermal linear expansion is a" Young's modulus
of elasticity is Y~and density is Pr Now the temperature of the
compound rod is mcreased by 6. Find the time taken by a transverse
wave pulse to travel from end A to other end C of the compound
rod. Assume that there is no significant change in the lengths of
the individual rods due to heating.
I,
I
A~ B m, ~c

Consider a composite rod shown in the figure.


The radius of inner rod is a and outer most
radius is 2a. The Young's modulus and ,
coefficient of thermal linear expansion for ~~~J'l'
inner rod are Yl and lXl and that for outer rod ...-------L-.
J)
are Y, and a, respectively. The length of rod is . "
l. Find the final length of the rod, if the I
temperatore ofthe compositerodisincreasedby 6.
li~I......U.:E.. ..
A long horizontal glass capillary tube open at both ends contains a
mercury thread 1m long at 20° C. A scale is etched on the glass
tube. The scale is correct at 20° C. Find the length of mercu5f,
thread as shown by this scale at 120° C (It is given lXgI." = 9 x 10 I
K and 1menury = 18.5 X 1O-5/K) .

M",I
A tube of uniform cross-section is
A B
divided into two parts by a movable
VI Kl V, K,
conducting separator. The two parts of
the tube are filled with two different
liquids A and B. For the liquid A,
-, ., Y,

volume, coefficient of thermal cubical


expansion and bulk modulus are VI' 1, and K, respectively and
those for liquid B are V" 1, and K, respectively. Find the final
volumes of the liquids A and B if the temperature of the system is
increased by 6. Neglect the expansion oftube.
42 PRACTICE PROBLEMS

11551 - -- -----"" "-----" "-"----" ------

An ice cube ofmass Mois given a velocity


Vo on a rough horizontal surface with
coefficient of friction 1'" The block is at
its melting point and latent heat of ~_ •• _
fusion of ice is L. The block receives heat ",,"","",
only due to the friction forces and all
work is converted into heat. Find the
mass of the remaining ice block after-
time t.
Iilll_- _
A container of mass 0.5 kg contains 1kg ice at - 23° C. The specific
heat ofcontainer varies with temperature T according to the relation
8 =A+ BT, where A= 100callkg-Kand B =2 x 10--'callkg-K'. The
specific heat of ice is 0.5 caVgm-K and the latent heat offusion of
ice is 80 caVgm.Find the mass ofsteam at 100°C required to convert
the ice in the container into water at 27° C. The latent heat of
vaporisation of water is 540 caVgm and specific heat of water is 1
caVgm-K.
11571_- _
A rod AB of length 1with thermally insulated lateral surface has
uniform cross-section" The coefficient of thermal conductivity of
1
the material of the rod varies with temperature as K ex: T . The
ends ofthe rod are maintained at temperatures T, and T,. Find the
temperature T at a distance x from the end whose temperature is T,.
~I T
J
Twoblocks are connected by a metal rod
of length I, area of cross-section A and
Thermal conductivity K. For the left
block mass is m, and specific heat is 8,
and those for right block are m, and 8,
respectively. At t = 0 the temperature of
left block is T, and the temperature of
right block is T,. Find the temperatures
of the blocks after time t ?

.\
PRACTICE PROBLEMS 43

Consider a composite cylinder shown in the


figure. The conductivity of the material
the cylinder from radius a to 2a is K, and
that for the cylinder from radius 2a to 4a
is ~. The temperature of inner most wall
is maintained at temperature T, and the
outermost wall is maintained at
temperature T2• Find the temperature Tof
the intermediate cylindrical wall of radius,
2a.

A thin spherical shell ofradius R made up of


highly conducting material contains water
at 0° C. The temperature ofthe atmosphere
is ~ 8° C. The thermal conductivity ofice is K
and the latent heat of fusion of ice is L. The
density of water is p. Find the time required
to freeze the whole mass of water. Neglect
the expansion during freezing.

Abody coolsaccording to Newton's Law ofcooling.The temperature


of the body cools from 8, to 82 in time t. The temperature of
surrounding is 80, Find the time taken for the temperature of the
body to decrease from 8, to 8, .

A cylindrical container of cross-sectional' area A and height h


a
contains gas at a constant temperature. The density p ofthe gas
varies due to acceleration due to gravity. The density of the gas at
the bottom is P, and at the top becomes Pi' Find the mass ofthe gas
in the container.
A

I
;
P,
---- ...
--_ .. _ ......... -
-_ ..__ ..........
--_ ....
.. _-----_ h
.... ..........
__
_._-_ .. _._--
...._. __ ..-- , ...
P,
..'-.
j
44 PRACTICE PROBLEMS

1163 1 -._-_-
.._-_. -_~_._-
__ l

A vessel of volume Vocontains an ideal gas at


a preswre Po, Gas is continuously pumped out

of this vessel at a constant rate c:;; = r,


keeping the temperature constant. The
pressure of the gas taken out equals the
pressure inside the vessel. Find the pressure
of the gas inside the vessel as a function of
time.
11641_, .-~ .. _-_~-_._. -- 1
Find the speed of sound in a mixture ofgases containing 3 moles of
Re, 2 moles ofR, and 1 mole of CO, at temperature 27° C.
1 165!_r ----~_._-~_._- '-'--'-' ---------
An ideal gas is taken along the process AB shown in the P-V
diagram. Find the volume of the gas where the temperature
becomes maximum.
I'

11661;" ,- .•~
~ .....• ...- .•. -,""
" ----:.-.."',~.. "':"''':
~ )J

'.An ideal monoatomic gas is taken along the process AB shown in


. the P-V diagram. Find the volume of the gas where the heat flow
reverses its direction.

1"

. 32p" A

~ ~~~~~n
, ,
:
8""
'
64"-v,--

"", ." ... '


PRACTICE PROBLEMS , 45

One mole of an ideal mono atomic gas is taken around a cyclic


process shown in the figure. The processes AB, BC, CD and DA are
isochoric, adiabatic, isobaric and isothermal respectively. It is given

TA = 4TB and VA = (s.J2) Vn. Find the efficiencyofthe cyclic process.


D

A
V

One mole of a mono atomic ideal gas is taken around a cyclic process
shown in the figure. It is given TB = 4TA. Find the efficiency of the
cyclic process.
p

An amount Q of heat is aded to a mono atomic ideal gas in a process

in which the gas performs a work ~ on its surrounding. Find


(a) equation' of the pr'lcess, ar.d
(b) molar heat capacity for the process.

In an adiabatic container the molar specific heat at a constant


volume of an ideal gas varies as
Cv = (a + bTl, where a and b are constant and T is temperature in
Kelvin. .
.Find the equation ofthe process. The universal gas cOllsta:ntis R.
46 PRACTICE PROBLEMS

11711_.. ~ __ ~_
A cylindrical container of cross-sectional
area A contains an ideal gas at atmospheric p. k
pressure Po and temperature To' The
pistons used are massless. Initially the
spring is unstretehed and the separation
between the pistons is h as shown in the ..............
_.. -.'f,j' .
T
h
..............
figure. The force constant of the spring is
k. Find the work done in displacing the
lower piston upward by a distance h!2, ]',
1
keeping the temperature of the gas
constant.
11721 _

An insulated container divided into


three parts contains an ideal gas
(y = 1.5) at the same pressure Poand II
temperature. Initial volume of each
part is Vo' The pistons I and II are
connected by a non-conducting rigid
rod and can move without friction
in side the cylinder. Piston I is
conducting and the piston II is non-
conducting. The leftmost part is
slowly given heat through a heater
such that the final volume of the
4
rightmost part becomes "9 Vo' Find
the amount of heat supplied by the
heater?

A simple pendulum with a bob of mass m,


charge q and string length I is given a
horizontal velocity u at its bottom most point.
Ahorizontal electric field of strength E exists
in space. The acceleration due to gravity is
g. Find the minimum value of velocity u so
that the particle may complete the vertical
circle. .
PRACTICE PROBLEMS ' 47
It14-IH:,••.•
,•.•••............................................
<.«»..H!H!:..
A positively charged particle of charge q and mass
m is suspended from a point by a string oflength I.
In the space a uniform horizontal electric field E
exists, The particle is drawn aside so that the string g I
becomes vertical and then it is projected
horizontally with velocity v such that the particle
starts to move along a circle with same constant
speed v. Find the speed v.

Twocharged particles of masses' 2m and


3m and having equal charges +q are 2m 3m
released from rest at distance of "(":?,~n""''''\(&&:?'''''n''
separation d on a frictionless horizontal I-- d --I
surface as shown in the figure. Find the
time -after which the distance between
the charges will become 3d,
Ir7.•...•.
6I ,:.,•.,•.•......••..•.••..•..•..
:...•.•.•.•.•.•.•.•.•.•.....
:.:.•.
•.:,.:.•.
.:•..'.•.
•.:.•...•.•........
,.,•..•..........•...••...•.•
,...•.•.•.•..•..................................
,....................•..
.......... ,.,.,.,.:.:.:.,.,.:.:.:::,:,:::::::::=::::::::=;,:,-:.'.
A charged particle of charge +q and mass m is projected towards
another fixed charge + Q from a very large distance with initial
speed v 0 with an impact parameter b as shown in the figure. Find
(a) distance of closest approach of the two charges, and
(b) the angle of diviation 9 when the +q charge finally goes to a
very large distance,

b
+Q
---------------,--

A small dipole of dipolemoment p


is placed at the origin oriented
along the y-axis, Find the potential
V and the magnitude of electric
field E at the point P (r, 9, <1»as
//y
shown in the figure,
, , '/
- - -- _.__ .- - --- - -_',1/'
.•.
x
4~, " PRACTICE PROBLEMS

A thin non-conducting ring of radius a


is placed in x-y plane with its center at
the origin 0 as shown in the figure, A z
charge +q is distributed uniformly over
one half of the ring and a charge -{] is
distributed uniformly over other half of y
the ring, Find the electric field intersity
x
vector It on the axis of the ring as a
function ofthe distance Zfrom its centre,

Twothin rods each oflength I and uniform linear change density 1.-
are arranged as shown in the figure, Find the force of interaction F
between the rods,
I
'/2

'/2
I

A thin disc of radius R carries a uniform surface change density <:5,


Find the potential energy of the di8c,

The surface of a,cone of slant length I as


shown in the figur3 is charged uniformly
with the charge Q, Find the potential at
the vertex of the cone,
PRACTICE PROBLEMS " 49

The half part of a hemisphere of radius R, as shown in the figure is


uniformly charged with a surface charge density 0. Find the
magnitude bf electric field E at the centre O.

..•..•..•... "
,,
,,
,
,/ ----... \
,/ - ..•... '
R -----------1
,/,/,/'0 ,,'/

-------

The electric field in a region is radially outward and is given by


E = ar, where a is a constant and r is the distance from origin.
Find the charge contained in a sphere of radius R centred at the
origin.

a(xi + yJ + zk)
The electric field in a region is given by It =
3/2
( x2 + y2 + z2 )

Find the flux through the surface of a cube of side length I centred
at the origin.

A solid conducting sphere ofradius


R is surrounded by a hollow
conducting sphere of inner radius
2R and outer radius 3R which in
turn enclosed by a thin conducting
spherical shell of radius 4R as
shown in the figure. The innermost
sphere and outermost spherical
shell are connected together by a
conducting thin wire. The outer
shell is given a charge equal to
90~C. If the middle sphere is
1:
grounded by closing a switch S, "'"
then find the charge on the outer
surface of the outermost shell.
50. , PRACTICE PROBLEMS

11861_: .. '_-_' "'_' "_1-


Three identical parallel conducting plates
X, Y and Z are placed as shown in the
figure. Switches S, and S, are open and
canconnect X and Z to earth when closed.
The charges 60 ~c, 100 ~c and 30 ~c are
given to the plates X, Yand Zrespectively.
(a) IfS, is closedwith S, open, how much
charge will flow from plate X to
ground?
x .Y ,z
(b) IfS,is closedwith S, open, how much
charge will flow from plate Z to
ground?
(c) If S, and S, are closed together, how
much charges will flowfrom plates X
and Z to the ground through S, and
S, respectively?
11871'_1 . ~ ~'
1\vo conducting spheres of radii a ~ ~
and b are separated by a large ~ ~
distance d. Find the capacitance of i i
the system under the condition I d I

d »a and d » b.
11881 .~._._'._1
Consider an arrangement of six large parallel plates as shown in
the figure. The emf ofthe battery is V.Find the electric field between
the plates numbered (1) and (2).

d 2d d 2d d
PRACTICE PROBl-EMS . 51

I~~~I~:
~~~~~~~~~~~~~~~~~=
A parallel plate capacitor consists of two
square plates of side length a. There is
dielectric slab in the shape of a cylinder of
parabolic cross-section between the plates, K
as shown in figure. The dielectric constant
of the material is K The length of latus ____ a __ -'
rectum of the parabola is equal to the side
length of the plate. Find the capacitance.

Two identical rectangular conducting large plates len.gth I and


breadth (b - a) are arrangeed as shown in the figure. The
between two plates is ex. The emf of battery connected across the
plates is V. Find the charge on the plate connected to the positive
terminal of the battery .

,,
.r- "rei:__
,,
,,
,,
,,
,,
,,
,,
,,
I _--
~ __ :: _1-Cl:.. _

I a b I

Find the equivalent capacitance of the circuit shown in the


between points A and B. The capacitance of each capacitor is
121 ,n.
A
52' ' , PRACTICE PROBLEMS

I~~~'ili)'
Find the equivalent capacitance of the circuit shown in the figure
between points A and B. The capacitance of each capacitor is 471lf.
A
,

"
1= "

f= 1= 1= f=
"
B

Consider a network shown in the


figure, Initially the switch S was
open. If the switch S is closed now,
then find (a) the amount of charge
flowing through the switch. (b)
the amount of heat loss.
210V
I~~~I»'
Initially charged three capacitors
and a battery of 100 V are arranged
as shown in the figure. Now
the switch S is closed. Find
(a) the final charges on the
~OH~h
I~F 2"F 3j
capacitors and (b) the amount of
heat loss,
h
IOOV

A parallel plate capacitor made up of two


rectangular plates oflength l and separated by
a distance d is charged to a potential difference
V and then disconnected from the battery. Now -...: d _
the plates of the capacitor are held vertically
and the lower ends of the plates are brought in
contact with the surface of a dielectric liquid of
density p and dielectric constant K It is found ===,K===== .
----------
that the level of liquid rises inside the plates
as shown in the figure. The acceleration due to
gravity isg. Find the height ofliquid level inside
the plates of the capacitor, .
PRACTICE PROBLEMS ' 53

1~~~ljH nH2;.""•.•..
c=<=H=T:=,>;=""c=;'=... =.,...=.. =;===""="'=
A parallel plate capacitor with air as
dielectric is arranged horizontally. The
lower plate is fixed and the upper plate is
connected with a spring offorce constant
m
K as shown in the figure. The area of each
plate is A and the mass of the upper plate
is m. The capacitor is connected with an
electric source of voltage V. In steady
position, the distance betwen the plates
is d. Now the upper plate is sligntly
displaced in the vertical direction.
Find the angular frequency of small
oscillations.

A long conductor of circular cross-


section has radius r and length I as
shown in the figure. The conductivity
of the material near the axis is cr1 and
increases linearly with the distance ----{)- -------::---l-)I_
from axis and t ecomes cr, near the I---- I ----I
surface. Find the resistance of the
conductor if the current enters from
the one end and leaves from the
other end.

A long prismatic conductor or of varying rectangular cross-section


and oflength I is shown in the figure. The dimension of rectangular
cross-section on the one end is a x b and on the other end it becomes
c x d. The side length of rectangular cross-section varies linearly
with the distance from one end. The resistivity of the material of
the conductor is p. Find the resistance ofthe conductor, if the current
enters from the one end and leaves from the other "nd.

,,
,,
,,
a
, --- "".. , ,
I c
---1--- " ,
", "
~ •..•
I
54 PRACTICE PROBLEMS
I~~~I
••.•.
/> ...••....•........
Consider an infinite ladder of net work shown in the figure. Find .-
(aJ the equivalent resistance between the terminals A and B, and
(bJ the current through the resistance R connected between points
C andD.

?J.
" ,tt:r:!l B H D nR n2R n3R

A frame made of thin homogeneous wire of resistance per unit


length r is connected with a battery of emfE between the terminals
A and B as shown in the figure. The frame ABC is in the form of
equilateral triangle and the number of successively embeded
equilateral triangles (with sides decreasing by half) tends to infinity.
The side length of equilateral triangle ABC is a. Find the current
flowing through the battery.

Consider an arrange-
ment of cells shown in
the figure. There are Enr I
m-branches and in each
branch there
n-cells. Suppose this
are

combination of cells can


be replaced by a single
2E
nr T
m branches

battery of emf V and


internal resistance R,
mE
nr _J
then find V and R.
PRACTICE PROBLEMS 55

1~(l21=,'~~~~~==~=====~=
Consider an arrangement of cells shown in he figure. Suppose this
combination of cells can be replaced by a single battery of emf E
and internal resistance r, then find E and~.
ov . 20V

10 20
~ 50V

A 5n B
40V
4,l

In the circuit shown in the


figure; the switch S was initially
open. Now the switch S is
closed (I = 0). Find the current
through the switch as a function
oftime.

In the circuit shown in the figure c, = 3 f!F,C2 = 6 f'F and C3 = 9 f'F.


Find the charges q" q2 and q3 on the capacitors c" C2 and C3
respectively in steady state.
50

24Vq~,~C, 30
q,
2H2 - +

. c\
+ IV
10: q2 C2

.,
56 PRACTICE PROBLEMS

205
1 1_' -------------------
A capacitor of capacitance C filled
c
with a resistive material of effective
resistance r is connected with a
battery as shown in the figure. Find
the charge on the capacitor and
current through the battery as a
function of time. Assume that the
switch is closed at t = o.
12061 _

Initially switches 8, and 8, were closed and 8, was open. Now the
switches 8, and 8, are opened and 8, is closed (at t = 0). Find the
potential differences V, and V, across the capacitors c, and c,
respectively as a function of time t.

...•.... .__ .~.~ I


12071
-------------------------
A thin wire bent into the form of a semicircle of radius r carries a
current I. Find the magnetic field of induction at the point P as
shown in the figure. I

G..
,,
,
,, ,,
, ..•..
.•.._J._
p
120 81:-------------------------
An infinitely long wire is bent into the form of a parabola of focal
length a. The wire carries a steady current 1. Find the magnetic
field of induction at the focus F as shown in the figure.
J
PRACTICE PROBLEMS 57

A square frame made of a thin wire carries a current 1. The side


length ofthe square is t. Find the magnetic field of induction at the
point P on the axis at a distance x from the centre of the square.

"
,
,,
"' ~---
,,," J
------------1
'1.///y."',.. . x P

VI .

A wire carrying current I and in the form of a semicircle of radius


a lies in the y.z plane with its centre at the origin 0 as shown in the
figure. There is a point P on the x-axis at a distance x from O. Find
the magnetic field induction vector It at the point P.

x p x

..
A frustum of cone having end
radii a and b and slant length \-=.::::::::; ,- , \1
I is wound with N turns ofwire
carrying current I, as shown .~:
;J-
_- , ,l'
in the figure. Assume 0 '", ,,
continuous distribution of -',,"
wires over the surface. Find N
the magnetic field ofinduction
B at the vertex 0 of the cone.
58 ~ PRACTICE PROBLEMS

An infinitely long tape of width b carries a current I which is


distributed uniformly over its width as shown in the figure. Find
the magnetic field ofinduction B due to this tape at a point P which
is on the bisector position of the tape. The distance ofthe point P is
'a' from the .tape.

--
--
An infinitely long wire of uniform circular
cross-section carries a current whose ,,
density varies as} = br", where b and a.are ,,
constants and r is the distance from axis of :-rl
,
the wire. Find the magnetic field of ,,
induction as a function of distance r inside ••,
the wire.

Consider a toroid of inner radius a and outer radius b as shown in


the figure. The total number ofturns of wire isN and the current is
1. The cross-section of toroid is circular amI the diameter of the
circular cross-section is (b - a). Find the magnetic flux through the
cross-section of the toroid.

N gE_a
Cross.sectionai
PRACTICEPROBLEMS 59

12151
--------------------------
A circular loop ofradius a and carrying a current I, :
is placed near an infinitely long straight wire :,
carrying a current I, as shown in the figure. The :
straight wire and circular loop are coplanar. The ) I !
0):
distance ofthe centre ofthe loop from the wire is b. :
, : 12
Find the force of interaction between the straight :,---,
b
wire and the loop.
12161 _

'I\vo straight wires oflength I, and I, are placed in


a plane parallel to each other of a separation of
distance a, as shown in the figure. The currents
are I, and I, in the wires of lengths I, and I,
respectively. The mid-points of the wires are at the
'I '
' ------
JI a I,
'

same level. Find the force of interaction between


the wires.

12171
--------------------------
A semicircular wire of radius 'a'
carries a current I and lies in a
horizontal plane (x-y plane) with its •
ends P and Q hinged on the y-axis I
as shown in the figure. A uniform g

magnetic field it = - B t exists in y

the space. The acceleration due to


gravity is g. If the loop is released x
it is fou nd to stay in horizontal
position in equilibrium. Find the
mass ofthe loop.
12181 _

A square loop of side length I and carrying


a current I, placed near an infinitely long
straight wire, as shown in the figure. The
straight wire car:ies a current I,. The plane
of the square loop is perpendicular, to the
plane containing straight wire. Find the
magnitude offorce F and torque T acting
on the loop.

"
60 ,PRACTICE PROBLEMS

2191_' ------------------
1
A particle ofcharge q and mass m is projected on a rough horizontal
plane (x-y plane) from a point (a, 0, 0,) with initial velocity It =
voJ, In the space uniform electric field it = - E k and uniform
magnetic field B=> = - B k" exists, The acceleration due to gravity is
g, The coefficient offriction between the particle and the plane is
fl, The particle moves on a spiral path and finally reaches to the
origin 0, Find the time taken by the particle to reach to the origin,
~
E I I B

,,
,~4-------------------, ''
I ,/

, ,
y

,/
/
,--------------- ---Vo
I (n, o. 0)
2201_:
1 ----------------~-
A particle of charge q and mass m is
°
projected at t = from origin with velocity
z

~ ="0 (sin CJ. J + cos CJ. k), A uniform


magnetic field it = B k exists in the
space, Neglect the effect of gravity, Find
the velocity vector It and position vector
r' as a function of time, x
12211 '_"~_-_~--_'-'
The seperation between the plates of a parallel plate capacitor is d,
A battery of emf V is connected across the plates, A uniform
magnetic field B exists in the region between the plates as shown
in the figure, A positively charged particle of charge q and mass m
starts from the positive plate ofcapacitor with zero initial velocity,
Find the minimum value of V for which the particle can reach to
the negative plate, Assume that the space is gravity free,

d Bo
1
,000
0 00
000
00 00
oJ
o~
0


PRACTICE PROBLEMS . 61

222
1 1_.' ----------------
A cylindrical capacitor consists of two
coaxial cylinders of radii a and b as shown
in the figure. A battery of emf V is
connected across the cylinders. A uniform
magnetic field B exists in the region o 0

between the cylinders. A positively charged o


particle of charge q and mass m leaves the o 0

inner cylinder with zero initial velocity. o


Find the minimum value of V for which the
particle can reach to 'the outer Lylinder.
Assume that the space is graVity free.
12231~,. ~ _
A circular ring of radius a and resistance R is placed (fIxed) in x-y
plane with its centre at origin as shown in the figure. A magnetic
"" _
J A
field B = (a. x)f + <p y) + (y t) k is switched on at t = O.Find
the force acting on the ring.
y

12241 _

Consider a frame made up of uniform wire shown in the figure. A


time varying mt .;netic field B = b t is switched on. Find the charge
on the capacitor in steady state .

••..--- 5, ----.
62 PRACTICE PROBLEMS

A right circular solid cone is made up of


non-eonducting material. The axis ofthe
cone is along z-axis as shown in the
figure. The mass of the cone is m. A
charge q is distributed uniformly over
its surface. A time varying magnetic
field It = bt k is switched as at t = O.If
the cone is free to rotate about its axis,
y
find angular velocity It of the cone in
x
terms of magnetic field vector It .
122~~"
"_-'- ""._-_""-_-_". ~_
A long hollow cylinder oflength I and
radius R has total chargeQ distributed
uniformlly over its surface. The mass
of the cylinder is m. The cylinder rolls
down on an inclined plane of
inclination 8 with the horizontal as
shown in the figure. The acceleration
due to gravity is g. There is sufficient
friction between the cylinder and the
inclined plane to ensure pure rolling.
Find the acceleration of the centre of
mass of the cylinder.

12271:_" ----------------
A long straight wire lying in a vertical plane
carries a current I. At distances a and b from it R
there are two other wires, lying in the same
vertical plane, which are inerconnected by a m
resistance R as shown in the figure. A conductor
slides down without friction along the wires. The 3
mass ofthe conductor is m. The acceleration due b
to gravity is g. Assume that the resistances of
the wires, the conductor the sliding contacts, and
the self-inductance of the frame is negligible. If
the conductor is released from zero initial
velocity at t = O. Find the velocity of the

•.
conductor as a function of time .
PRACTICE PROBLEMS 63

228
1 1_' ------------------
A conductor of mass m and length I can c
slide freely on a pair of smooth, vertical
rails. A magnetic field B exists in the region
in direction perpendicular to the plane 'of x X
Ix
rails. The rails are connected at the top end m

by a capacitor of capacitance C and at the l


bottom end by an inductor of inductance L x
as shown in the figure. The acceleration due x x
to gravity is g. The conductor is released x
from the top from rest at t = O. Find the L
displacement x of-the conductor from the
top as a function of time.
12291~~'_' -_._.._.._-._.._.-_._-~-_-~~_-_- . .
A conducting circular loop of radius I is
placed in uniform magnetic field B as
shown in the figure. The magnetic field is
perpendicular to the plane of the circular
loop. The centre a and a fixed point C on
th~ loop are con.lected by a wire of
resistance R. A metal rod aA of length l
and mass m is pivoted at the centre a and
the other end ofthe rod touches the loop at
A At t = 0 the rod i6 given an angular
velocity Iil. in the anticlock wise direction
from the position ac. Find the angular
displacement' e as a function of time.
12301_' ---' ._._._"-"._. -'-"-"-'--" ----_.~-_._---==_.-._. __ .
A conducting ring of mass m, radius a,
inductance L and negligible resistance
is located in the space as shown
in the figure. The magnetic field 13 =
x
- b(xt + yJ +~ ) exists in the space.
The acceleration due to gravity isg. The
z-axis is vertically down ward. The ring
is released from rest at t = 0 from the z
position shown in the figure. Find the
displacement z of the centre of the ring
as a function of time.
64 '. PRACTICE PROBLEMS

A small conductiIlg ring of mass m, radius a and resistance R is


releaged from ",est .alonga smooth inclined plane of inination 6
with the horizontal as shown in the figure. The plane of the ring is
perpendicular to the magnetic field existing in the space. The
magnetic field is given by It ;
-Bo(l + ax) k.
If the ring is released
from rest at t; 0, find the speed of the ring as a function of time.
'.y'

1~~~I',:!
..!,n/»" .,.,.,.,.,.,.,.".,.,.,.,',',',',», '.
Twocoaxial cylinders of radi a and b are made up of high conducting
material. The inner cylinder is solid. This system is immersed in a
magnetic liquid of permeability IJ. and mass density p. The constant
current I passes through the circuit. Find the height h to which the
liquid rises.

T h

.........................................
t...
1~~~I:,n>.
Two planar coils have .mutual inductance M. By
introducing a ,voltage sour:ce .,a -current I = !~o
(1- e lis maintained in.the bigger coil. The inner
~at '

,
.

.
coil has the resistar:ce R and inductanceL. If the 0. .~~"

switch in bigger .coil iis closed at t ; .0, find the


current in the smliller coil as a function of time.
PRACTICE PROBLEMS 65

Consider a circuit shown in the figure. The switch S is closed at


t = O.Find the currents as a function oftime through (a) the inductor
and (b) the battery.

r-.!s
525V

An oscillating circuit consists of a capacitor


of capacitance C and an inductor of
inductance L,. The inductor is inductively M
couppled with a short circuited coil having c~~
inductance L, and negligible resistance. ~LJ
Their mutual inductance is M. Find the
natural angular frequency of the given
oscillating circuit.

Consider an oscillating circuit shown in the figure. Find the possible


natural angular frequencies of the circuit.

c c

L,

2371
1
--------------------------
The time dependence of current is shown
in the figure. The peak value ofthe current
is I•. Find (a) the average value of the
current and (b) the root mean square value
of the current.
66 PRACTICE PROBLEMS

12381 ~ _
The time dependence of voltage is v
shown in the figure. The peak value of
voltage is Vo' Find (a) the average
value of the voltage and (b) the root t
mean square value of the voltage.

123 91 _
The current through the two L

:,":'i::;,'":h:,;"::;k ::: 1: II" 11


1,= [20J2sin(wI+600)] Find the
current supplied by the voltage
L ----
I-----
r
source?
12401 ~ __ ~ __
+
Three alternating voltage sources ."."va
VI = (3 sin 001).volt, v, = 5 sin
(001+ 30°)volt and v, = 6 sin (oot-1200) +
10
volt are connected across a resistance
10 as shown in the figure. Find the
current through the resistor. ~
~
12411 _
. X"
An alternating voltage source V = (170) sin
(3771+ 10°)volt supplies a current! = (14.4)
sin (377t - 20°) Amp to the network
connected to it. Find the average power
supplied by the voltage source. v
12421 ~ ~_~
Consider a circuit shown in the figure. Find the energy dissipated
in the circuit in 20 minutes.
20mH lOOuli' 50')

W
.1 0 Sin (314l) Volt
PRACTICE PROBLEMS 67
12431 _

The resonance frequency of the


circuit shown in the figure is
(4 X 10') rad/s. At resonance the
potential difference across resistor
is GOVand across ind uctor is 40V.
Find the values ofinductance L and
capacitance C.
12441 _

Consider a network shown in the figure.


When an ac 250 V,2250 rad/s is applied a
current 1.25 A flows from the voltage
source. When the angular frequency ofthe
voltage source is changed to 4500 rad/s
R
the current through the voltage source
becomes maximum. Now ifthe ac voltage
source is replaced by a dc voltage source
of 250 V, then a current of 1 Amp. flows
from the voltage source. Find the values
of inductance L capacitance C and
resistance R.
12451 _

In the circuit shown in the figure R ~ SO, XL~ GO and Xc ~ 30. Find
the current through the voltage source.

12461 _

In the circuit shown in the figure, XL ~ GO, Xc ~ 30 and R ~ so.


Find the current through the voltage source.
XL

:-~,

JOOV

L
68 . PRACTICE PROBLEMS

247
1 1_' ------------------
Consider a circuit shown in the figure.
The frequency of the ac voltage source
can be varied without changing the
voltage amplitude. Find the angular
frequencies at which the power in the ~
V =voSinwi
circuit becomes half of the maximum
power.
12481_' _
Consider a circuit shown in the figure.
The frequency ofthe ac voltage source can
be varied without changing the voltage
amplitude. Find the angular frequency ill
at which the voltage amplitude is ~
v=v" Sinwt
maximum
(a) across the capacitor
(b) across the inductor.
12491'--------------------------
Consider the circuit shown in the figure. Find. the resonance
frequency for the circuit.
H L

12501 --------------------------
Consider a network shown in the figure. Find the current supplied
by the ac voltage source.

100 Ion

I
PRACTICE PROBLEMS 69

12511_. ------------------
Consider a network shown in the figure. Find the condition that
the detector D detects no current.
e,

12521 _

Consider a network shown in the figure. Find the current supplied


by the ac voltage source connectd in the circuit.
3'l

246 V

-'----------'~--
12531. _
A particle is dropped along the axis from a height ~
fl2 on a concave mirror offocallength( as shown .L f
in the figure. The acceleration due to gravity is 2
g. Find the maximum speed of image.

~---------------
A cube of side length Imm is placed on the axis of a concave mirror
at a distance of 45 cm from the pole as shown in the figure. One
edge ofthe cube is parallel to the axis. The focal length ofthe mirror
is 30 cm. Find the volume of the image.

f~30cm
70 PRACTICE PROBLEMS

12551 _
A ray of light is incident on a composite slab at an angle of incidence
i as shown in the figure. Find the lateral shift x of the ray when it
comes out from the otherside.

12561 ~ _
An object 0 in side water at a depth h
from the surface is seen by an observer
out side the water. The line of vision
makes an angle i with the normal to the
surface as shown in sthe figure. Find the h
horizontal distance x of the image I from
the vertical line through object O. The
refractive index of water is 1'.
12~1l_. -'--- __
A person at A wants to reach to the 9m 12m
point B after crossing a lake of still A -----
water of width 12 m as shown in the
figure. The speed of walking of the
person is 2.5 mls and the speed of
swimming is 1.875 mls. Find the
minimum time taken by the person
to go from A to B.
12581~ ~_~ _
A ray of light is incident along the unit
vector e on the interface of two mediums
of refractive indices 1', and 1'2 as shown in
the figure. The unit vector along the normal
to the interface is n." The ray is incident
from the side of medium of refractive index
1', and gets transmitted to the medium of
refractive index l',.Find the unit vector ~
along the refracted ray.
PRACTICE PROB~EMS 71

12591 _
A prism of apex angle A is made up
of a material of refractive index fl.
The refractive indices of the
mediums on the left and right sides
are fl, and fl. respectively. A ray of
light is incident from the side of
medium ofrefractive index fl, at an
angle i and comes out from the
other side as shown in the figure.
Find the angle of deviation.
12601, ----' _

A hemisphere of radius a/2 and made up


of a material of variable refractive index
is placed with its base centre 0 at the y
origin as shown in the figure. The
refrative index of the material of the
a
hemisphere varies as fl = --a-x . A ray of
light is incident at the point 0 at an angle x

o with the normal in the xy plane and


comes out through a pointP on its curved
surface. Find the coordinate of the point
PifO ...•O.
126]\, _
Consider an equilateral prism ABC as shown in the figure. Aray of
light is incident on the face AB and gets transmitted into the prism.
Then total internal reflection takes place at the face BC and the
ray comes out of prism through the face AC. The total angle of
deviation is 120°. Find the refractive index fl ofthe material of the
prism. B

c
72, I' PRACTICE PROBLEMS

A person at point P wants to go to the point Q after crossing a


triangular lake of still water, as shown in the figure. The speed of
walking ofthe person is u and the speed of swimming is u/n. Find
the minimum time required for the person to go from the point P to
the pointQ .

.•
Q

A ray of light is incident on the sphere of


radius R and refractive index Il as shown
in the figure. The incident ray is parallel b
to a horizontal diameter and the distance
between the incident ray and the horizontal
diameter is b. Find the angle of deviation Il
suffered by the ray.

A ray of light is incident on the sphrerical


surface of radius of curvature R as shown
in the figure. The refractive index on the
right side of spherical surface is Il. The
medium on the left side of the spherical h
surface is air. The distance of the incident
ray from the axis of the spherical surface
is b. After refraction the ray intersects the
axis at a pointF. Find the distance {of the
point F from the pole O.
F~,.f'''''-~-'''~'~~',~",-~-"
265 t-."I;~' :\c,..,.,..,~_•., \II-'l!' '.'"~~~'''IJf.r:.••
It ~~;'-"",.;._.f .. -....,...•• : ..••..•.••
.'h;~~'''li!I-'
.,rc.••.•.'',1'\!''_''~''''''.
-'": •.;:~;.( .. ,-,,,,(J4"'i..,,.r""',,::::"'''''
"~';.I
~ ,-. ,; ,- ,:J. ,.:J; , .\J•.•.••.
~ j","'.J ....••.-•.•. _ \.:."':,_~I-;_ - -1 ,~',_ ._" _:;.~ ',' -:i-." .;, 1";/.; ";"'7':
~~f<

A sphere of radius R and refractive


inde Il = 3/2 is polished on its half
portion as shown in the figure. A
paraxial beam of light is incident
along the axis (OP) of the
o
hemisphere. Find the position of
the final focus.
PRACTICE PROBLEMS 73

12661 _

A curved surface separates a


medium of refractive index f1 from
air. All the rays incident parallel to
axis after refraction pass through F
a point 'F at a distance {from the
pole 0 as shown in the figure. Show AIR
that the curved surface is elliptic.
Find the semi-major axis a and
eccentricity e of the ellipse.
12671 -------------------,,-----
The distance between objectand eye is fixed 0 f
equ al to I. A convex lens offocallength {is -j--t-+----~
to be placed in between object and eye. The
distance between object and lens is less
than focal length f. Find the maximum
possible angular magnification.
12681 _

Consider an arrangement of two equibi


convex lenses offocal length in air 10 em.
The refractie index of the glass of which
the lenses are made is f1 ~ 312 and the re- 10cm lOclIl
-----
fractive index of wate; filling the space -----
-----
between the two lenses is f1w ~ 4/3. A small P, -----
--- ,
object 0 is placed on the axis at a distance 0 AIR -----
----- AIR
-----
of 10em from the first lens in air as shown v:::::: v
in the figure. The distance of separation
between the two lenses is 10em. Find the
position and magnification of the final
image.
12691 _

A thin convex lens offocallength 1m is


cut into three parts A, Band C along
the diameter. The thickness of the mid-
dle layer C is 1 em. The middle layer
is now removed and the two parts'A and
B are put together to form a composite
lens. Then the part C is also placed
infront ofthis composite lens symmetri-
cally as shown in the figure. A paraxial
beam of light is incident along the axis
of the part C. Find the distance betweer,
, .1he two images formed.
74 . PRACTICE PROBLEMS

12701'
An equibiconvex lens offocallength 10 em in AIR and made up of
material of refractive index 3/2 is polished on one side. Another
identical lens (not polished) is placed infront of the polished lens at
a distance of 10 em as shown in the figure. The space between the
two lenses is filled with a liquid of refractive index 4/3. An object 0
is placed infront of the unpolished lens at a distance of
10 em. Find the final position ofthe image.
IOcm lOclIl

. There are two coherent point sources 81 and J'


82 pIeced on the y-axis as shown in the 50 d
figure. The wavelength ofthe light emitted
from the sources is A. The distance between 5,
two sources is d = 2),. Find the locations of
all the minima on the positive x-axis.

12721'
1\vo coherent point sources 81 and 82 are Screen
placed on a line perpenclicular to the screen p-
as shown in the figure. The wavelength of D x
the light emitted by the sources is )•. The
distance between two sources is d = 2A. The s, d So
distance of 81 from the screen is D (» A).
Find the mimmum (non zero) distance x of
a point P on the screen at which maxima is
obtained.

1\vo choerent point sources 8, and 8


emitting light of wavelength ), are placed
symmetrically about the centre of a circle
oflarge radius, as shown in ti,e figure. The
distance between the two sources is d = 2A.
Find the positions of the maxima on the
circle in terms ofthe angle O.
PRACTICEPROBLEMS 7S

1
2741
--------------------------
Consider an arrangement of slits shown

in the figure. It is given that d = ~ I:


and D» d. Here I, is the wavelength of
the light used. Find the minimum D D
distance x of the point P on the screen
where maximum is obtained.
12751 _

Consider an arrangement of slits and


screen shown in the figure. The wavelength
ofthe light used is I,. The distance between 11
8, and 8, is d «< 0). The distance between -=-lI"'bI["3
. ~O.
8, and 8. lS b = 4d' Fmd the ratio of the
.
-
Jr: ~2
84
Screen
.,
maximum to minimum intensity observed. O. D
on the screen.
12761 _

Consider an arrangement of slits and


screen 2S shown in the figure. The Screen
wavelength of the light used is I,. The
distance between slits 8, and 8, is d = 21,. o
The distance of screen from the slit plane
is 0 (»d). Find the distance between two
minima on the either side of the mid-point
o of the screen.
12771 . _
"
In young's double slit experiment two
thin films are placed in front of two l'
slits and the parallel beam of coherent I'
light is incident at angle () with the )'
normal to the slit plane, as shown in
the figure. It is given fl, = 1.7, fl2 = 1.5,
t, = 3mm, t2 = 2mIll, () = 30., t = 2 mm Semen

and 0 = 1m. find the y-coordinate of


the central maxima P.
76 PRACTICE PROBLEMS
.,• 'II!" •••••••• ,,,.-......-------1,
Consider an arrangement ofthree slits and
screen as shown in the figure. The
wavelength ofthe light used is A. It is given 1 Screen

D » d. It is observed that there are two ---;rIo


types of maxima formed' on the screen ~
called principal and secondary maxima, ;rl,
Show that the intensity of principal ~TI,
maxima is 910 and the intensity of t--D-
secondary maxima is 10• Alsofind the width
of principal maxima ~1 and the width of
secondary maxima ~2.
1279W -
Consider an arrangement of a
Biprism
biprism and screen as shown in the
figure. The refracting angle of-tlie Screen
Source
biprism isA'" 20"lmd the refractive
iildex ofits material is fl = 1.5. The
wavelength ofthe monochromatric
light emitted from the source is 2.In lOOcm
6000 A. Find the fringe width ofthe
interference pattern obtained on
the screen;
.•. - .
12801 ,.,' ,

A convex lens offocallength 50 cm is cut along the diameter into


two identical halves A and B and in the process a layer C of the
lens ofthickness 1 mm is lost. Then the two halves A and B are put
together to form a composite lens. Now infront of this composite
iens a source of light emitting wavelength A = 6000 A is placed at
a distance of25 cm as shown in the figure. Behind the lens there is
a screen at a distance 50 cm from it. Find the fringe width of the
interference pattern obtained on the screen.

A 1
Imm
SOllrt:p.
Screen

B
T 25 em 50 cm
PRACTICE PROBLEMS 77

~r-~'~"r-:~--~---
-:w~-~c-~:,'-c~J
A parallel beam oflight ofwavelength 6000 A is incident on a thin
film ofinfractive index 1.33.The angle of incidence is 60 Find the 0•

minimum thickness of the film for maximum reflection.


r;;;;:;tr .....-- •...-
~'"T .--~-_ .. - •• .1'.'" --_._'
~~. .•••.•• _.~
."'----~-. ".- --

)~I'c. : ..... :. :' .' '"

A parallel beam of light of wavelength 5500 A is incident normally


on the surface of a wedge made up of glass of refractive index
i.1 = 1,5. The angle between wedge faces is 3'. Find the distance
between the two neighbouring maxima on the surface of the wedge.

-
1283Jl "--'~ .'
A parallel beam of electromagnetic radiation of
wavelength 200 nm is incident on a perfectly
.•.---~...•....••.•.•.. '1

reflecting surface. The power incident on 1 mm2


area is (5 x 10-")watts. Find the pressure exerted

¥-
by the radiation.
1284li~.:- ...,.. ....' -. _ ••.. _v. _'.. ._~_

A plane electromagnetic wave of intensity 5 x 10-'


W/m2 is incident on a plane surface with reflection
coefficient 50%. The angle of incidence is e = 450. e
Find the normal pressure exerted by the
electromagnetic wave on that surface.
12'85Ir~;~~'.~
~'~..:- ."--~~-~~'.~".~
.~'".~.:~,
~.~~.~~-~~-
.~..~~.'~"--"~'--""--"'-....
'..' ~~
Ametallic surface is illuminated alternatively with electromagnetic
radiations of wavelengths 3000 A and 6000 A. It is observed that
the maximum speeds of the photo electrons under these
illuminations are in the ratio 3 : 1. Calculate the work function of
the metal.
1286V-~..'-~'::: -c~~.:":'. -". '.~'
A source of.power O.IW and emitting
light ofwavelength 5000 A is placed at
a distance 20 cm from a sphere ofradius
30 cm as shown in the figure. The work
function of the sphere is 1.5 eV. The
*'
Sou""

20 ern
efficiency of photoelectric emission of
the sphere is 10-".Find the time from
the beginning of the experiment after
which the photoelectric current stops.
78 . . PRACTICE PROBLEMS

1
2871_" ------------------
P Q
Two metallic plates P and Q are separated by a
distance of 0.1 M. These are connected through an x x
x
ideal ammeter as shown in the figure. A magnetic x x
field B exists parallel to tqe plates. A .light of x
wavelengths between 4000 A and 6000 A fall on X x
the plate Q whose work function is 2.39 eV. Find
;A;~
the minimum value of magnetic field B for which
the current registered by the ammeter is zero.
12881 _ ~
I

Consider an an'angelllent shown in


the figure. The width of the slit is
1.6 cm. The convex lens placed in f= 40 em
contact with the slit transmits 80%
of the light incident on it. The Source Sphere
radius of the sphere is 8 mm and 8mm
the efficiency of photoelectric 1.Gem
emission ofthe sphere is 10-'. The
power ofthe source is 3.2W and the SO em 40cm
energy of the photons of the
radiation emitted by the SQurce is
5 eV. Find the photoelectric current
from the sphere.

12891

A moving hydrogen atom makes a head on inelastic collision with


a stationary hydrogen atom. Before collision both atoms are in
ground state and after.collision both atoms move together. What is
the minimum velocity of the moving hydrogen atom if one of the
atom is to be given the minimum excitation energy after colli~ion.
12911 _ ... / '.~ ~, .la:~...,..~
•.•

A hydrogen like ion in first excited state absorbs 10.2 eV photon


and gets excited to higher energy level. Now the excited ion can
emit six different photons, some of higher and same of lower
energies than the absorbed.photon. Find the quantum number II of
the excited state and also find the atomic number Z of the ion.
PRACTICE PROBLEMS . 79

Two hydrogen like atoms A and B are of different masses and each
atom contains equal number of protons and neutons. The energy
difference between first Balmer lines emitted by A and B is 5.667
eV. When the atoms A and B moving with same velocity strike a
heavy target they rebound back with the same velocity. In this
process the atom B imparts twice momentum to the target that A
imparts. Identify atoms A and B. '

~ m:i:':'ii::i::iimiiim:iiiiii'::,:i':':i,ii.i::iiiii.'.::: :",:,:.::,::::;::,:::,t';'! ..•,":::'::i;':::::: ii ;:::';iii':;


,~' ..
Suppose ,the potential energy between electron and proton at a
'.-- .
2
d~tan'C;; ; is given by - ,e , 3' Use Bohr's theory to. obtain
121l£or '
energy levels 9(such a hypothetical atom.

IgiHI:m:i;i:iiimiiii::i:i:i::"i,:':i"'i'i'iii'i":imii"ii,,':i:::'ii,im"i,':',i":'ii::i!i'i:iii'i:i,i::":!::':'.;,;::m':;i'::::.',~:
. l' ~ ,/'
'Two charged particles A:.mdB move on~",,' ~:0
circular paths abo~t theit cenU:e~o'(rnass .• ,-' . ~ ,-'"
" , under their mutual electrostatic attract;'o'ii.-:",'~m ' ' m,tJ
TheparticleAhasmassnl~electro.nic~ass) ~-2l""""~Br--..:- .I'~fr ,A ..•
'and charge +e (electromc charge). The " - - _.,~ "'-'-- •
•' particle Bhas mass 2m and charge -2e,' _
Using Bohr's theory, obtain the energy level. ' ,. .

~::i:iU:iii:i:ii:ii:iii:Umi:i:,i:iH:ii:iiiiiiU:iiiii:iiiiiii:i:i:iiiii:iimiiiUiiHiiiiiUiiii:mii:iUiiii:Ui:ii:::iU:iii:Hi:i:iii::
The wavelengths afKa and ~ X-rays of a rn~teri~l~re2L3p~ ahd
18.5 pm respectively. Find the wavelength of La X-rays of the
. I
rna te na. r
. I
•..• ~

r~~~F::i:i:':i:::i:i:::i:i:i:i'i:i:i:i:i:i:i:i:i:::i:iiiii:i:i::ii:iii:iU::ii!U:i'i'i:,,::'!'!:!:i::!:i:::i:i:i:!iii!i!i:i:i!,:i:::,:i:i:i:::i'i:i:i:i:i!i
• In an X-rays tube'the anti cathode '!Iaterial has atoms.Of atomic
, . , number Z = 28. Find the voltage applied to the X-ray tube if the
,'wavelength difference between the k~line and'short-wave cut off
of the the continuous X-ray spectrum is equal to 84 pm.

~i:,i'i:i'i,tiiitii'i:iitiiii:!iii:iiitii,tiiiiii:iiiiiiiiiii'iiiU:UiititiiitUtU:i,i!:i:iititiiitii:tii't:!:tititit:tititUt'tit:tititititiiiiititU
A ('Li) target is bombarded with a proton beam currentof 10-<Amp.
, . for 1hr. to produce ('Be) of ac'tivity '1.8 x 10' disc/sec. Assuming
that one ('Be) radioactive nuclEiusis produced by bombarding 1000
.' . protons. Determine the half life of ('Be). ';.' ';
.~ • .".~.b t:'
80' . PRACTICE PROBLEMS

A radionuclide A with decay constant ;t


transforms into a radionuclide B with same
decay constant ;t. Assume that at t = 0 the A~B~
number ofradionuclidie "A"is No and number
of radionuclide B is zero. Find the number of
radionuclide B afu!r time t.

A nuclear explosion is designed with a nuclear fuel consisting of


U-235 to run a reactor to deliver 1 MWof heat energy. The amount
of energy released per fission is 200 MeV.Find the amount ofU-
235 needed to run this reactor for one year.

Consider a following nuclear reaction.


jO+ ~O....•~T+lp+aE

It is given m' no).,;; 2.(ji~58~U,m nT) = 3.01605, amu, m Up)


=..1.00728 amu. Calculate the mass of dueterium nO) required
per day for a power output of 10"\V. Assume that the efficiency of
process to be 50%.

~,." .••.
;..;},:,<:?\:"
_:.:-:.:.:-:.:.:::::::::::::::::::::::::,::;:::::;::::.:.".
.•.
'.;
.•.
'.'.'.;
.••
'.;I.

In the relation P = (:)e {:;) , P is pressure, x is distance, k is

Boltzman constant and 9 is.temperature. Find the dimensional for-


mulaoffj.

A quantity X is given by X = E0 C where £0 is the permitivity of free


B2
space, C is capacitdnce and B is magnetic field of induction. Find
the dimensional formula ofX.
PRACTICE PROBLEMS 81

130$1.•.•.•.
;':':':':.::.: ,.,.,.,.,.,.,.,.
............................. """""""""""""
,.'.'. .•.•.•.•.•.•.•.•.•
.'.'.'
.•.
'.'.'
.•.•.•.
'.'.•.'•.••.
••••••••
';:" ".;.;.;.;.;.;.;.;.;.;.;.;.;.;.;.;.;:

If the energy E, angular momentum J and universal gravitational


constant G were chosen as base quantities, then find the dimensions
of mass in terms of E, J and G.

. Assuming that the classical radius r of electron depends upon the


electronic mass m, electronic charge e, speed of light in vacuum C
and the permitivity of free space Eo, find the expression for radius r
using dimensional analysis.

~
r.;;rn.', ..... "'.... , ',','.'.', >;;: »..;..:"""",.,.'.
.-:.:-:-:,:;:-:."
:::::~::; .'.'.'.» ;';";:;:,:,.,.,.,.,.,.,.,.,.,."".'
~<>:~< - -.- •••» ••••
. ".»);;
.Assuming that the minimum possible measurable time t depends
upon universal gravitational con'stant G, Planck's constant h and
speed of light in vacuum C, find the expression for time t using
dimensional analysis.

27
flom e ..
Check the dimensional accuracy of the relation B = 256,,4s8h3

where B is magnetic field of induction, flo is permeability of free


SPlice,-EO' is permitivity of free splice, h is Planck's constant, m is
electronic mass and e is electronic charge.

...' "'I:..:."',.'.
" - ;.....
,.,.,.,.,.,.,.,....................
. .,-,-,-,-,-,-,-,-,-.,,"
.
::::.,:.:.:
..'.:.';,:
:.:.:. . ' -:-:.":-:.;.:
.
1mI ~:~:>«»«~:~;:. ;.;.:.,:;:;:;:::::::,:::::;::,:::,:
:::::::::;': -.-;':'- .,...•.
,'-' '.,.'.',,.:
-'..':':.'
- ':.','
..','",'.',"
..-..'..',.>:
A cube has side of length 1.2 x 10-2 m and mass 3.10 x 10-' kg.
Calculate ita density with due regard for significant figures.

romi1
~
Calculate the temperature of a gas in true significant figures from

the V~der Walls equati~ns '(P + ;2) (V - b) = nRT ~ following


data. ' .
P = 90.00 atm, V = 0.250 litt-e..
.'. .... .' (iltffil(litre)
n = 2.000 moles, R = 0.082 (mole)k
82 . PRACTICE PROBLEMS

(atm)(litre)2
a = 1.85 2 b = 0.042 (litre)/(mole)
(mole)

The value of one main scale division is 1 mm and 9 main scale


div'isions are equal to 10 vernier scale divisions.' There is no zero
error in the instrument. When a'rod is placed between the jaws of
-the callipersethe 39th division of the main scale'c'oinci(hi~with the
7th division of the vernier scale. Find the lengtn-of'the'j.da .
•.~~.t" .
:.:.: : :.:.::: :
.3...•1..9 '.:'.'..•.
~
'.•.
'.•":.:.;.:.:.:.:
'"", ::::: .',.: ,.,:,:,:::::.'.'.'
:.:.:.:.:.:.:.:.:.: .'.'.•.
•.'.'.'.•... ,.",.H;.'.'.................................•..•......
:.'.,'.,:.:.'.',
'.•. :.:. ".,' , ,'.:: =::: ",:::: ,.,.,.,.,..
:::::::::: :': :,::~::

. The value of one main scale ,division. i~ 1.mm and 9 ma,in scale
divisions are equal to 10.~ernier scal~' divisions ..When n~th;ng is
put between thejav:,s ~f the cailiperse. the:zero orth~ vernier
scale lies right side of the zero of the main scale .and the 4th
division of the vernier scale coincide.s \\ith a main scale divlsion.
While measuring the inner diameter-of a hOllow cylindeT'the'zero
!If the vernier scale lies between 2.4 cm and 2.5 cm of the main
scale and 8th division of the vernier scale coincides With a main
scale' division. Find the inner diame~~~fthe q.linder.

I~~(I
n:!.:m::!:m!::m::::r:::r:::,m!:.:::!::.:m.::!!::::r:'m:.:.!:,::::.:m:m:::::.:.::::::fm::r:!::::: ••:mu::fm:::~m::m:::
The value of 1 main scale division is 1 mm and there a;~'10diviorlons
on the vernier scale. The value of one vernier scale division is
O.~ mm. When the two ja~s of !he vernier calliperse touch .each
other the zero'ofthe vernier scale'lies to the left of the zero of the
'main 'scale and 3rd division of the vernier scale coincides .Witha
main scale division.' While me~suring' the depth'~f hollow a
cylindrical can the 6th division of the vernier scale coincides,With
the 38th division of the main scale division. Find the deptli'ofthe
cylindriea1 can.
.' . .

In a'screw gauge, the linear distance travelled by the head scale is


5 mm in 10 complete rotations. There are 100 divisions' on the
circular scale of the screw gauge. While measUl,ng the diam~ter of
the Wire, the main scale reads 2.5 mm and the circular scale reads
33 divisions. Find the diameter' of the Wire' (There is 'nb zero error
in the screw gauge.)
PRACTICE PROBLEMS. 83

1~~~Hm:::~~:mm!ml~liWi~m~~::i:m:;::l:m~:::~~:~:wm~::m:m~m~~~m~mmillt~l:ll:~:::~:~::::muw:ml:~~~mt~~:m
. The pitch of a screw gauge is 1 mm and the number of divisions on
" J' the circular scale is 100. When nothing is'put in between its' jaws,
,~ :the zeroof the circuhir scale lies 5 divisions below the 'reference
r- " ~: line; i.e.;: the zero error
is +5 diVisions 'on the circular sca.1e. When a
~'::TWire is h"c1il'in between its jawsth"i;'reading ofiOli.iri scale is
8 divisions and on the circular scale the reading'is 78 diVisions.
Find the diameter of the wire.

I~~~I ::':i:::::i::::ml':~':::i:i::~i:l:::lmlm:::mm::~i~'::i:::l:i::"i:::ui:l:~iimm:::mi~l:::::i::l:i:m:::::m@~::i:i:~:
L.' ""."..1. - .•..••. ' •••.•.••••~ - _.v ~ ~ -" ...•
_~••

The pitch of a 'screw gauge is 1 mm and its cap is divided into 100
divisions. When nothing is placed betw~en its studs, the zero of
:" l.;the circular s.cale is 6. djyisions>bove the,'refe"rence line. (e., the
zero error is ~ diVisions on ilic"ciic'6.lar scaie. WJien a wire is placed
::;:~;.~betWeenits stUds the main scale readillg is 3 divisions 'and 37th
,J1~r division""of CIrcular, scale 'coincides ,with reference line.' Find the
~2'l<lUii'. ~
diameter of the wire. -"'-fB.:-;;..,.!O--.-~~~~~~
~ _•.•..tr 1\ ~ hi.,:- 1J ." ",-

J I~~~I
:i::i:::::i~::::::;::m::::ll:::::i:i:m:i~::~:::m:::mm:::mi::mm:i:m:::j:l:i::::::mi:l:l:::::m::::m::m:1Wm:lmmm::m:::m
A wire has (0.3 :!:0.03) gm; radius'(0:5"",0:005) mm and length
(6 :!: 0.06),cm. Find the maximum, percentage ,error in the
10 ,"n ~easur~tI1_en't.of i!"
d,~sity.: "".LdJ 1" :!:t'.;,,..1(LX:' ~-:> k:"{,

r~tm::[:l:l:[:::l::~::::::[:l:l::ml:l:[::~li:':::~lm:::~::l:i:::l:l:[:l:l~;l~~:~m~::[jl:[:m;i:::i:l;l:::::l:::l:l~::l:l:m:lml:lml:[:l::'
The length and breadth of a rectangle are measu-"'ed (33.2 :!:0.1)
cm, and (16.8:!: 0.1) cm respectively. Find the percentage error in
the measurement of its perimeter.

In an experiment the refractive index of the material of the prism


are found to be 1.54; 1.53 •.1.44, 1.54, 1.56 and 1.45 in successive
measurements. Calculate the percentage error in finding the
refractive index.

1~~~Hi::l:::l:l:::l:l:n::::::::l:l:l:l:l::ml:[:l:l:l:::::::l:l:l:l:l:::l:l:l:l:l::::::~::l:l:[:l:l:l:l:l:[::::::'lml:lml:::::::::::l~l:[:lml::mm:l
The diameter of a cylindrical wire is measured 0.247 cm using a
screw gauge of least count 0.001 em. The length of this wire is
measured 5.63 cm using a vernier calliperse ofleast count 0.01 cm.
Find the maximum permissible error in the measurement of its
volume in cubic centimeter.
84 PRACTICE PROBLEMS

1mJ
....•.
3
.-,',','.,',','
"••.... ...
:::;::::':'::::::,::'::;:,:::::;:::;:::::;:;:::;:;'::;:::,:::::::,:;:::::::::::::::::::::::::::;:;'::::;:::::::::::;:::::;:;:::::::;:::::::;:;:=,=:::=,j:::
',',',',','.','
jlilijlll! .::::::::::;,::;:::::::::::;:::::::::::::::::;:::::;: ':::;::::: j:!:j: j: j:!:~:::j: j: j:::j::::: j:::::j::::: j: j: j:::j: j: j:::j: j: j::: j ::' j::::::'

In an experiment to fmd the focal length. of a concave mirror the


readings of object distance and image distance are (40.0 :l:0.1) cm
and (120.0:l: 0.1) cm. The image formed is real and inverted. Find
the maximum percentageerrorin the measurement offoaallength
of the mirror.

YU...
la.•..•~....o....I.;.'.'.•..':.:.:.'.:.:.'.:.:.:.:.,.,.'.'.'.:.,.,.:.:,.'.'.':.'. »:",,,,,,,,,,,,,,,,,,,:':U'/':::,::;:
.................................................. ;.;.:-:-:-:':-:-:':-:-:-:-:':':'. :.:-:.:.:.:.;-,.;.;.,.; .....

The focal length of a thick biconvex lens is given by

1
7 = (I-' - 1)
'( i ..1
R + R - MRi R .
(M -1)t) where R1 and R2 are radii of
1 2 2
curvature, t is the thickness and I-' is the refractive index of the
_glass.•In an experiment to find.the focal length.of-a thick lens
-fOlloWing'Observaticms.'!'i'er~
taken.
R1 = (20.0 :l:0.1) cm,
~ = (60.0 :l:0.1) cm,
t = (15.0':l:{),l) cm;and
:1'={1.5 ..:l: O.Ol)cm. __
-Eindtlie focaJ.lengthoftl}e l'ms'.and express the result in terms of
maximum absolute emt:

••
".'~."'.,.:- ...' -'-',",'-" ,-'

.,,'

':::~--"-'

..-..-::.:~-;O-'"'
I"" -,?~-,
(:'

'.".;

, •.
' .. :
, '
_1--------
PRACTICEPROalEMS

1. 4 seconds
• '85

rmin =24 m

d2 + 12
3.
ud + ~d212 + 12(U2 - U2)

0:.13 (:.13 + 1)1


4. ----
2u

2
d(U+W)[W(U+W)-U~(U+W)2 _U ]
5. t
uw2 ~(u + w)2 _ u2
.. " ..",
"

.... ,
•.
'
"

6. d ~

7. a = tan-'(2)
'f
,.
u = 'u.J5
. ~"
8. 9= tan-1 (2tanll-tana)

u 3u
9. <u>t!mc = 3"' <U>space:;:: 5
10. u = 3 mis, S = 6 m
."
86 ,
PRACTICE PROBLEMS
.
1
13. V =H8(6- f)(12 + d]4
V()
14.

1+(:!)
15. x=-;(wt-sinwt)
w
211 flll3
16. T= 3
(2flll- 1..)2'

17. displacement = 85.1 m distance = 85.3 m

18.. ~elocity>= 5,[2 mis, <speed> = 8.12 mls

19.


..
20.

O'!

2asin2(a + 13),
25. T =
g[sina - sin 13cos(a + I3)J
PRACTICE PROBLEMS 87

26.

2u2 sin acos a(2v + ucosa)


27.
g(v + u eosa)

6v
28. T=
gJl +8sin2 a
2geosa
29. w=
u(1 + 3eos2 a)
g

[(usina _ gt)2 + (ueosa)2 ]2


30. P =
(ugeosa)

31.
(U211 - U112)
max
( + u~ ) ur
w ~ (1 _ U2t)2 + (11 _ Ult)2 ' w = U211 - U112
2

WIRr + W2R~ - R1R2(WI + W2) eosO


32. W = 2 2
R1 + R2 - 2R1R2 eosO
2
33. U = 2m/s, Ul = 50 m/s2, u2 = 85.4 m/s

2uR1R2Rg
34. vo=
'2 (rg + Rg)(RI sino. - rd

35. (-28i + 4j) m/s2


R1(R1Wr -~w~ +2R2WIW2)
36.
Rl+~

37. ii=(5i + 24j) em/s, ii=-(15i +20j) em/s2

38. u=]fuu 4+
(ru)2
Ru
88 • PRACTICE PROBLEMS

40.

,41. (a) w = 0.732 rad/s (b) VB = 3.6 mls


(c) a = 1.05 rad/s2 (d) aB = 6.65 mls2
42. (a) w = 0.816 radls , a = 0.l93 rad/s2
(b) VB = 1.115 mls , aB = 5.83 mls2

43. If mls

44. v = 4,38 mis, a = 56 mls2


45. vB = 22.7 mls

46. v = [ (v? + v~ + 2"lV2 cos e) ]cosec e

g(m3 - m1 cose)cose
47. a=
(m1 cos2 e + m2 + m3 + m4 cot2 a)

48. Block C moves up with acceleration a= (:~)g


mg(1 + cos e) sina
49. a =
M+m[(1+cose-cosa)2 +sin2aJ

50. a = (~~)g upward

51.

.~ ..• (II + ~ }m1 sinacosa",.mz sinfl cos fl)-(sina +sinfl)(c.osa -caSfl1]


a~
[ (ll + ~)m3+m1sin2a+nlzsin2fl)+(cosa-ed'ofl)2
g
PRACTICE PROBLEMS 89

52.
I

1'2(ml +~)
53. a = (cosa -1'2 sina)

ml (ml + ~)(l'l -1'2)g


"F" (~ (cosa - 1'2sina) - ml (1'2 + I'dsina) .

54. (aj at = a.= aa= 1mls'


(b) at = 2m1s' a2 = aa = 2.6 mls'
(c) at = 2m1s', a, = 3.67 mls', aa = 4.71 mls'
g[mlk - 2~(cose + I'sine)]tane
55. a=
mktane + 2~(cose + I'sine)
where k = (sin 8- 21' cos e _I" sin e)

56.

57.

58.

[2 ( 'J2]
R Vo vO'
59. s= 21'In Rg + l+Rg

1 ,

60.
w>(iRY
61. F=mg
90 PRACTICE PROBLEMS

62. t= (Jf}n(J2+1)
63. :7 (../5 + 4J2)R

64.
v= ~~2+~)

3 2
65. V = -vo + 2gb
4
66. W= 1.2 Joule

67. V = Jgl(l-11)

136
68- 3 Joules

l{ml COSa + ~ cosp)


69.
(ml +~+ma)
70. (9.49 em, 6.11 em)
71. 133.57 J

4mlm2C
72- Xrnax =
(ml +m2)K

73. .0 = eos-1 ('/3- 1)

2m2glsinOcosO .
74. (a) v •••.• = (m+M)(M+msin20)

", ~:". , ,;
PRACTICEPROBLEMS 91

M(M+2m)vff
75.
8(M+m)

2vo
76. 19

2vsinct v2 sin2ct
77. (a) g(1- e)' (b) g(1- e)

2 2
m m2vO
79.
2J.lg(m + ml + m2) (m + ml)2

81 . ml
m2
=2

e= mlsin2~+~sin2ct
83.
ml cos2 ~ + m2 cos2 ct

84. 8max = sin-1 --3 - e' (1+e)e = tan-


'.'
1 R-e
--
2

~ (1 + e)usinct
85. . 2
. ;.,
ml +m2s1n a. .r

./3(1+ e)Mu
86.
M+6m

(1 + e)musin2 e
87•
. M+msin2e
92 PRACTICE PROBLEMS

mvO(1 + e)sin8 cos8


88. msin28 + 2Mcos2 8

89. [3:og (1- e-1'1!2"'O) - gt]


1
90. -gt
2( ---
2ro + fll)2
8 ro+fll

91.

95. ¥(a2 +b2 +c2 +ab+bc+ca)

95. J2gRsec8

96.
mv8
kl

3 3
97. l' =ta I -tan -1( sin d. - sin. ~ )]
{ 2 . sin2 a COSa+ sin2 ~cos~

4tanatan~
98. l' - tana + tan~

4
99. 37 g

F
100. 7m
PRACTICE PROBLEMS 93

2mgsin9cos9
101.
3M+ m(l + 2sin2 9)

/
102. Monkey A,- (M+4m)
----
2u M+3m
3,f3g
103. 10/

mgsin9
104. 1 + 3sin2 9

2
105. vA;:;
3g/(1- sin9)sin
2
9 ,vB::;;
)3 g.L-Sln
/(' . 9)
1 +3cos 9

13
106. 4 tan9

36,[2uo
107. 65/

2muo
108. (M+ 3m)R

109. h = 42 m, J = 30 kg mls
. 53uo. uff
110. tIme = 28~' distance = 2~

111. u(M+m)
R M+2m

.,,=
112. .• ta n_1[tan9
--+ (1 +~!'
e
1)]
e
94 i PRACTICE PROBLEMS

(rolRj + ro2R~) , (rolRj +Cil2Rl)


113. ro'l = ~---~ ro
R1(R; + Rl) , 2 R2(R; + Rl)
114. a = ro> e = sina

2
1 V= -2GM u=- 2GM
16. R' 3R

m2g2Z
118.
10n:R2y

3 (H ~R)
119. 2R pg ---
3 16

3
120. (cos2a)"2

121. A1A2 ~2H


(AI + A2)a g

J'2g (H - h)
122.
R

123.
PRACTICE PROBLEMS 95

2 2
125. 41njlw ( 2a b 2)
b -a

126. (l~)vO
127. 2"J ~(4
2
+tan a)

128- T=2[~+ff(,,-tan-I ~)]

129. 2"~
3g

130.
2nJHi
~1+=]

lI~ (li +~) ~ (li +~)


J31. T=x +It
(t? +4)c liC

3m(2M+3msin2 e)
132;, 2"
2K(2M+3m)

(b
l!33. /j2,,2 = a2 2 _ y2)(b2 _ 4y2)

mo: y =a sin21t (~ - ~)
where, a = 8 mm, T = O.58-second,A= 284.2 em.
96. I PRACTICE PROBLEI,1S

1~. y = 4 sin (lOd +~% + :)

where %~3I'dyarejn an and t is in second.


137. (a) Along astraigbt line in ry plane through origin at 30° with
x-axis.
(b) 1m
(e) 4n
138. (a) 10-10 WIm'
(b) 2x 10-' .N/m'
l3e. (a) (6.J3):e
'b3
(b) Y = 2
b2 + (X - (e-vlt)
140. 1.44%.
141. 98.7W

2
142. (a) 'i.A
(b)88.9%

zrh2
143. I

1« y=[5Sinn(4l-0.7llcm

145.

146. 20cm

A = (v-'w+:u):{v + w -v.)
110 (v +w -'u)

148. I042JHz.30;=
It% .It:)'
149. z = Asin 1 -JP-D1 .sin ro't
PRACTICE PROBLEMS 97

150. 391Hz.

(I'Y2 + l:!Yd
151. t = (I, .Jp; + 1 .,JP;)
2
e(l,.C1., + 12C1.2)Y' Y2

153. 1.0155 m

154.

155.
_m.e-~(
....,
~,_~Mt2)
156. 153.4 gm

T
157. T=T1 ( T
2)'/l
,
158. T; = T, _ m"S2 (T, - T2) (l_e-at)
m,S, +m"S2

T~ = T2 + mIS, (T, - T2)(1 ~ e-at)


m,S, + m"S2

where C1. =~ (m,IS, + ~S2)

il59. T= T,K, +T2K2


K, +K2

16L
98: PRACTICE PROSLEr.IS

162. Ah (PI - P2)

(In ~J
163. P=Po e-rtfVo

164. 607 mls


5
165. "4 Yo
166. (41.13) Yo
167. 41%
168. 8.33%
1
169. (a) pya = const
(b) 3 R

170. V" T' e'T = const

171. W=nRT
o
In [1--
4
(PoA)
-
2kh
+ ...!-+3PoA+(PoA
16 4kh
r
2kh

172. 18 PoYo

173.V=[3g1(~+Jl+(:n]i
174. v=qE (T
mVli
175. t=.j2.hEomd [/6+in(3-J2)1:
2

176. (a). (
qQ
4ltEOmVO
2) + b" +
( )"
qQ
. 4llS0mv~

(b)9=2t8nc' ( qQ")
4nEomvob
PRACTICEPROBLEMS 99

177.
V = psin8 sin~
2
4"sor

E= P 3 ~ 1 + 3sin2 8 sin2 ~
4"00r'
A
qRj
178. E=
,,2S0
( r
Z2 + R2

179. " [,••nH" ".'J 1


F= 2"00 In a(I+~ 12 +4(a+I)2)

2cr2R3
180.
300
Q
181.
2"sol
cr
182. E=
4,/2°0
183. 4"s.OtR'
184. 4"a
185. + 20 f!C
186. (a) 190 fiC
(b) 190 fiC
(c) 85 fiCthrough S" and 105 fiCthrough S,.

4"°0
187.
e-+.!-~)
a b d
V
188.
d

189. 400aK tan -1( ~K - 1)


~K-1

190. (S:V)ln%
191. 69 fIT
192. 22 fIT.
'00 ' , PRACTICE PROBLEMS

193. (a) 105 JlC, (b) 2.625 mJ.


194. (a) 150 Jlc, - 10Jlc and - 135 Jlc on 1 Jlf, 2 J.Lfand 3 J.Lfrespectively.
(b) 3.3 mJ.

Z2 C ZV2 Z
195. + 0
4(K _1)2 pgd2 2(K -1)

196.

3Z
197.
1t<i2(", + 2"2)

198. ( pZ )lnad
ad-bc' be

199. (a) :. [(3n-1)+J9n2+2n+1]

2
E [J9n +2n+1-n-1]
(b) -
R ..J 9n2 +2n+1 +3n-1

200. ~(J7
2ar
+1)

201. V= n(n+1)E
2
n(n+1)r
R= 2m
202. E = 43.24 V; r' = 2.1 Q.

203. (1 +~e-t!T ) Amp

where ~ ,; 25 Jl sec
204. q, = 28 JlC,q. = 14 Jlc, q, = 42 JlC

205. Charge = (::;) (1_ e -1/, )

Current = C:R)(l +~ e-I/')

where, ~ = CRr
. R+r
PRACTICEPROBLEMS 101

206.

and

where,

207. ::~ In (/2+1)


floI
208. 4a

209.

floIa 1.\ ~
210. -~--3/-2 ("a! + x JJ
2 2
2,,(x +a)

211. B = floNI(b-a) In.!'.


12 a

212. B = floI tan-'(..!!...)


"b 2a
I!obra +1
213.
CL+2

214. floNI (a;b -M)


215. floI,I2 (J 1)
b2b_ a2

216. ~2" 1,a12 [ 2 + 2


a + -2-
(II 1 )2 - 2 (12_1,)2] .
a + -2-

217.
,,2aBI
4g
102 I . PRACTICE PROBLEMS

218.

aqB
219. ll(mg + qE)

220. It = (vo sina.sinrot)f + (vo sina. cosrot)) + (vo cosa.)~


7= R (1- cos rot) i + (R sin rot) J + (vot cos a.) ~

qB mv
where ro = -.m and R = -B
q .

221. qB2d2
2m
2 2
222. qB (b- a )2
81ll b
a• (a. + f3h
223. F=-----
,,2
R
12be
224.
7
5qB
225. ---
Gm
226. g sinO
2+ lloQ2)
( 4"ml
mgR(I-e-a,)
where a. =
1 (llol
-mR -In-
b)2
227.
(
110
1 In~) 2
2" a
2" a
228. x = mgL (1- cos rot)
B212
Bl
where, ro = + cB212)L . J (m
PRACTICE PROBLEMS 103

4mRmo -bt 3B2l2


229. 3B2l2 (l-e ),whereb=--
4mR
mgL
230. z = 2,,2a'b2 (1- cos rot)

where,-ro -1Ul'b~ 2_
mL

whereb = ,,2a'B20a 2 cos2 8_


mR

232.

(a) 21 (1- e-2lt/50j Amp.

(b) (252_2e-21t/50) Amp.

L2
235. (L,L2 _M2)C
1 1
~LtC' .J (L, + 2L2)C
Iu .:!!L
237. (a) 2"' (b) .J3

238. (a) ~VO' (b) (~) Vo.

239. [(37.4) sin (rot + 41°)] Amp.


240. (5.l) sin (rot - 32°) Amp.
104: PRACTICE PROBLEr~S

241. 1040 W.
242. 952 J.

243. L=0.2mH,C= U2)1'1"


4
244. L = 81 H, C = 11'1", R = 250 n,
245 . .50 Amp.
246. 10 Amp.

247. 00 = ( ~C+ ::2 ~ (~)

248. (a) LC - 2L2


~

(b) J 2LC~R2C2

249. 1M
2" LC - L2
250. 10 Amp.
251. L = C,R,.R, = C,R,R,
252. 10 Amp

253. ~"3fg

254. 16mms

J +t2 (1 JJSin; ., .
255.
X=[t.(1 J).L~-
cosi
sin2 i
cos i:
Jf.l~-sin'; .
,.
h(f.l2-1)sin3i
256.
(2f.l -sm. 2.t, 2
257. 5 second
~\;>" ::HI")' ("t'(::1'-'\oJ
.,.259.
,
1\= i-A + sino' [:: {<SinAl (:J-sin'; - COSASin;}]

264.
r=R[l+ gl
~2_~: -~1-~
]
265. R/2 behind the polished surface.

266. a=--,e=-
~r 1
~+1 f1

1
267.
l+UrJ
268. 25 em from the second lens on the right side, magnification m =
269. 0.5cm
270. 6 cm back side of unpolished lens.
TA. 15"
271. x=- -
12' 4
272. n.[3
273. e = 0, 300,900,1500,1800,2100,270",3300
274. d
275. 34
2D
276• ..[l5
277. Y = 10.5 em
278. p = 2W A = ~Q.
I 3d' 1'2 3d

279. 0.52mm
280. 0.6mm
. 281. 0.15 JUll

282. 0.21mm
283. 3.33 x 10-' N/m2
284. 1.25 x 10-' N/m'
285. 1.81cV
286. 8.3 msec
\, 287. 2.86 x 1000T
288. 5.12 x 10-' Amp.
289. 47.7eV
290. 6.24 x 10' mls.
n=4,Z=2.
A is ,He', B is Iff'
~ n6h6
241l4c4m"
-me 4
3s~n2h2

140.7 pm
15 x 1000V
r

EXPLANATIONS
PRACTICE PROBLEMS 107

EXPLANATIONS

1. Suppose the velocity of the dog is (vJ + V21).


If after time 't' the dog can catch the rabit, then

(7.5i+101)t = (46i +281)+(vJ+v21)t

Comparing the coeficiants of i and 1 on both sides we get


7.5t-46 = vjt ...(i)

and 10t-28 = vzt ...(ii)

also, we have ~Vj2 +V22= 5

=:> vjZ +vz" = 25 ...(iii)


Solving equations (i), (iiJ and (iii), we get
t = 48, 5.25s
Therefore tmin =: 45

3. Suppose swimmer's velocity v makes an


angle S with the line AC. The resultant
velocity should be along AC. The
components of velocity perpendicular to
AC must cancel out.
v sins = u sina ...(i)
The resultant velocity along AC is ( u cos a + V cos S )

JZ2 + dZ
...(iO
Therefore t=
u cosa + v cose

.[i2;;J2
t =
ucoscx,+Jv2 _u2sin2a
[using equn (i))

.: sina d )
( ~Z2 +d2 .
108 I PRACTICE PROBLEMS

5. Suppose the man swims at an angle S


with the line A B and then walks the
distance CB that he has been carried
away by the stream.
v
Time taken to swim across the river
d
----
vcosS
. d
The distance BC carried away by the stream is vcosS (u-vsinSl.

Time .taken by the man to walk the distance BC is

_d_ (u-VCOSS)
vcosS (j)

Therefore total time taken by the man to reach from A to B is

d d
t = vcosS + vcosS
(u - VCOSS)
(j)

dt
For the minimum time,. dS =0

~ sinll = (_v )
u+(j)

Substituting this value of II , we get

d(U+W)[w(w+u) -vJ(u+w)2 _v2 ]


t =
vw2 J(u+w)2 _v2
7. Suppose the steamer is moving at an angle a. with AB with speed
u as shown in the figures in two cases.
In the first case,
v = u cos a ...(i)
In the second case
usina B
tan45 = ...(ii)
v+ucosa
From equations (i) and (ii), we get
tan a = 2, and u = v>15
A B
PRACTICE PROBLEMS .109

da
9. dt =a.
a =ai
ai"
v=- ... (1)
2

ai3
S=- ...(2)
6

<v>. = -=-at v
s 1 " =- [using (1) and (2))
Now, timet63

Also,
Jvds J:(ia.s
S
-
2

S
r ds 3
--v
5
[using (1) and (2))

11. x" = a'? +2bt+c ...Ii)

Differentiating with respect to t, we get


dx
=:> x dt = at+b

dx
But =v
dt
xv=at+b ...(ii)

Differentiating again with respect to t, we get


2 dv
v +x-=a
dt

dv a-v"
-=--
dt x
From equation (ii),

...(iii)

From equation (i)


'10 . PRACTICE PROBLEMS

t=
-b+Jb2- a(e-x_2)
a

=> (b + at) = Jb2 -ate - x2)


Substituting this value in equation (iii), we get
dv ac_b2
dt x3
13. u2= 12%-3x3 ...(i)

2U:=(12-9X2)U

2/= 12-9x2
2 12-21
x =--- ...(ii)
9
Now, from equation(i)

u. '=(12X~3x3t = 9x2 (4 - x2)

u. =(12-2/l(4- (12~2/))
• 8(6-/)(12+/)2
U 81

U= M8(6- 1)(12+ 1ft

15.

du 2
-+0> ,-V =fk
ilt
Solution of this differential equation can be obtained as

"'= -;'(1-COS<oI)
<0
PRACTICE PROBLEMS 111

dx k
Now - = -(I-cosrot)
dt ro'
k
x ~ -(rot-sinrot)
3
ro

5,/3
17. ax = 5cos30° =--
2

ay = 5sin30=~
2

X" (5 x 5)+H 5:}25) =79.126m

y " H%}25)= 3L25

Displacement =J x' + y' = 85.1 m

Distance, s=f (:f +(:f dt

s "f: ( 5+ 5,/3
2 t r (5 f dt
+2"t

=} s~85.3m

19. Suppose velocityofparticle Ais v and that


ofB is 2v. y
v 2v
Then Vjj =(2vcos9)i +(2vsin9-gt)J
x
and VA=(vcos9) (-i)+(vsinO-gt)J

Now, Vjj,VA = 0
=} _2v2 cos2 o + (2vsin 9-gt) (vsin 9- gt)=O

t=.l!-(3sinO+J 1+7cos2 0)= 2v (.: 9 = 30)


2gg
112 ' PRACTICE PROBLEMS

Relative velocity lJRlA= (3vcose)i + (vsin e))

.. IlJRIAI=v~1 + 8cos2 e =v,/7


Separation between the particles

d = (v,/7) (~) = (2,/7):


21. Select a coordinate system with the origin at the point of
projection. Suppose the coordinates ofthe point where projectiIe
strikes the inclined plane is (x, Y).
y

k. II x 1I1)real &1
I-- d---l
Now, y = (usine)t- ~gt2 (i)

and x = (ucose)t (ii)


From the geometry of the figure,
d = x+ y cota ...(iii)
From equations (i), (ii) and (iii)

d = (ucose)t+((u sine)t - ~gt2 )cot a ...(iv)

For imin, ~ =0
de

tane = cota

=> e =~-a
2.
Nowsubstituting this value of e in equation (iv)

t.min =
( --------
u_~u2 -gdSin2a)
gcosa
PRACTICE PROBLEMS . 113

23. Suppose the range is R corresponding to the angle of


projection a. Then
R = (ucosa)t ...(i)

-h+~gt2 = (u sina)t ...(ii)


2

r
From equations (i) and (ii)

=u2 t2
2
R +(~gt2 -h

=> ~g2 t4 _(gh+u2)t2 +(R2 +h2)=O


For the maximum range. t2 should be unique.
2
Therefore (gh+u2) _g2 (R2 +h2)=O
uJu2+2gh
=> RMAX = g
.
25. PN is perpendicular to OB for motion parallel to OB.
A B

o
o = (uSin
<1» - (gSin ~)t ...(i)
For motion parallel to PN (perpendicular to OB)

= (u cos <I»t- ~ (g cos ~)t2


asin <I>
...(ii)
Also, = 1t-(a+~)
<I> ...(iii)
'From equations (i), (ii), and (iii)

. 2 2ag sin~
u = ...(iv)
sina -sinJ3(cos(a+ J3))
Now from equations (i), and (iv)

t = .1_--,--,..:2=a:.:s::in::.2~(.::a_+~J3!...)
---""
g(sina-sin ~cos(a+J3))
27. Time flight = ( Time from beginning to impact with thewall)
114 PRACTICEPROBLEMS

+ ( Time from wall to the floor)


2 usina. x x
=> --- - --+----
g - ucosa 2u +ucosa

2u2 sinacos a(2u+ucosa)


=> x =-----------
g(u+ucos a)

29. d= J(~)\H2
(u2 sin; cos a r +(u r2
~;2a

2
= u sina ~(1 + 3cos2 a)
2g
H
Now, sin 0= -
d

where

(ucos a)sinO H(u cosa)


Now Ol= =
d d
2gcosa
Ol=
u(I+3cos2a)

sinll-vl sina
V2
31. Ol=------
d
V2(11-v1t)-V1(l2 -V2t)
=> Ol = d2

Ol=--------
v211- v112
(12-V2t)2 +(11-V1t)2

For Olmax; :t ((12 -V2t)2 +(11 -Vlt)2)= 0

11V1+12V2)
t = ( 2 2
Vl +v2
Substituting, this value of t, we get
PRACTICE PROBLEMS . 115

2
33. ())1 R1 ~40

()), ~ 20 rad I s
v ~ lJ)IRI ~ 2 mls
ro.R2 =v
=> lJ), = 40 radls
Sinee u2R2= 30

ul R1 ~ u2 R2 ~ 30 m I S2

Now (())12Rl)2 +(U1Rl)2 ~50m/s

- ./1 Here, 00~ - 2k ,u ~ 4k


/' r = (4£+3J)
Now, ii = rox(ooxr)+(iixi')

a = (-28£ +4J)
37. It is given that 00= 4krad/s. ii~-10krad/s2. r~ B£em
-+
-+ dr •
v•.•l ~-~5iem/s
.dt
-t ....• -+-+
Now, u ~ Vrel+coxr

and

'\
\
116 PRACTICE PROBLEMS

39. It is given that

ro = (4i+3k)rad/s

and r = (3}-4k)m

therefore
I
v = (-9i+16) +12k)m/s I
I
41. From geometry of the figure I

CA = (sin75)4m J
sin45
= 5.46 m
Since, the instantaneous axis of rotation passes through point C.

Therefore, <o::;:~
B 30'
(CA)
4
=> ro= --= 0.732rad/s
5.46
Also Va = ro (CB)
A

= ro[( s~n60}]
sm45
= 3.6 mls
Now aB ::;aA +aB/A

=iiA+ (iiB1A)N +(iiBfA)T


Also, (aBfA)N = ro21
(aBfA)T = al
Using the vector diagram of acc'Clerations, we have
(aB) cos45=(aA)+(ro21)cos30-(al)cos60 (i)
2
(aB)sin45= (ro 1)sin30+(al)sin60 (ii)
Solving equations (i) and (ii), we get
a = 105 rad/s2
and aB = 6.65m / S2
PRACTICE PROBLEMS 117

43. The lengths of the rods remains same. For the rod BC the
component of velocity of end C along CB must be equal to the
component of velocity of end B along Cll. Therefore,
v cos (30 -8) = 3 cos 60 ... (i)
Similarly for the rod AC
v cos (30 + 8) = 2 cos 60 ... (ii)
Solving equations (i) and (ii), we get

v = ~M1S
c

45. Suppose the coordinates of A and Bare (0, 0, z)


and (x,y, 0) respectively.
Then x" + y" + z" = /" .....(i)
,
t"A
A (o,o,z)

Now, it is given
y =0.6 m,z= 0.6 m, / = 1m
Hence, from equation (i), we get
x= 0.53 m
Since, the values of y and I are fixed, therefore,

: ={~):
VB= (~}A (.; :: ~VA and : =-VB)
- (~)(02) m!s --
VB - 0.53 . 22.7 em!.,
\
118 PRACTICE PROBLEMS

47. From the geometry of the figure


~a,

a,i
m. -a2
a. m,g T,
a, = asec 0 ....(i)
az = a,cot ex ...(ii)
N
For the block of mass m,
m,a =T,-m,g .....(iii)
For the slider '8'
mza, = Tz - T, cos 0 ....(iv)
For the vertical motion of the
block of mass m3
m~, = m"g - T, - N cos ex .... (v)
For the horizontal motion of the block of mass
m.az = N sin ex .•... (vi)
m.
After solving these equations, we get
g(m3 -m, cos 9)cos0
a=
(m, cosz O+mz +m3 +m. cotZ a)
49. Suppose acceleration of the wedge is 'a' and the acceleration of
the block of mass m with respect to the wedge is a,.
From the geometry of the figure.
a, = a (l + cos 0) .... (i)

For the motion of the block of mass m,


mal = mg sin a + rna cos a - T ...(ii)
N=mgcosex-masinex .....(iii)

I
/
PRACTICE PROBLEMS 119

For the horizontal motion of the wedge,


ma=T+Tcos8-Tmsa+N~na ....(iv)
After solving these equations, we get

mg(1+cos8)sina
a ---------------
M+m((1+cos8-cosa)2 +sinZa)

51. For the block ml

N,

mial =m1gsina,+ffi1acosa-T ...(i)

N1=m1gcosa-m1asina ...(ii)
For the block m2
m2al = T-mzgsin~+m2acos~ (iii)
Nz =mzgcos~+mzasin~ (iv)

For the horizontal motion of wedge,


m3a= Tcos~-Tcosa+Nl sina-Nzsin~ ... (v)
From these equation, we get

(~ +~ )m, sinacosa - mz sin~cos~ )-(sina+sin~)(cosa-cos~) '.


a= ~--~--------------------- g
(~ + ~, }mz+ m sinz a+mz sin2.p)+(cosa-cos~)z
j

53. The direc,tion of friction forces are shown in the figure.


(jj)MAX = Ilj (mjg+Fsina.) ... (i) I m,
fl c

(fZ)MAX =Ilz(m,g +mzg+Fsina) ...(ii)


The equations of motions are
Fcosa-fl = mja ...(iii)
fl- f2= m2a ...(iv)

f?".
120 PRACTICE PROBLEMS

From equations (iii) and (iv)


Fcosa-f2=(ml +~)a ...(v)

If the plank moves on the ground, then


Fcos a>(f2)max

f12(ml+~)g
F>------
(cosa- f12.sina)
From equation (v)

Fcosa-(f2)
a::::: max
..,(vi)
ml+~
Now from equation (iii)
fl = Fcosa-"'Ja
=>. (Fcosa-"'Ja) < (fJl max

From equation (iv)

. (FcoSa-(f2) )
F cosa- ml max < (fJl max
ml+~

Substituting (f,I) rnf!x and (f2) max


, we get

. ..' . ml (ml +~)(f11- f12)g


f < (~(cosa-f12sina)-ml(f11 +f12)sina)
55. From geometry,

...(i)
For cylinder
"")'
, .•• t ~':
"'Ja= "'Jg-2(N cose+ fsine)
For prism
m2al = Nsine-fcose-f' ...(iii) .
R = ~g+Ncose+fsine ..(iv)
PRACTICE PROBLEMS •. 121

Also f = I1R ...(v)


and f = I1N ...(vi)
From these equations, we get
g(m,k-21""2 (cos 9+flsin9»)tan9
a=~---------~--
mk tan 9+2~ (cos9+flsin9)

where k= (sin 9-2flCOS9-fl2 sin 9)


57.

T,sin9, = T2sin92 ...(i)

T, cos9, = m,g ...(ii)

T2cos92 =~g" ...(iii)

T, sin9, =m,ro2l,.sin9, ...(iv)

T2sin92 = ~ro2L"sin92 ...(v)

From these equations

2
dv mv2 2
59. mv cIs = '-fl (II ) +(mg)

-J.' o
f1ds
R

_ Rln[V 2 (v/)2]"
8 --- --+ 1+-o
2,. Rg Rg
122 PRACTICE PROBLEMS

61. Suppose the velocity of the particle at the deflected position is Ub


From conservation of energy

.!.mu/ = .!.mu2-mgl (I-cos 9)


2 . 2

=> u/= u2 -2gl(l-cos9)


Radial acceleration at this position is
2
VI v2 -
aR=- =- - 2g(1-cos9) g
I I
Tangential acceleration at the position is
ar=gsin9
Net acceleration

2
=> a = (u -2g(l-cos9)2 )+(gSin9)2
1
Therefore, net forCeacting of the particle is
F =ma

F = mg Sin29+(~-2(I-COS9)r

63. Suppose the particle leaves the


sphere at point D is shown in the B

figure. Then the particle follows


a parabolic path.
If the speed of particle at D is u.
then )'

.!.mu2 =mg~(I-coS9) ...(i)


2 .. .
mu2 A p
and --=mgcos9-N Rsn9
R
Th leave the sphere N = 0,
.~~~
mv2" .
. -- =mgcos9 ...(ii)
R
From equation (i) and we get
(ii),
2
cos9=-
.3
PRACTICEPROBLE~S 123

sinO = J5
3
Now for the parabolic path,
x = (vcosO)t ...(iii)

and y = (vsinO)t+.!:.gt2 ...(iv)


2
Also y = R(1+cosO) ...(v)

From equations (iii), (iv) and (v) , we get

x =4(5~~J5r
Now AP=RsinO+x

65. From geometry


OC-OA=2ae ~ .~c

OB -OA=( Jb2 + (ae)2 )-a(l-e) A~b,--: __ ~

= a-a(l-e) = ae

[.:e=g)
From conservation of energy, for the points A and C

.!:.mv 2 = .!:.k(2ae)2 ...(i)


. 2 02
For the points A and B
1 2 1 2 1 2
-mvo .+mgb = -mv +~k(ae) ...(ii)
2 2 2
From equations (i) and (ii), we get

3 2
V = -v +2gb
4 0
67. Suppose A. is the mass perunit length of the chain.
Then for equilibrium
124 ' PRACTICE PROBLEr.1S

,,=~
1-'l
Work done.against friction,

W= rl-.)1 ,,(A.>:)gdx= .!:.""g(I_'l)212 _ 'l(1-'l)J.g12


Jo 2 2
Decrease in P.E.,
AU = Uunlol - Ufinal

.!:.(LA)v2 = •.g12 (1- 'l)


2 2

~ v=J~~-~
69. Suppose the wedge gets
displaced towards right by a
distance x and the horizontal
displacements of m, and m.
are x, and x. respectively
towards left with respect to
the wedge. The centre ofmass
of the system remains
undisplaced in the horizontal
direction,
Therefore

ml.r1 +~~
=> x = -----
m"+~+,,.,.
Now, from geometry
x,= lcosa and x2 = lcosll

l(m" cosa+~ cosII)


x = m"+~+,,.,.
PRACTICEPROBLEMS 125

71. Change in momentum


= (force) x (time)

=> 3x3+(2+5)u= (5xlO)x1

41
=> u=-m/s
7

1 2 1 (41)2
KE = "2(3)(3) +"2(2+5) 7
= 133.57 J

73. Suppose the speed of the particle


with respect to the hemisphere is
VI and speed of hemisphere is v.
From conservation of momentum
mv = m(v, cose -v) ... (i) v
From conservation of energy

%mv2 +%m[(v1 cose-v)2 +(Vl sine)"]= mgR(l-cose) ...(ii)

'lb leave the contact


Normal Reaction, N = 0

__mvR,_= mgcose
2
...(iii)

From equations (i), (ii) and (iii), we get


cos3 e-6cose+4 =0

cose = ,J3-1

=> e = cos-1(,J3 -1) = 43°


75. Suppose the velocity of the platform just after jumping of the man is
v"

in
!v, m 1.m+Miv,
Then m(vo -vI! = [(M+m)+mjvl ...(i)
126 , PRACTICE PROBLEMS

Retardation of the plat form


(M +m)g-mg Mg
a ~ ~ ...(ii)
M+2m M+2m
2
Now h ~ '!.L
2a
M(M+2m) vD2
h~------
8(M+m)
77, Total time

(vsina)g 2e(vsina) 2e2(vsina) 2vsina


T~2'--g--+--g-_'+ __ g __ +'" ~ g(l-e)

Total distance,

2
d- 2(vsina)(vcosa) + 2e(vsina) (v cosa) +.2e (vsina)(vcosa) +...
g g g
v2 sin2a
g(l- e)
79. Suppose the velocity of ml' just after collision with. bullet is v,,
Then
mVD~ (m+ml)vl ... (i)
When the block m stop sliding with respect to the plank, then
common velocity v2 is given by
mVD~ (m+ml +m.)V2 ... (ii)
If the displacement of m, with respect to m2 is x, then
2
1(m+m1)v/ _1(m+m1 +m2)v2 ~ f.1(m+mdgx ... (iii)
2 2
From equations (i), (ii) and (iii), we get
2
m "'-2v/
x ~ ~
2f'g(m+ml +m2)(m+md
81. Suppose along the line joining the
centres of the sphere, the Before
component of velocity of sphere A is .. Collision
V 1 and perpendicular to this line is
v2• The resultant of v, and v2 is VA
as shown in the figure. The
cOmponent of velocity
perpendicular to the centre line
remains unchanged.
PRACTICEPROBLEMS 127

Therefore v2 = vsin30° ...(i)


Along the centre line
mivcos30o= mivi +"'2vB ... (ii)
and vcos30o= VB -VI ... (iii)

From equations (ii) and (iii), we get

VI = ("" -m2 )VCOS300 ... (;V)


""+"'2 .
Now, since VAis resultant of v! and v2' therefore
V2 sin 30° = vi sin 60°

0 0
=} (vsin300)sin30° = [(:: ::: } cos30 }in60

Using equations (i) and (iv), we get


ml.= 2"'2

ml =2
m2
83. There is no impulse in the direction perpendicular to the
common normal. Therefore, we have

llJ sin a = Viain(90-a) ...(i)

and ~ ainJ3 = v2 sin(90-J3) ...(ii)


From the conservation of momentum
"'2v2 cos(90-J3)-""VI coa(90-a)
= mlu1cosa-"'2u2
cosJ3 ...(iii)
"Form the definition of coefficient of restitution
V2 coa(90-J3)+vl cos(90-a)
=e[ul cosa+~ coaJ3] ...(iv)
From equations (i), (ii), (iii) and (iv), we get.
2 2
"" sin /3+"'2 sin a
e = ml cos213+m2 cos2 ~
128 PRACTICE PROBLEMS

85. Suppose after the impact the velocity of


sphere ml is v, in the horizol1tal direction
and the velocity of sphere m, is v along
the common normal.
Applying conservation of momentum in
the horizontal direction v,

m,.v1 +m2vsina. = m2usino. ...(i)


Along the common normal,
VI sino: -v= eu ...(ii)
Solving equations (i) and (ii), we get
m2(1+e)u8ino:
v =------
I ~+~sin2o:
87. Suppose the velocity of the shpere after impact is v along the
common normal and the velocity of the hemisphere is VI along the
horizontal direction. If the impulse due to impact is J, then for
the hemisphere

MVI = Jsin9 ...(i)


and for the sphere along the common normal
mv = musin9 -J (ii)
Also vIsin9-v= eusin9 (iii)
From equations (i), and (ii) and (iii), we get
(1+e)musin29
vI------
M+msin29
89. Suppose the velocity of the trolly after time t is v toward8 right.
The velocity of the container is also, v vertically downward.
If the tension in the string at this moment is T, then the equation
of motion ofthe trolly will be
dv
(mo +fll)-+I'V= T ...(i)
dt .
and equation of motion of the container will b.e
dv .
(mo -fll)di = (mo -fll)g-T ...(ii)
PRACTICE PROBLEMS 129

From equations (i) and (ii), we get

dv +(-L)v ~ (.!_~)g
dt 2"'0 2 2"'0
Solving this differential equation, we get
V ~ 3mog (l_e-J'l!2"'O)_ gt
11
91. The equation of motion of the disc is
d udm
dt(mv)~F-dt

=> Fdt ~ d(mv)+udm


=> F.t+"'ou ~ m(v+u) ...w

dm
Now -~pA(v+u)
dt
~m ~pA(Ft+"'oU)
=> dt m
t

=> J~m ~ J pA(Ft+"'ou)dt


o

=> J:pA(Ft+"'ou)dt

=> m" ~ "'0" + pA(Ft" +2m.ut) ...(ii)

From equations (i) and (ii), we get

v ~ ( "'0" +;t(;~:2"'out) )-u A


93. Moment of Inertia about an axis through the ,J; I

WW
, ,,
Ie I I

flat face of hemisphere ' , ,


2 : 5R :
1 :;:: --mR2. '8
0
5 ,G I

". ' ~ R:
From the parallel axis theorem, the mome~~_o[~
inertial through c.m. of the hemisphere ""'\ '.

IG ~ I
o
-m(!R)" ~( )mR"
8
83
320
~\.
130 PRACTICE PROBLEMS

Using parallel axis theorem, Moment of inertia about the axis


AA' ,

I =IG+m(~Rr

~ 1= G~)mR2
95. Suppose the velocity of the particle when it reaches to the topic
top is v.
From conserVation of energy
1 2 1 2
-mvo = -mv +mgRcosO ...(i)
2 2
Now, applying conservation of angular momentum about the
verticle axis through 0, we have
mvo(RsinO) = mvR ...(ii)
From equations (i) and (ii), we get
Vo = ~2gRsecO
97. Suppose the radius of the cylinder is R and length of rod is 21.
COnsider the Clisewhen the end A slids up. Forces acting on the
rod are shown in the figure.

Resolving fo~es horizontally ll!1dvertically, we have


Nz =N,cosa+J1N,sina (i)

and N,sina= J1N2+J1N,cosa+ W (ii)


Taking moment about A,
N,(2R coseca) = W(lsina) ... (iii)
From equatio!ls (i), (ii) and (iii), we get,
2R = l[(1-,,2)sina-2I1COsaJsin2a ...(iv)
Similarly, when the rod makes lell$t angle II, we' get

2R = l[(1-,,2)sinll+2"COSII)sin211 ...(v)
PRACTICE PROBLEMS 131

.From equations (iv) and (v) , we get

3 3
tan[ltan-'( sin a-sin /3 )]
J.l = 2" sin2acosa+sin2/3cos/3
99. Suppose the acceleration of the block B is a, acceleration of disc is
a. and the acceleration of centre of mass of spool is a" also
suppose the angular accelerations of spool and disc are a, and a.
respectively.

Now

aIR
. => a=-- ... (i)
2
at = aIR ... (ii)
a. =a] (2R) + a2 (R) ...(iii)
For the block
rna =T, ...(iv)
For the spool
... (v)

2
. (mR
-2-)
al = T2R+T]R+fR ...(vi)

For the disc


rna. =mg-T2 ...(vii)

(imR2 )a2 = T2R ...(viii)

From these equations, we get


4
a=-g
37
132 PRACTICE PROBLEMS

101. Suppose the acceleration of cylinder with respect to the wedge is


a and the acceleration of the wedge is a'. If the radius of the
cylinder is R, then the angular acceleration of the cylinder will be

maO

a
u =- ...(i)
R
For cylinder

...(ii)

rna = mgsinO+rna'cosO-f ...(iii)

N = mgcosO- ma'sinO ...(iv)


For wedge
Ma' = NsinO-fcosO ...(v)
From these equations, we get
2mgsinOcosO
a' = 3M+m(1+2sin2 0)
103. Suppose C is the point through
which the instantaneous axis of
rotation passes and G is the
centre of mass of the rod. From
the geometry of the figure

CG= lcos30° A

The moment of inertia about C


ml2 5
I =
-+m (lcos300)2 = _ml2
12 6
Ifa is the angular acceleration, then

(~ml2)a = mg({COS300)

3.J3g
=> u= lOT'
PRAqTICE PROBLEMS 133

105. Suppose the centre of mass comes


down by a distance h and the moment
of inertia about the instantaneous
axis of rotation ( through point P) is
I . Then, from conservation of energy
P .'

1
mgh = _I p
oo2
2
2
mg..!.(I-sine)
2
= .!.[mI
2 12 . 2
+m(..!.cose)2]oo2

3g(l-sine)
00 = 2
1(1+3cos2e)

From geometry, PA = ..!.sine


2

and

Now

3gl(l-sine)sin2 e
2
1+3cos e

Also

=> VB = ~3gl(l- sine)


107. Suppose after collision the angular velocity of
rod is 00, the velocity of centre of mass of the
rod is V and the velocity of the striking
particle is v, in the backward direction.
From conservation oflinear momentum
+-{ji)
m v, m

mVD = (3m)v-mvl ...(i)


From conservation of angular momentum about centre of mass
134 PRACTICE PROBLEMS

(mvo)~(sin45°)= (2( m(HJ+ ~~2)-(mvd~(sin45°) 0) ... (ii)


From conservation of mechanical energy

1 2 1 2 +-(3m)v
1 2 +-
1( 2m ( -1)2 +-
2
ml J 2
-mvo = -mv] 0)
...(iii)
2 2 2 2 2 12

From equations (i), (ii) and (iii)

36,/2 Vo
0) =
651
109. Suppose the particle strikes the hemisphere with a velocity Vo and
the angular velocity of hemisphere just after collision is 0). Then
Vo = J2gh ... (i)
From conservation of angular momentum about the line joining
the supports

mvoR = (mR2)0) + (iMR2) 0) ...(ii)

Applying conservation of energy after collision

i(mR2)0)2 +HiMR2}2 = Mg (%R )-mgR ... (iii)


From equations (i), (ii), (iii) after substituting the given values we
get
h=42m
5
and O)R = -vo
21
Now select the axis as shown in the figure.
The reactive impulse

J= [mO)R(-k)+M(% R" )(-i) ]-(mvo(-k))


Substituting the numerical values, we get
4J = 2~
V( -
16k-15i
_)

when V o = J2gh
Then IJI = 30kgm/s
PRACTICE PROBLEMS 135

111. Suppose the centre of mass ofthe system is at C as shown in the


" figure.
rl+rz~R ..Ji)
and Mr1 =mr2 ...(iO P
III
" mr
From these equatIOns rj ~ --
m+M
MR
and rz~--
m+M
Suppose the angular velocity of back'¥ard spin of the ring is ill ,
then from conservation of angular momentum about the centre of
mass of the system
mvrz -(mrz 2)w::: (MR2 +Mr12)0)
Substituting the values ofr, and r2, we get
mu
'" ~ ----
R(M+2m)
Now suppose the angular velocity of the insect with respect to the
ground is "', then
0)1 ::: "
R -co
Substituting the value of "', we get

113.
"',~~(::2:)
When the slipping ceases
ro'lRl =ro'zRz ...(i)
If the friction force is (between the cylinders, then
Il(""I-"'tl~ -(Rlt ...(ii)

and Iz(""z-"'z)~ (Rzt ...(iii)

Also
1 z l(
II ~ 2'm,R, ~ 2' "R, lp RI
Z)" z ..(iu)

and 1 zRz 2 ~ 2'1(1t RzZ)lp Rz2


12 ~ 2'm .. (5)
From these equati )Us, we get
' (DIRt 3 +CiIzRz 3J
Ol I:::
( R,(R/
-------

+R':)

",' ~ ["'jR/ +"'2RI3)


and z Rz(Rjz+R/)
136, PRACTICE PROBLEMS

115. g = -
GM ... (i)
R2
and Ve = 2gR J ..(ii)
Suppose the velocity of the body at a distance from the earth's
center is v. Then from conservation of energy
1 2 GMm 1 2 GMm
-mv ---:= -mv ---
2 r 2 e R

v = R~2:
dr =R~2g
dt r

t
J,odt = R,,2g =
1 f,R+h
R
,Jrdr

117. Suppose tension is F in the rod at a distance x from the end about
which the rod is being rotated. Then

F_ j/(mdx)ro x
- x l
2 =
2 2
mro2 l (1-:£)
l2
Now the elongation in the rod

119. Consider a rectangular strip on the semi-circular gate as shown


in the figure. .
y= RsinS ...(i)
x= RcosS ...(ii) r
d<=y(dF)
~ d<= y pg(H -y)(2xdy)
H
1
Now using equations (i) and (ii), we get
{"12
T= 2R3pg J (H-RsinS)sinScos2SdS
o

T = 2Rpg 3 (H
---
3
ltR)
16.
PRACTICE PROBLEMS 137

121. Suppose at any instant h is the difference in liquid levels, h, is


height of level in left container and h, is height of level in second
container.
Then
dh = dh, +d"-2
and Al dh, = A2 d"-2
dh, ( A2 )dh
dt = AI+A2 di ...(i)

Also Al ~; = - a~2gh ... (ii)


From equations (i) and (ii)

AIA2 )dh = -a~2gh


( Al +A
2 dt
-AIA2 rO dh
~ t = (AI +A2)a JHbgh

~ AIA2
t = J2H
(AI +A2)a g
123. If the new ,'adius of the mercury tablet is R' , then

(7tR2)h= 7t(R,)2~ ~ R'= R../n ... (i)


n
From the geometry
h
r = 2cos'"
e = 180 - '" ... (iii)
Initial excess pressure inside drop
T 2Tcos'"
p=-;:= h [using equn (ii)]

27tR2Tcos'"
Initial force F-p7t
-
(R2) =----
h
2
Similarly final force F'= 27t(R') Teos'"
hln
Now F'-F= mg
27t(R,)2Tcos", 27tR2Tcos'"
-------------- mg
U~) h
13B

cos<l> =
I • PRACTICE PROBLEMS

[using equn (ii)]


I
Hence, from equation (iii)

e = ,,-cos-1 [
2"R2
mgh
+(n2 -1)
J
125. Consider an elementary cylindrical layer of radius r.
Then t =fr

=>

f d3 = 2"'11f.wdw
b
r
=> t
a r 0

=> t
a2b2
= 4"'lIW ( 2 2
J
b -a
127. Consider an elementary disc of a distance x from
the vertex. Radius of this elementary disc
r= xtana
Now moment of inertia about the line through
suspension point .

Now

T=2" ~(4+tan2a)
5g
PRACTICE PROBLEMS 139

129. Suppose the length of the rod A


is L and the mass of the rod is
m.
In equilibrium, tension in the
thread

F=mg f.")
••• \1
2
From geometry,
and Ie = L~ ... (ii)
Now, equation of motion is

(mLZJdZ~
-
3
-
dt Z
= -(Fsine)L

=-FLe (.: sine ",e)

= - (~gJL(~~J [Using (0 & (ii)]

=_( m~f2}
dZ~__
dt Z
(3gJ~
21
-:

f3i
",,/ ill = V21
Thus, T = 2"
ill

T = 2" f2l
'lSi
131. Suppose P, =P
Then pz = ap
Z
P3 = a P
For equilibrium
. A(l, +lz)apg = Al1pg+AlZ<iLzpg
=> ~=a~
For downward motion
dZx
A(l\ +lz)ap dtZ = A(ll +.lz)apg

I
140 I PRACTICE PROBLEMS

-A(12 +x)a!pg "


-A(I! -x)apg
d2x
--;;;;:
2
-001 X
2
dt

where, "', = ( 1,2+I?


1,12(1, +12
))g
For upward motion
d2x
A(I, +12)ap-2 = l,pg+(12 -x)a2pg_(I, +12)apg
dt
d2x 2
dt2 = -"'2 X

I,g
x
where, "'2 = 12(1, +12)
.•••••••••. :::::. ~J. : : : : :
Now time period
~.~-x~.--

" (_11_12_(11_+_12_)J
+"
T :-
- (2I, +122) g

133. x = a cos 3",t


...(i)
y = b sin ",t ...rii)
From equation (i)
x = acos",t(1-4sin2",t)

x~ a()l-sin ",t 2
)(1-4sin2"'t)
l'
Now substituting sinwt~ ~ from equatio
;.

x
PRACTICE PROBLEMS 141

135. Suppose the equation of the wave is

y = aSin2"( ~ - ~)
In the first case
0.1 10)
6 = 8 sin 2" ( T-T

~ ( 1 T10) = (49) 180"


2" lOT -

1 10 49
...(i)
lOT •. - 360
In the second case

2" (:....!..- _ 25) - 6"


lOT •.

1 25 1
~ 10T-T = 12 ...(ii)

From equations (i) and m), we get


•. = 284.2 cm and T = 0.58 second
, 137. Comparing the given equation with

i,,' 'I' =a sin (rot - 2•." (.>:cosa+ycostl}).


,
I
, .J3 and
1
we get COSa=- cosll=-
2 2
and •. = 1m
I
I .J3
I
Now, COS<X=-
, 2
~ a= 30"
1
and cosll=-
2
~ II = 60"
142 PRACTICE PROBLEMS

(a) The wave is moving along a straight line in x-y plane through
origin making 30° with x-axis and 60° with y-axis.
(b) The wavelength of the wave is 1m.
2"
(c) A~ =T [(x2 -x.Jcosu+(Y2- y.J cos f3]

_ 2 " [(/3)cos,30° +(1) cos60°]


a
,=4n

139. (a) PartiCleve'locity

Att=O

Now

b
x= ..[3

1 2"
141. P = -"or ,a ,v
2
PRACTICE PROBLEMS 143

Now (0 2,W
= --and
Ie
v= f!
-
fJ.

3
Therefore P = 2,,2a2 ~ T .
,,2 fJ.
Substituting, a = 10 mm, " = 0.5 m, T = 500 Nand fJ. = 0.8 kg/m
we get
P =98.7 W.
143. Energy of oscillations equals to the work done against tension in
. giving the initial extension in the string. Therefore

145. In the nthovertone

2" "
K =T = (2n + 1) 21
Amplitude of the pressure variation

pv
M'm= pv'AK= -1-
2
A" (n+2"1)
The equation for pressure variation
Ll.P= Ll.P
m
cos Kx

P = (n+.!)
.2
"pv A I
2
cos [(n+~) "t]
147. Frequency of the sound received by the wall,

n n (v+w-.u)
1 +w -
z ~ V Vs
...m u

.vo • v,
The waves reflected from the wall are
received by the observer. The frequency
received by the observer

....cii)
144 PRACTICE PROBLEMS

Substituting n, from equation (i) in equation (ii), we get

n = n (" ++ OJ - u) (u - OJ + "0)
o v co - Us V - co + u

Wavelength of the sound received by the observer


).. = "-w+u
n,
Substituting n, from equation (i), we get
).. = (v-OJ +u)(v+OJ -v,)
no (v+OJ -u)

149. The equation for standing wave will be in the form


z=A sin ~x sin K,y sin ('" t + 8)
Since, at t = 0, the centre passes through mean position, therefore
8=0.
the edges of the plate are clamped.
Since,
~ l = n, 11 and K, l = n, 11
For the fundamental model
nl = n, = 1
11 7t
Thus ~= T and also K, = T
Hence, the required equation is
. 7tX . 7ty •
Z = A sm -l- sm -l-. sm '" t.
151. Final length of the rod AB will be
, l Fl
II = J (l + ale) - -- i
Y,A
Final length of the rod Be will be
l~= l, (1 + a,e) _ Fl2
Y2A
Now l, + l, = ~ + 4.
F (llal + l2a2}YI Y2 e
A = ~Y2 +l2Y1
PRACTICE PROBLEMS 145

Time taken by transverse wave pulse to travel from the end A to


the end C will be

Ii I;
t=
V
r=F+ IF
AP; V AP;
But it is given that z;. '" I, and I~ '" 12
I I
t ",_'_+_2_

ffJI
I'Y2 +12Y,
9(1,0., + ~<l2) Y'Y2

153. V2 = V, (1 + YHg(82 -9,»)

A.12 = A,I, (1 + YHg(92 -9,»)

I =AI (1 + YIW(92 -9,»)


2 " A, (1+2<lg (92 -9,»)

= I, [1+ (YHg - 2<lg) (92 - 91)]


Now, Readingonsc~e

1- 12 = z,[1+ (YIW- 2<lg) (92 - 9d]


- 1+<lg(92-9d 1+<lg(92-':9d

= I, [1+(YHg-3ag)(92 -91)]

Substituting the v~ues, we get


I '= 1.0155 m

155. llmgv =- (~7)L


rmomdm = - I'gf(vo
Lo
- Mt)dt
146 PRACTICE PROBLEMS

dT
157. H =-KA dx

a
Suppose K--
-T
aA dT
Then F!.=- T dx
1 T2 dT
Now JHdx
o
=-aA JT
T,

T,
HI =aAln T 2
...(i)

x T dT
Also JHdx =-aAJT
o. T1

T1
=> Hx =aAln T' ...(ii)

From equation (i) and (ii)

T =T, G:r
159. Since, the radial curre~t from inner cylinder to the outer cylinder
should be equal, Therefore
T1 -T
In (2a1a))
( 27tK1i

~ ,

161. According to Newton's Law of cooling


de
di =-a(e-eo)
PRACTICE PROBLEMS 147

"1 dS '
1S_8 =_jadt
6) 0 0

S2 - 80
In S _ S =-ut ...(i)
1 0

"3 dS T
js-s =_jadt
82 0 0

83-S0) ...(ii)
In ( S2 -So =-aT

=::)
In(S2 -SO)]
From equations (i) and (ii), we get T = t In (::
[
82 - So

163. The temperature of the gas is constant, therefore


PV = constant

pdV+VdP =0
dt dt

VdP = -rP
dt
,
J dP = j
o
rdt
Po P

In ~ = -rt
Po Vo
P = Po e-"/V,

165. The equation of the process AB is


P = 5P _ 2PoV ...(i)
o Vo
Now PV =nRT
Therefore, from equation (i)
2
nRT =5PV- 2POV
, Vo
148 PRACTICE PROBLEMS

Now, dT 5
forT ,- =0 => V= -Vo
mUdV 4

167. Suppose TB = To and Vc = VD = Vo


Then TD=T,=4To andV,= (s[2)Vo

Now

and

3
= - 3RTo - 2" RTo + 0 + 14 RTo In 2
= (5.202) RTo
~ e absorOOd ::; L\.QCD + AQDA

=3RTo+ 14RToln2
= (12.702) To

efficiency, '1 = AW _ = 0.41 = 41 %


. L\.Qabsorbed

169. (aJ Given AW=-


Q
2
From first law of thermo dynamics
AQ =AU+AW

AU = Q
2 '
dU =dW
CvdT =PdV

(~)
y-l
dT =PdV
PRACTICE PROBLEMS 149

=>
3
"2 (PdV + VdP) = PdV (.:y =~)
=> PdV + 3Vdp =; 0

dV 3dP
=> - +-- =0
V P
=> VP' =.constant
=> PVll3 = constant

dV
(b) C =Cv+P-
dT

3 3
= -R+ -R=3R
2 2

171. Suppose the upper piston moves upward by a distance y when the
h
lower piston is moved upward by a distance "2' Then

Po(Ah) =PA(h-i+y) =nRTo (i)

and (P - Po)A = Ky (ii)

From these equations, eliminating Y. we get


. 1 1
p2 (2PoKAh) + P (2A'Po - KAh) - 2N = 0

=> Po =.!. _ (PoA) +


P 4 2Kh

Po
Now, work done, W = nR ToIn p

1 JPoA)
W=nRToln ['4l.2kh +

173. Suppose at the point B the tension in string just becomes zero.
Therefore
150 PRACTICE PROBLEMS

2
mVB
,

qEsina =mgcos', ...(i)


2

an d mOB
-/- = q E'COS a + mg sm a , )
...,ii

Applying conservation of energy between points A and B, we get


1 2 1 2
ZmoA = ZmoB + mg (l + / sin a) + qE (l cos a) ... (iii)
From equations (i), (ii) and (iii), we get

175. If the relative velocity of separtion of particles is 0 for a separation


x. Then the relative acceleration.

(where k = _1 )
4"£0

vVV=
d 5kq2
__
IX .dx2
I
o .6m d x

0
2
2
= 5 kq2
6m
(.!._.!.)
x d

dx
=
dt

t
I
o
dt
= t 3d[i; md
5kq2 d
j --dx
d-x

- j '_. '- ••
PRACTICE PROBLEMS • 151

md2
t = - [f6-ln(3-.J2)]
~ 5kq2 .

d
t = ~ 2.4 Bomd 7[ [J6 -In (3 - J2)] q
177. The angle between dipolemoment vector and radius vector y is given
by
cos y= J. [ (sine cos~) i + (sin e sin~)} + (cos e) k]
= sine sin~
k pcosy p sin e sin ~
Potential, V = r
2 = 41t So r2
and magnitude of electric field
kp ~~~-
E = -3
r
J 1 + 3 cos' y = 4 P
1tEOT
3 ~ 1+3sin2esin'~
.

179. Consider a point on the perpendicular bisector of the rod at a dis-


tance r from the middle of the rod as sliown in the figure. Th find
electric field at this point, consider two points on the rod at a dis-
tance x from the middle. Then

dE = 2 [(_1
41tEo
)( 'Adx
r2 + x2
JJ cos e
dx
E= 2!:-
. 27[80 l
'/2
(r2
dx
+ x2)3/'
.
x

.,
u
=
47[80 ~ ~t +r2 . dx 1
Now, consider an element on the second rod at a distance r of the
length dr. Then
dF =E (),dr)

• 2 I u+ l dr a
F =
! R' +~
_1_,-
r
. 47[80 dr
r I'
4
152 PRACTICEPROBLEMS

=> F=-ln
),2

2rr80
-=~J
[(a+l)(l+~

a(l+~
l2 +4a
2

l2 +4(a+l)2)
)l

181. Consider an elementary ring at a distance x from the vertex along


th& sloping length, as shown in the figure. Then

dV __ 1_ [rr(ls~rx)l.2rr(XSinrx)dx]
4rr80 x
I dx
Q x
V = J dx
2rr8ul2 0

=> V=~.
2rr8ul

183. If a sphere is uniformly charged through out its volume with a


total charge Q, then

E=

=>
185. Suppose the charges on all the surfaces are as shown in the figure.

Since, potential of the innermost sphere and outermost shell will


be same, therefore,

_1_ .(!!J.. _ .!!.L _' k +!f2 +q3 )


4""0 ,R 2R' 3R 4R
PRACTICE PROBLEMS ' , 153

1
= 41t£0
(!lL_!lL_
4R 4R
'12 + '12
4R
+'13)
4R
.,,(i)
6q, = '12
Since, potential ofthe grounded sphere will be zero, therefore

_1_ (!lL _!lL _ '12 + '12 + 'I") = 0


41t60 2R 2R 3R 4R
3'1, = '12 ••• (ii)

Since, the total charge of innermost sphere and outermost shell


will remain same therefore
'I, + '12 + 'I, = 90 "c ... (iii)
From equations (i), (ii) and (iii), we get
'13 = 20 I'c
187. Suppose charge on the sphere of radius a is + 'I and the charge on
the sphere of radius b is-q.
Then potential difference between the spheres wi!! be
V=(_'I 1_(_'1
41tcoa 41tcod)
-_'I __
41tl:od 41t0ob
'I )

=_'1 (~+.!_~)
41tl:o a b d

Now, C=!L
V
C _ 41too

-(~+~-~)
189. Consider the coordinate axes as
a/2 y a/2
shown in the figure.
Equation ofthe parabola is ""
x' = ay ,,/iJ
Now, cousider an elementary
capacitor at a <!istance:; from the
axis ofparabola as shown. Then th~
capacitance of this elementary
capacitor will be '
a

.
,-
__ ..
154 I PRACTICE PROBLEMS

dC = EO (adx)

-"--
4K +(1-~)
K - v

Now using equation (i), we have

C= XT EO (adx)

x == -a/2
-"-- +
4K
(1_~)K X
2

"/2
J ax 2

o 2 ( a )
x +2,jK-l

=>

191. Using the symmetry of the network, the charges can be distributed
as shown in the figure.

+
q,

+ -
q,

Now applying voltage law in the loops(j) ,<2> and @ , we get


q-q, + q- ql -qa (q, -q2) ql = 0 "J
C C .C C ...,'

qa + qa -q2 (q-:2qa) (q-ql -qa) _ 0


C C C C - .. Jii)

q2 + q2 (q, -:q2) (qa- q2) _ 0 ""j


C C ,C C - ... ,n,.
Suppose the equivalent capacitance is C . Then
••
PRACTICE PROBLEMS 155

--.!!.- = q - q, + q. + q2 + q2 + 'lL ...(iv)


C~ C C C C C
From equations (i), (ii), (iii) and (iv), we get

C,. = (:1)C
Now,it is given C = 121)IF.
C•• = 69 )IF.

193. (a) The charges on the capacitors in initial and final conditions
can be calculated as shown in the figure.
l4o"e 140"e
+ 1- +I- 2;O"~~ 1,OS,,~
2JiF lJiF 2"F l"F

360"" 36O"" ~15,,~ ~20,,~


+ 1- +1-
3JiF 4"F 3J1F 4JiF

(lilitial) (Final)
210 V 210 V

Comparing the inItial lind final conditions, the charges flowing


through differeilt branches can be obtained as shown in the figure.

; L"I,'
25"e
, 70
~c
rs
J£ los"e.3S"c
6O!!C
2S"C

Therefore, the charge flowing through thesw'itch is 105llC.


(b) Increase in energy stored in capacitor
A Uc = (Filial energy stored) - (Initial energy stored)

= [j(i)<210)2 -M:~)(210)2] pj

:0 2.625 mJ
Extra energy supplied by battery
Au,. = 25 " 2io 1'.1 :0 5.25 inJ
lleai toss, All '" lilts - Auc
= (5.25-2.625) mJ = 2.625 mJ.
156 ' • PRACTICE PROBLEMS

195. From conservation of energy

...(i)

Here _ (ecbl)V
q - d ...(ii)

From equations (i) and (ii)

____12 e IV2
+_0 _ l
x=
4 (K _ 1)2 pgd2 2(K -1)

197. Consider a' coaxial cylindrical shell of radius x and thickness dx.
All slich type of cylindrical shells will be in parallel combination.
Conductivity at the distance x from the axis

cr=

Therefore, conductance of the cylindrical shell will be


cr (21tx dx)
dG = l

r
j%=a (cr2-cr,) 121tx-,-
xJ-l-=
=> G = %=0 l
crl + a.

1ta2 (crl + 2cr2)


=> G = 31
Now,'resistance of the conductor t

, 1
R=G

_ 31
=> R - 2
1ta (crl + 2cr2)

199. (a)

=~~:,;=oo_n~:nr-
Suppose the equivalent resistance between points A and B is x.
PRACTICE PROBLEMS ' 157

=> nx2 - (3n - 1) Rx - 2R2 = 0

=> x = ~ [(3n -1) + ~9n2 + 2n + 1]

(b) Suppose the current through the section CD is I, and the


current through the battery is 1. Then
Ix = E
and 2IR + IIR = E
From these equations, we get

E [~9n2+2n+1-n-1]
II = R ~ 9n2 + 2n + 1 + 3n-1

tttr
_~.

201. Suppose an external resistance x is


D
on

connected between terminals A and B.


From Kirchoff's Current Law
II + 12 + ... + 1m =I ...(i)
Using Kirchoff's Voltage Law
n(n+1) ]
II [ 2 r + Ix = nE B
A

n(n+1) l
12 [ 2 rJ+Ix=2nE ...{ii)

n(n+1) ]
1m [ 2 r + Ix = mnE
From, these equations

This is of the form I=~


x+R
158 ,
PRACTICEPROBLEMS

n (m+ 1)
Therefore V = 2 E

n (n + 1)
and R = 2m r

203. Suppose charge q has flown from capacitor up to time 't' and at this
instant the currents are distributed as shown in the figure.
i+iI ilO~

IOV ~T~ 5a
r
i
I

20V
0t. 11 q
2~F

5a
Now, applying Voltage Law in loopQ) and@, we get (currents are
in amp. and charge is in ~C)

5 i, + 5 (i + i,l = 10 ...(i)

and .9: + 10i - 5i, = 0 ... (ii)


2
dq .
Also 1 ='- ...(iii)
dt
From these equations, we get
dq
(25) dt + q = 10

q = 10 (1 - e-'i') ~c where ~"25 ~sec

. dq
Now, 1= dt
2
=-e -ti' Am' p.
5

. i
From equation (i), 1 = 1 --
I 2
The current through the branch containing switch, will be
. . 1 i
I. --'Z+L- 1 -
+-2

I = (1 + ~e"i' ) Amp. where ~" 25 ~sec


PRACTICEPROBLEMS

205. The equivalent circuit diagram is shown in the


figure. Suppose the charge on the capacitor at
time t is q and the current through battery is I,.
Applying Voltage Law iu loops Q) and @ ,we
lE (ill,
@
~l +
T
Ie
1,-1
159

~t ~~~
q E R I,
C - (I, - I) r = 0 ..Ji)

and I,R + (I, - 1) r = E ... (ii)

I = dq ,
Also dt ...,iii)
From these equations
rRC) dq CEr
( r+R di +q = r+R

II
q = (~~;) (l-e- ')

CRt
where 't==r+R'

Now

=>
From equation (ii)
E+Ir
1---
,- r+R

I, = (E) (1
r +R . + Re
r -11<)
207. Consider a current element at an angle 8 v,ith the radius OP' 8.S
shown in the figure.
From geometry,

8
r =2Rcos-
2

e
and $=900-"2 p

Now, the magnetic field due ro Uris current element at P is


160 PRACTIC~PROBLEMS

dB J::Q.. I(RdO)sinej>
= 4" r

209. The magnetic field B, due to one side of square will be inclined at
angle (90 - 0) with the axis shown in the figure.
.. Net magnetic field will be

B =.4 B, sinO
I
B = 4 (flO2" .;smex )
sinO

From geometry r = J +/:


x
2

. 1/2
sm ex = Jx 2
+ I;
and sin 0 = 1/2

~ x2 + I:
floIl2
Therefore
B = 2"(X2+~)H
211. Consider an elementary ring of radius r at a distance x from the
PRACTICE PROBLEMS 161

vertex along the sloping length. From geometry,


b-a
sina =-Z-,r=xsina a r h
o
The magnetic field at 0 due to this ring is
!lolr" (ndx)
dB = 3
2x'

bcosec 0; !lJ(xsin«)" (Tdx)


B = J
acosee a. 2x3

!loNI sin" « b
B= Z In;;:
B = !loNI(b - a) In f!-
I" a

213. The current enclosed within a loop of radius r,


1= J j(2 x)dx1C

,i\. .. From Ampere's Law ITtI + ~.itOj"l - ':t

fil.dl = !lol

=>

J.l brct.+ I
=> B=~o--
«+2
215. Consider an element on the circular
loop at an angle 8 with the
horizontal. From the symmetry of
the figure, only the horizontal
component will contribute in net
force. Therefore
162 I PRACTICE PROBLEMS

dF = 2 (dFt) cos 0= 2 (2" (b~O~lcos a») (12 ada) cos a


F = l-'ol112a J: .cos a dO
" (b-acosa)

=} F =1-'.1112 (b2b 2 -1) _a

217. Consider lin element on the semi-circle at an angle a with the line PQ.
The torque due to Iilagnetie forces about line PQ.
't = f: [BI(adO)sinOJasinO
2
= ~~ " IlIa2". -
The centre of riiass is at Ii distlince (2:) from line PQ
For eqirilibrium

nB1a. 2.- '" (itig)


-~ '- (20).
-. PQ]$l. - B .~
2" .
::::::) Iii
,,2 itB!
= 4g
219. Flit the tlingeritilil ditection
rndv
"'.:. i1 (ihg + qE) '/l-;"; ,,'-'''''JCHflA :o-:-! ... (i)
dt
Fot the radial direction
intJ2
r
1iW
f '" qS
iIt ;" mdii
qB

dr = - - Ii (mg + qE) iJi [using equation (i)}


qB .

;
1.o tit =
-I
P ( tiB
~g ~~~ J" di
J rO .

<tqIl
t'" l-'(mg+qE)
PRACTICE PROBLEMS 163

221. Select the axes as shown in the figure.


-. -' V
Then B =Bk,and E =Ej.Also, E="d'
Suppose the velocity at any instant of time t is
y
u =(u)i+(u»).
Now F =qE+qiixB •o • 000000
00000
o 0 000000
F =(qE - qu.B) ) + (qBu) i x
dux
Therefore m -- =qBu ...(i)
dt y

duy
and. m - =qE-quB ...(ii)
dt •
From equation (ii)
d2u y
[using equation (i)]
dt2

.. u;= (:) sin (~)t. .: att=O,


y
duy
u =0 --=-
'dt
qE
m

Y == 1.o,E-sm-t
B
. qB
m
=
Em (
--2
qB
. qBt)
1-s1n--
m

2Em 2Vm
Therefore . _.,. y_ = qB2 = qB2d

If the particle reaches to the other plate, then


y_~d

qB2d2
V ~ 2m

223. Consider a current element on the circular


ring at an angle 0 with the positive x-axis
as shown in the figure.
The current induced in the ring,

i= ,,;2 (d~z)
l
. = --"a2y ...(i)
R
dF = (iadO) (axcos 0) + (iadO) (Jlysin 0)
164 PRACTICE PROBLEMS

rx/2 2
F = 4.'Lan 2JO cos ada + 4'R
tpa
2 rx/2
Jo

SIn
2ada
F = rria2 (a + 13)
2 4
F = rr a (a+f3)Y [using equation (i)]
R
225. Suppose the base radius of the cone is R and slant height is L.
. R
Then sm a = L'
Consider an elementary ring of radius r at distance I from the vertex
.along the slant height.
The induced electric field on the surface of the ring E is given by

,E.dl = I ~: I
2 dB
E(2rrr) = rrr -
dt
1 dB
E = -r-
2 dt
y
1
E = -rb (':B=bt)
2. x

Now, torque acting on the ring due to induced electric field,

d~ = [(r:)(2rrrdf'rr~L)] r
bQ
T=RL
f rdl=RL2bQ (sin3a)Jol3dl
3 . r L
(.: r = I sin a)

bQR2
T=-- ,. (.: Lsina= R)
4
2
Now ~mR2)dro = bQR
( 10 dt 4
dro = 5Qb
dt 6m
ro = 5Qbt
(lm
5QB
0> = - ,8m'
(Taking into account the direction of w)
PRACTICE PROBLEMS • 165

227. Suppose at any instant the conductor is moving down ,vith velocity
l'. Consider an element on the conductor at a distance x from the
current carrying wire.
The emf developed across this element,
R

<IV = (~) l'dx


2nx

V -
_ Jb
a
/loI v dx
2 nx
x

--
dx
J
a
b /loll' b
=--In-
2n a
The current through conductor will be
. V
l =-
R
/loI v b
= 2nR In -;;- ...(i)

Now the force acting on the conductor

= i (/lOI)
2n
In .£.
a
2
/loI b v
F = ( 2" In-;;-) R
[substituting i from equation (i)]
The equation of motion is
2
.' dv /loI b v
m- =mg- -In- -
• '. dt ( 2n a) R
2
dv 1 /loI b
.dt+ al'-=g, wherea= -- ( --In-
mR 2n a)

" -- dl' -
I
dt
5.og-al'- fo
166 I ' PRACTICE PROBLEMS

229. Suppose at time t the angular velocity ofthe rod is m,


1
" Induced emf in the rod is V = 2Bl2m ,
Therefore, torque acting on the rod

't - _ [(BI m) --
2
I _ B1m
Bl ] - - --
2 4

2R 2 4R

The equation of motion is


2
mI ] dw
( =---
3 dt 4R

dm 3B212
dt = -bm, whereb= --'
4mR'
(Q = COo e-bt

de
-" - ="00" e-bt
dt 0'

e= (4mRwo] (1- -l,t)


3B212 e

231. (Work done by gravity) = (Increase in KE) + (heat dissipated)


~ (mg) v sinedt =(mv)dv + (i'Rdt) '" (i)
dx
dt = v cos e

dy
= - v sin e
dt

Now, i=
2
(1ta )
~
I di I =
dB 1ta2Boa vcose
R
PRACTICE PROBLEMS • , 167

Substituting this value of i in equation (i), we get


dv b .
dt + v =gsme

,,2 a4B5a2 cos2 e


where, b = mR
mgRsec e tane
v = ,,2a4B~a2

233. The induced emfin the inner coil is


dI
V =M dt =Mloae-
Applying Voltage Law for the inner coil
di .
L dt +lR =MIoe-
Integrating, we get

. ',. MIoa (-o.t .


-RtIL)
l = (R-aL) e. -e

235. Applying Voltage Law in the first and second circuits, we get
L dio +!L +Mdi, =0 '")
I dt C dt ...Il

di.l di,
and L, dt + M de =0 ...(ii)
From equations (i) and (ii), we get

di1+!L=0 ...(iii)
dt C

.. dq
Now l =-
I dt
Therefore, equation (iii) can be written as
d2q .
-2-"+ ",'q.= 0 .
dt
L2,
where", = C(L1L2 - M2)
168 ,
PRACTICEPROBLEMS

237. (a) The equation for the time dependence of the current is

j~Idl
Now I
average
=--=
J:'dt

d1
(b)
I' = =j~_I_2
_d_1 = _j~--,(~I=-~I-")_2c- _ = I~
••"... j~dl T 3

Now I •.•••= ~ I~verage = Ja


239. I = 11+ I,

= (10,/2) SinooI + (20,/2) sin (00 I +60e)

= (10,/2) sinooI + (20,/2) (sin 001 cos 60° + cos 001 sin 60°)

= (10,/2 + 20,/2 cos 60°) sin 001 + (20,/2 sin 60°) cOSOlI

= (20,/2) sin 00 I + (1O~ cos 00 I


= I. sin (00 I + 8)

where, I. =~ (20.[i)2 + (1OJ6t = 37.4

10./6 ,J3
and tane=~=- => 8=41°
20,,2 2 .
1= [(37.4) sin (00 I + 41°)]Amp.
241. Given: I. = 14.4 Amp, V. = 170 Volt
1
Now P .vef~e = -VT
2 trO cose
1 .
= "2 x 170 x 14.4 x cos 30°
= 1040 Watt.
PRACTICE PROBLEMS 169

243. For resistor


60
1= 120 = 0.5A
For inductor (OloL)I = 40
=> L = 2 x lO-'H = 0.2 mH

Now 1
Ol=--
o JLC

C = _1_ => C =
0l6L .
(-.!..)
32
f'F

245. The impedance ofthe circuit is given by

1
Z

y
.. Current, 1= Z =y ~+(~-~J
R2 Xc XL

=240

=50A
(~r +G-H

y2.
247. Pm",
R

y2R
and P=
2
R +(OlL- io~r
Now P = Pmax
2

y2R y2
=>
2
R + ( OlL- Ol~ r = .-
2R
170 PRACTICE PROBLEMS

1
ffiL---+R
ffiC --

ffi=( ~C+::ZJ~~

249.
!_ 1 __ ,_1_ = z R z + j (~ zX
L
Z)
Z - R+}XL }Xc R +XL Xc R +XL
For the resonance, the imaginary part should be absent,
1 XL
Xc RZ + xl = 0 1'"

251. The detector D detects no current. Therefore the bridge is balanced.

(R} + j:C,) (jffiCZ) = (~. ;~ffiL) ,


Equating real and imaginary parts, we get
L
="Rz
PRACTICE PROBLEMS 171

.. L = C,R2R, =C,R,R,
253. If the distance of the virtual image from the pole isy, then
1 1 1
=
y x f
A
-Y = f-x

Now
: =(f~J(:J
v, = (f~J)2g(f-xJ
For v, to be maximum
dVI
=0
dx

x=L 3

255. From the geometry of the figure

tl
x = --sin(i-e) t2
+ --", . ('
sm !-~
"')
...(i)
cos e cos ~

Now . e =--
sm sini ...(ii)
III
172 PRACTICE PROBLEMS

and . '"
SIn,!, sin
=-- i ...(iii)
fl2
Subs.tituting the values ofsin 8 and sin <I>
in equation (i), we get

25'7. The person must move along the path of a "light ray" for the short-
est time. --9 12m 9m":"-
A

"d 2.54
Ref ractJve m ex fl = --1.875 = -3
20Am

From the geometry of the figure


2(9tan8)+IZtan<l> = 20.4 ...(i)
From Snell's Law,
4
"3 sin <I>= sine ...(ii)

From equations (i) and (ii), we get

.
sm e =54 an d' Slil<l>-5 3
Now, I, = 9 sec e = 15 m and = 15 m
12" 12 sec <I>
.
Time = .-21]
2.5
1
+ --' 2
1.875
= 5 sec.

259. From Snell's Law

sin r, = ( ~ J sin i ... (i)

sin r2= (fl: J sine ...(ii)

From geomet~y
r, t r2 = A (iii)
8 = i +e- A (iv)
PRACTICE PROBLEMS 173

Now sine = (:.) sinr2 [using equation (ii)]

f'
=> sin e = ;; [sin (A - r j)] [using equation (iii)]

f'
sin e = ;; (sin A cos r j - cos A sin r j)

=>sine= ~~ [(Sin A) (~r 2


-sin i-cosAsini]

[using equation (i)]


Therefore, from equation (iv), we get

B=i-A+Sin-j[~~ {(Si~A) (~r 2


-sin i -coSASini}]

261. From Snell's Law


f' sin (60° -8) = sin i ... (i)
Fot total internal reaction,
f' sin e ;;,1 ...(ii)
Since, total angle of deviation is 120°, hence, from geometry, we
get, i = 30° . ...(iii)

Nqw, fr~m equations (i), (ii) and (iii), we get

~=R
263. From the geometry of the figure
174 'PRACTICE PROBLEMS

.. b ...(i)
Slnz,=-
.R
From Slllill's Law
sini
sin 6 = -~- ...(ii)

'Angle of deviatiun, 0 = 2 (i - 6)
. 0
=> sm - = sin (i - 6)
2
= (sin i cos 6 - cos -; sin 6)

sini
= (~ ~2 -sin2i -COSi)

[using equation (ii)]

[using equation (i)]

=>

265. For the refraction at the spherical surface,


~2 - ~1
- ...(i)
R

I,
PRACTICEPROBLEMS 175

For the first refraction


3
#, = 1, #, = "2' u = "', R = R
v = 3R [using equation (i)]

The image is formed aU,


R
For the reflection at polished surface, u=R,f=--.
2

1 1 1
-+- -
V U = f
R
:::> v=--
3
The image due to reflection is formed at 12,
Now, for the refraction at the spherical surface
3 5
fl, = "2' fl, = 1, u = - '3 R , R = - R

5
v=--R [using equation (i))
2

Therefore, the final image is formed at I at a distance ~ behind

the polished surface.


267. Suppose the distance of object from the lens isx and the distance of
corresponding image from the lens is y.

h,>
I x

1 1 1
Then =
v u f
1 1 1
- -+- =
Y x f

.y=A
f-x
176 PRACTIC!! PROBLEMS

Now
!!L y f
ho-x-f-x
The angular magnification is

m=! = hr!(y+(l-x»)
holl
= (!!L) (
ho
l
l+(y-x)
J
a

fl
m=
2
x +If-lx
Form m to be maximum

.!!...- (x2+lf-lx) =0
dx

x:=: I.
2

1
Therefore
(m)m" =
1+ -
(I,),
4f

269. The lens C will focus the parallel rays on its focus. Therefore, for
a
the composite lens there will be virtual object lit a distance of 1 m
on the right side. '
1 1 1
From lens formula ;;--;;=7
1 1
__ 1
..L..
we have
V i-I
v= 0.5 m
, u 0.5
and magnification, m = ~ = -' -' = 0.5
v 1
Hence, there will be two images at a distance of 0.5 m on the right
side of the composite leos. The image due to part A will be below
the axis at a distance of(0.5) (0.5 em) = 0.25 cm from the axis. The
image due to part B will be above the axis at a distance of 0.25cm
from the axis. " ,
:. Distance between the two images is (0.25 + 0.25) em = 0.5 em.
271. Consider a point P on the positive X,axis at a distance x ftom the
origin, ',' " "
PRACTICE PROBLEMS 177
y
For minima 8,
d
x p x

_x=(n+~)A
(4-(n+~nA
X = ""-~(2-n-+
1) , where n = 1,0
For n = 1, X = ~A
12

and for n = 0, X = _.15 A


4

2'73. For maxima dsine =:tnA


. e
SIn :;:::t-
nA
d

sin 9 :;::0, :t: .!., :t 1


2
e = 0, 30°, 90°,150°, 180°,210°,270° 330°

275. If the intensity of light through 8, and 8, is I,. Then,


The intensity through 8, will be I, = 41,.
The intensity through 8. will be
1, = 41, cos' 1-2

= 41, cos' ~ [2An( ~~) ~]

n .
= 41, cos' "4 = 21,

Now
178 PRACTICE PROBLEMS

=
~ +1
vI;
~-1
= (.[2
.[2
+1)
-1 .
= 34
vI;
277. yd = (fl, - 1) t, - (fl, - 1) t, + d sin 8
D

~ y = ~ [(fl,- 1) t, - (fl, - 1) t, + d sin 8]


Substituting the given values, we get
y = 10.5 cm
279. Angle of deviation due to prism
8 = (fl- 1) A

= (1.5 - 1) (~~) x 1:0 radian

= Radian
_11_
1080
Distance between two coherent sources,
d = 2 8 (25cm)
11
= 2 x -- x 25 = 0 145 cm
1080. .
Distance between coherent sources and screen
D = (25+ 100) cm = 125 em

Now, fringe width p= Ad = 0.52 mm

281. If the angle of refraction is r, then


'. \ sin i
Slur = --
sin 60° = 0.65
= ---
Jl 1.33
~ cosr=0.76
Now, for the maxil11ain the reflected rays

2flt cos r =

t = (n + i) -2-,,-~-o-8-r
PRACTICE PROBLEMS 179

A
tmin ::=
4flcosr
6000
tmin::= -- A
4 x 1.33 x 0.76
~0.15 flm

2I
2 (51 x 10-6
x 10-3)
x

283. Pressure : - :
C 3 x 108

: 3.33 X 10-5 N/m'

285. At ~ 3000 A,
1.2 :6000 A
hc
E ~ - : 4.14eV,
1 1.1
he
E ~ - ~2.07eV
, . l- 2

1 . ...(i)
Now (4.14) eV - W : - mv~
2
.1' . ~. ...(ii)
(2.07) eV - W : - mV2
2
...(iii)
and ~ :3
V2
From equations (i), (ii) and (iii), we get
W : 1.81 .V.

287. I-m;n = 4000 A.


he
E Dlall ~,. Amin
= 3.1 eY.
KE of photoelectrons,
K: 3.1-2.39
= 0.7 eV
180 ,PRACTICE PROBLEMS

Now, mv <d
qB

and ..!.mv2 =K
2

B>~2mK
qB
B.mm = 2.86 X 10-6 T

289. Here, A\ = 1085 A,

A2 = 304 A

E, = hC
Al
= 11.5 eV,

E,= hC =40.8eV
A2
E, + E, = 52.3eV
Kinetic energy of electrons after collision
E = 100-52.3
= 47.7 eY.
291. Since, from the nth energy level six photons are emitted, therefore

~(n-1) =6
2
~ n =4
Itis given
"
E. - E, = 10.2

~ ( - ~:;~ Z2 J - (- ~:j~
J Z2 = 10.2

~ 2.55 z' = 10.2


~ Z =2'
PRACTICEPROBLEMS 181

293.

Now ...(i)

nh ...(ii)
mvr =-
2"
1 e' ...(iii)
Energy, E = -mv' - 3
2 12,,60 r
From equations (i), (ii) and (iii), we get
6 6
~ nh
E--~--
4 4 3
- 24n: e m
295. From the energy level diagram

hC hC
=--+--
A.L~ A.K~

1 1 1
=> -- =-----
A.w A.K~ A.K~

=> A.w = 140.7 pm

297. Number of protons incident per second

10-4
= ----19 = 6.25 x 1014
1.6 x 10-
Number of ('Be) atoms produced per second
6.25 x 1014
= 1000
= 6.25 X 10".
182 t, PRACTICE PROBLEMS

Now,

1.8 x lOB
=
625 x 1011
= 2.885 x lOll/sec
0.693
Halflife, ~ = -)..-

0.693
= 2.885 x 10-4 sec
= 2.4 x 10' sec. = 40 min.
299. Number of fissions per second

106
= (200 X 106) x (1.6 xJO-19) .

= 3.125 X 1016
Mass ofU-235 required in.ime year

(3.125 x 10"16) ]
= [(
6.023 x 10
23)
x 235 x (365 x 24 x 60 x60)

= 382.4gm.

••
PART - II
Problems from
Previous Years IIT-JEE
(From 1972 onwards)
PRACTICE PROBLEMS " 183

(PRACTICE PROBLEMS)
IPART-III

On a frictionless horizontal surface,


assumed to be the x-y plane, a small
tr"lley A is moving along a
straight line parallel to the y-axis ..~

of (./3 - 1) mls. At a particular


instant, when the line OA makes an
angle of 45° with the x-axis, a ball is
thrown along the surface from the
origin O. Its velocity makes an
0" 45"
x

angle 'P with the x-axis and it hits


the trolley, .
(a) The motion of the ball is observed from the frame of the
trolley. Calculate the angle 6 made by the velocity vector of
the ball with the x-axis in this frame.
(b) Find the speed of the ball with respect to the surfa~, if 'P =
49
3 ' (llT2002)

~ ••••••............•..........••.•.•.•.••••.•.•....••.•..
:.n ..H.t.{ .
A mass A is released from the top of a frictionless inclined plane
18 m long and reaches the bottom 3 seconds later. At the instant
when A is released, a second mass B is projected upward along the
a
plane from the bottom with' iiettsin initial velocity. The mass B
travels a distance up the plantl';'stopsand returns to the bottom so
that it arrives simnltaneously with A. The two masses do not
collide with each other at any stage. Find the acceleration and
initial vel'lCityof n.
(lIT 1978)
184 PRACTICE PROBLEMS
.~. - -,' ... -,
.
..... . ". ' .
~
..

A particle starts from rest at time t = 0


and undergoes acceleration a as .(mfs')
shown in the figure. 3
(i) Draw a neat sketch showing the
t(8Oo)
velocity of particle as a function of
time during the interval 0 to 4
o 2 4

seconds, indicating each second on


the abscissa. -3
(ii) Draw a neat sketch showing displacement of the particle as a
function of time during the same interval. (liT 1977)
ETI_" __ ~'_' '_'~_' _'~._'.'_'_~ __ ' ._.._.,.~~-
A car accelerates from rest at a constant rate a. for some time
after which it decelerates at a constant rate Il to come to rest. If
the total time lapse is t seconds, evaluate :
(i) maximum velocity reached, and
(ii) the total distance travelled. (liT 1978)

W~
..~ ~-'"'--~_-'--
A metal ball is allowed to fall freely on a perfectly elastic plane
from a height of 3 metres. At t = 0 the speed of the ball is zero.
Diagramatically represent the variation of velocity with time
specifying the proper units on the X and Y.axes. (liT 1975)
0_. ~~~~ __ ._
..
1\vo cars are moving in the same direction with the same speed
(= 30 kmlh). They are separated by a distance 5 km. What is the
speed of a car moving ;"nthe opposite direction if it met these two
cars at an interval of 4 minutes? (liT 1975)
[D ~
A rocket is fired vertically from the ground with a resultant vertical
acceleration of 10 mls". The fuel is finished in 1 minute and it
continues to move up. What is the maximum height reached?
(g = 10 m/s")' (liT 1975)
ITI ~
A car covers the first half of the distance between two places at
-a speed of 40kmlh and the second half at 60 kmlh. What is the
ave,'age speed of the car? (liT 1974),.'
PRACTICE PROBLEMS 185

[!J fAt ~.
.::~n:~t~7ltf~~~
1~~~-
.1~ .,.~::}S;j~,~-~~~~:r-::_I
:rt.-~~'1
A block slides down a smooth inclined plane when releasee:! from
the top, while another falls freely from the same point. Which one
of them will strike the ground .
(i) earlier,
(ii) with greater velocity. (lIT 1974)
r:::l r.:~7I:~. l(lI:/t~ ~'li.~~~-r ~IM>~t~--~~"~l}"ifi-m1
~.'_
..
~ 10 Co,.t. .•. ~':",_ ::;'.(•..•.... "',.' _~••' .••''':.':~£_ "-' .":';." ..,.... ,'" ."-~:.j~~ •...- ,,' ""-. ~J..,__
..~J
From the top of a building, a ball is dropped while anotheris thrown
horizontally at the same time. Which ball will strike the ground
first? (lIT 1974)

x •
Figure gives the displacement of a • •
particle along the x-axis as a
function of time. Find the direction
of the velocity and acceleration of
the particle between the following
points.
(i) Between 0 and A
(ii) Between A and B
(iii) Between C and D (lIT 1973)

The acceleration due to gravity on a planet is 196 cmls2.1f it is safe


to jump from a height of 2 metres on the earth, what will be the
corresponding safe height on the planet? .
(georth = 980 mls') (lIT 1972)
113:1iiji~i:
_~~!i~~e7:~-lilm~~:
4~\J~~&F~d,.~)~.m:;"';;Y.otl>{j
A block of ice starts sliding down from the top of the inclined roof
of a house (angle of inclination of roof = 30. with the horizontal)
along a line of maximum slope. The highest and lowest points of
the roof are at heights of 8.1 metre and 5.6 metre respectively from
the ground. At what horizontal distance from the starting point
will the block hit the ground? Neglect friction. (g = 9.8 mls2).
(lIT Iln2)
PROJECTILE

[ill ~..._ ~"'


14- ,_. ..,_..~ ..
....~~.~"""
.,_•.•..••-~i(:'-<-}' .. -":-,
.tok";;1;4
•••• '.r.!'~.~~';,~,!
.~ ...
"'.,$.. ,.y-, ••.. ,;j"' •• ..,.~t.
- .rf'..,
-'10 -
•..""'..~...•..•....•.•..•.
.. ~,...:.~ '----.~
__~_._ ••"l: ..-•••... ",_ .•.•. }." ~:....
.•, w:li;t\<"h':'~:b~'i:"~:"\.~";:i-'
:!.-."'t~"r,<.-t:
. .
, An object A is kept fixed at the point x 3 and y 1.25 m on a = =
plank P raised above the ground. At time t = 0, the plank
186 , PRACTICE PROBLEMS

start moving alongthe +x direction with an acceleration 1.5 mls".


At the same instant a stone is projected from the origin with a
velocity u as shown. A
y
stationary person on the
ground observes the stone l.25m
hitting the object during its
downward motion at an angle
of 45° to the horizontal. All
u
the motions are in the x-y
3.0m
plane. Find u .and the time
after which the stone hits the
object. Take g = 10 mls". (lIT 2000)

~---------------~--
A large heavy box is sliding
wliliout friction down a sffiooth
plane ofinclination e. From a point
P on the bottom of the box, a
particle is projected inside the box,
the initial speed of the particle
with respect to the box is u, and the direction of projection makes
an anglea. with the bottom as shown in the figure.
(a) Find the distance along the bottom of the box between the
point of projection P and the point Q where the particle
lands. (Assume that the particle does not hit any other surface
of the box. Neglect air resistance.)
(b) If the horizontal displacement of the particle as seen by an
observer on the ground is zero, find the speed ofthe box with
respect to the ground at the instant when the particle was
projected. (lIT 1998)
161_.
1 ----------~----
Twoguns, situated on the top of a hill ofheight 10 m, fire one shot
each with the same speed sJ3 ms.l at some interval of time. One
gun fires horizontally and other fires upwards at an angle of 60°
with the horizontal. The shots collide in air at a point P. Find
(i) the tinIe-interval between the firings, and
(ii) the coordinates of the point P.
Take origin of the coordinate system at the foot of the hill right
below the muzzle and traject.ories in x-y plane. (lIT 1996)
PRACTICEPROBLEMS 187
[ill _
A body falling freely from a given height 'H' hits an inclined plane
in its path at a height '/z'. As a result of this impat the direction of
the velocity of the body becomes horizontal. For what value of
(/zIH) the body will take maximum time to reach the ground?
(lIT 1986)
lill _
A gun, kept on a straight horizontal road, is used to hit a car
travelling along the same road away from the gun with a uniform
speed of 72 kmlh. The car is at a distance of 500 metres from the
gun when the gun is fired at an angle 45° with the horizontal.
Find (i) the distance ofthe car from the gun when the shell hits it
(ii) the speed of projection of the shell from the gu n.
(g = 9.8 m/s2) (lIT 1974)

~~-----------------
~-' --------------
A particle of mass 10- 2 kg is moving along the positive x-axis
under the influence ofa force F (x) = - ( K2 ) , where K = 10-2 Nm2•
2x

At time t = 0 it is at x = 1.0 IIIand its velocity is u = o.


(a) Find its velocity when it reaches x = 0.50 m.
(b) Find the time at which it reaches x = 0.25 m (lIT 1998)
lill _
A smooth semicircular wire-track of radius R
is fixed in a vertical plane
One end of a massless spring of natural length
3 R is attached to the lowest point 0 of the P
4
wire-track. A small ring is held stationary at
point P such that the spring makes an angle of
60° with the vertical. The spring constant
K = IIlg/R. Consider the instant when the ring is released, and
(i) draw the free body diagram of the ring,
(ii) determine the tangential acceleration of the ring and the
normal reaction. (lIT 1996)
188 PRACTICEPROBLEMS

Cill _
S '
Two blocks of mass 2.9 kg and 1.9 kg are
suspended from a rigid suport S by two
inextensible wires each of length 1 metre nee
figure. The upper wire has negligible mass and
the lower wire has a uniform mass of 0.2 kg/m.
The whole system of blocks wires and support
have an upward acceleration of 0.2m/s2•
Acceleration due to gravity is 9.8 m/s2.
(i) Find the tension at the mid-point of the lower wire.
(ii) Find the tension at the mid point of the upper wire.
(lIT 1989)

@]------------------
Two masses m and 2m are connected by a
massless string which passes over a light
frictionless pulley as shown in the figure. The
masses are initially held with equal lengths of
strings on either side of the pulley. F;',d the
velocity of the masses at the instant the
lighter mass moves up a distance of 6.54 m.
The string is suddenly cut at that -instant. Ground I
Calculate the time taken by each mass ''''''''''''''''''''''
to reach the ground. (g = 981 cm/s2) (lIT 1977)

~-------------
A horizontal uniform rope of length L, resting on a frictionless
horizontal surface, is pulled at one end by a force F. What is the
tension in the rope at a distance I from the end where the force is
applied? (lIT 1978)

~-------------
Twocubes of masses m, and m be on two2
frictionless slopes of block A which rests
on a horizontal table. The cubes are
connected by a string which passes over a
pulley as shown in the figure. To what
horizontal acceleration f should the whole
system (that is blocks and cubes) be
subjected so that the cubes do not slide
down the planes. What is the tension of
the string in this situation? (liT 1978)
PRACTICE PROBLEMS 189

@]--------------
A spring offorce constant K is cut into three equal parts. What is
force constant of each part? (lIT 1978)
1261 _

A lift is going up. The total mass of the lift and the passengers is
1500 kg. The variation in speed of the lift is as given in the graph.
(a) What will be the v(m/,)
tension in the rope
pulling the lift at t
equal to
(i) 1 sec
(ii) 6 sec
(iii) 11 sec. 0 2 10)2 t (sec)

(b) What is the height to


which the lift takes
the passengers?
(c) What will be the average velocity and the average acceleration
during the course of the entire motion? (g = 9.8 m/s2).
(lIT 1976)
[EJ _
Two balls A and B of masses 100 gm and 250 gm respectively and
connected by a stretched string of negligible mass, and placed on a
smooth table. When the balls are released simultaneously, the
initial acceleration of ball B is 10 cm/s2 westward. What is the
magnitude and direction ofthe initial acceleration of the ball A ?
(lIT 1975)

~,-,'---------------
A mass A (500 gm) is placed on a smooth
table with a string attached to it. The A
string goes over a frictionless pulley and
is connected to another mass B r-- 200cm

(200 gm).''At ,t = 0, the mass A is at a


B
distance J1f 200 cm from the end and
moving with a speed of
50 cm/sec towards left (see figure). What
will be its position and its speed at t = 1
sec9.nd ? (g = 980 cm/s2) (lIT 1975)
190 PRACTICE PROBLEMS

..
A spring weighing machine kept inside a stationary elevator reads
50 kg when a man stands on it. What would happen to the scale
reading if the elevator is moving upward with
(i) constant velocity
(iij constant acceleration? (lIT 1972)

1iiriimI:»...... . »»)Unl
I*~I» ....
Two identical blocks A and Bare
placed on a rough inclined plane
of inclination 45'. The coefficient
of friction between block A and
incline is 0.2 and that of between
B and incline is 0.3. The initial
separation between the two blocks
is .J2m. The two blocks are released
from rest, then find
(aj the time after which front
faces of both blocks come in
same line and
(bj the distance moved by each block for attaining above position.
(lIT 2004)

In the figure masses ml'


m2 and Mare 20 kg, 5 kg P,
and 50 kg respectively. The
coefficient of friction
between M and ground is F
M
zero. The coefficient of m,
P,
friction between m! and M
and that between m2 and
ground is 0.3. The pulleys and the string are massless. The string
is perfectly horizontal between P! and m! and also between P 2 and
m2. The string is perfectly vertical between P! and P 2' An external
horizontal force F is applies to the mass M. Take g = 10 mls2•
PRACTICE PROBLEMS 191

(a) Draw a free body diagram for mass M, clearly showing the forces.
(b) Let the magnitude of the force of friction between mj and M
be f, and that between m2 and ground be '2.
For a particular F it is found that
'j
= 2'2.
Find f, and f2• Write down equations of motion of all the masses.
Find F, tension in the string and accelerations of the masses.
(lIT 2000)

1321 ~ _
Block A of mass m
block B of mass
2 m are placed on
a fIxed triangular
wedge by means
of a massless, in
extensible string
and a frictionless pulley as shown in Figure. The wedge is inclined
at 45° to the horizontal on both sides. The coefficient of friction
between block A and the wedge is 2/3 and that between block B
and the wedge is 1/3. If the system A and B is released from rest,
fInd
(i) the acceleration of A,
(ii) tension in the string,
(iii) the magnitude and direction of friction acting on A,
(lIT 1997, May)
00 _
A particle of mass m rests on a horizontal floor with which it has a
coefficient of static friction fl. It is desired to make the body move
by applying the minimum possible force F. Find the magnitude of
F and the direction in which it has to be applied.
(lIT 1987)

Two blocks A and B are connected to each other by a string and a


spring; the string passes over a frictionless pulley as shown in the
fIgure. Block B slides over the horizontal top surface of a stationary
block C and the block A slides along the vertical side of C, both
192 . PRACTICE PROBLEMS

with the same unifonn speed. The


coefficient of friction between the B

surfaces of blocks is 0.2. Force


constant of the spring is 1960
c
Newtonslm. Ifmass of block A is 2 kg,
calculate the mass of block B and the A I
energy stored in the spring.
(lIT 1982)
Ilm=
..•.••..,.,.=
=.).,= .."".,=., = =..=======~~
Masses M" ~ and M:.are
connected by strings of
negligible mass which pass
over massless and
frictionless pulleys P, and
P 2 as shown in the figure.
The masses move such
that the portion of the
string between P, and P 2 is
parallel to the inclined plane and the portion of the string between
P2 and M:. is horizontal. The masses M2 and M3 are 4.0 kg each
and the coefficient of kinetic friction between the masses and the
surfaces is 0.25. The inclined plane makes an angle of 37° with the
horizontal. (lIT 1981)
If the mass M, moves downward with a unifonn velocity, find :
(i) the mass of M,
(ii) tension in the horizontal portion of the string.
• 3
(g = 9.8 mls 2 , sm 37° = 5)

In the Figs. (a)


and(b),AC,DGandGF A
D
are fixed inclined
planes, BC = EF = x and
AB = DE = y. A small
block of mass M is
released from the point
A. It slides down
AC and reaches C with B C E (b) F
a speed Ve• The same
PRACTICE PROBLEMS 193

block is released from rest from the point D. It slides down DGF
and reaches the point F with speed VF. The coefficient of kinetic
frictions between the block and both the surfaces AC and DGF are
I'.Calculate Vc and VF• (lIT 1980)
lliJ _
Two blocks cOf\nected by a
massless string slide down an
inclined plane having an
inclination of 37°. The masses of
two blocks are m, = 4 kg and m2 =
2 kg respectively and the
coefficient offriction of m, and m2
with inclined plane are 0.75 and
0.25 respectively. Assuming the
string to be taut, find
(i) the common acceleration of two masses and
2
(ii) the tension in the string (sin 37° = 0.6, g = 9.8 mls )
(lIT 1979)

~---------------
In the diagram shown, the blocks A, B
and C weight 3 kg, 4 kg and 5 kg
respectively. The coefficient of sliding
friction between any two surfaces is
A
0.25.Ais held at rest by a massless rigid B
rod fixed to the wall while B and C are C
connected by a light flexible cord
passing around a frictionless pulley.
Find the force F necessary to drag C along the horizontal surface
to the left at constant speed. Assume that the arrangement shown
in the diagram, B on C and A on B, is maintained all through. (g =
9.8 mls2) (lIT 1978)

~---------------
An aeroplane requires for take off a speed of 80 kmlh, the run on
the ground being 100 metres. The mass of the plane is 10000 kg
and the coefficient offriction between the plane and the ground is
0.2. Assume that the plane accelerates uniformly during the take
off.What is the minimum force required by the engine ofthe plane
for the take off? (g = 9.8 mls2) (lIT 1977)
194 PRACTICE PROBLEMS

1401 ~ _
A block of mass 2 kg slides on an inclined plane which makes an
angle of30° with the horizontal. The coefficient offriction between

the block and the surface is ~.

(i) What force should be applied to the block so that the block
moves down without any acceleration?
(ii) What force should be applied to the block so that it moves up
without any acceleration?
(iii) Calculate the ratio of the powers needed in the. above two
cases, if the block moves with the same speed in both cases.
(g = 9.8 mls') (lIT 1976)

~'-----------------~
lliJ _
Two blocks of mass m, = 10 mg and m, = 5 kg, connected to each
other by a massless inextensible string oflengthO.3 m are placed
along a diameter ofa turn table. The coefficient of friction between
the table and m, is 0.5 while there is no friction between m, and
the table. The table is rotating with an angular velocity of 10 radl
s about a vertical axis passing through its centre O. The masses
are placed along the diameter of the table on either side of the
center 0 such that the mass m, is ata distance of O.124 m from O.
The masses are observed to be at rest with respect to an observer
on the turn table.
(i) Calculate the frictional force on mI.
(ii) What should be the minimum angular speed ofthe turn table
so that the masses will slip from this position?
(iii) How should the masses be placed with the string remaining
taut, so that there is no frictional force acting on the
mass m,? (liT 1997, July)

@]_. -----------~-
. A hemispherical bowl ofradius R = 0.1 m is rotating about its own
axis (which is vertical), with an angular velocity (i). A particle of
mass 10-' kg on the frictionless inner surface of the bowl is also
rotating with the same 01. The particle is at a height h from the
bottom of the bowl.
PRACTICE PROBLEMS 195

(i) Obtain the relation between hand ro.What is the minimum


value of roneeded, in order to have a non-zero value of h ?
(ii) It is desired to measure g (acceleration due to gravity) using
this set-up, by measuring h accurately. Assuming that Rand
ro are known precisely, and that the least count in the
measurement ofh is 10-4 m, what is the minimum possible
error f>gin the measured value ofg ? (lIT 1993)
[ill _
A large mass M and a small mass m hang at
two ends of a string that passes over a smooth
tube as shown in the figure. The mass m moves
around a circular path which lies in a
horizontal plane. The length of string from the
mass m to the top of the tu be is I and e is the
angle this length makes with the
vertical. What should be the frequency of
rotation of mass m, so that the mass M remains
stationary ? (lIT 1978)
lEl _
The driver of a truck travelling with a velocity v suddenly notices
a broad wall in front of him at a distance r. Is it better for him to
apply brakes or to make a circular turn without applying brakes
in order to just avoid crashing into the wall? Why ? (lIT 1977)

@]------------------
A string oflength 1 metre is fixed at one end and carries a mass of
100 gm at the other end. The string makes (.;) revolutions per
second around a vertical axis passing through the fixed end.
Calculate: .
(i) the angle of inclination of the string with the vertical.
(ii) the tension in the string, and
(iii) the linear velocity of the mass.
(g = 9.8 m/s2) (lIT 1976)
!ill --'-- _
A sphere of mass 200 gm is attached to an inextensible string of
length 130 em whose upper end is fixed to the ceiling. The sphere
is made to describe a horizontal circle of radius 50 em.
(i) Calculate the time period of one revolution.
(ii) What is the tension in the string.
(g = 980 cm/s2). (lIT 1974)
196 PRACTICE PROBLEMS

1471~: -_.~------------~
_
A spherical ball of mass m
is kept at the highest point
in the space between two
fixed, concentric spheres A
and B (see figure). The
smaller sphere A has a
radius R and the space
between the two spheres
has a width d. The ball has
a diameter very slightly
less than d. All surfaces are frictionless. The ball is given a gentle
pu sh (towards the right in the figure). The angle made by the radius
vector ofthe ball with the upward vertical is denoted by a (shown
in the figure).
(a) Express the total normal reaction force exerted by the spheres
on the ball as a function of angle a.
(b) LetNA and NB denote the magnitudes ofthe normal reaction
forces on the ball exerted by the spheres Aand B, respective11
Sketch the variations ofNA and NB as functions of cos a in the
range 0 s;a s;11 by drawing two separate graphs in your answer
book, taking cos a on the horizontal axes. (lIT 2002)

@]---------------
A particle is suspended vertically from a
point 0 by an inextensible massless string :A
,
of length L. A vertical line AB is at a
L
distance 8 from 0 as shown. The object is "T.Ot
L',.
~ U8~ i
,,
given a horizontal velocityu. At somepoint, :. ,,
its motion ceases to be circular and ... f.. u .B
eventually the object passes through
the line AB.At the instant ofcrossing AB,its velocity is horizontal.
Find u. (lIT 1999)

@]---------------
A string with one end fixed on a rigid wall, passing over a fixed
frictionless pulley at a distance of 2 m from the wall, has a point
mass M = 2 kg attached to it at a distance of 1 m from the wall. A
mass m = 0.5 kg attached at the free end is held at rest so that the
PRACTICEPROBLEMS 197

string is horizontal between the M


wall and the pulley and vertical
beyond the pulley. What will be the
speed with which the mass M will
hit the wall when the mass m is
released? (g ~ 9.8 m/s2). (lIT 1985)
~ ••••••.•.
n)..•................
A 0.5 kg block slides from
the point A (see figure) on
a horizontal track with an
initial speed of 3 mls
towards a weightless
horizontal spring oflength
--~~B!~~-
A D

1 m and force constant 2 Newton/m. The part AB of the track is


C

frictionless and the part Be has the coefficients of static and kinetic
friction as 0.22 and 0.2 respectively. If the distances AB and BD
are 2 m and 2.14 m respectively, find the total distance through.
which the block moves before it comes to rest completely.
(Take g ~ 10 m/s2). (lIT 1983)

Twoidentical cylindrical vessels with their bases at the same level


each contain a liquid of density p (rho). The height of the liquid in
one vessel is h, and in other is h2. The area of either base is A.
What is the work done by gravity in equalizing the levels when
the two vessels are connected? (lIT 1981)
I$.~I>.
The displacement x of a particle, moving in one dimension, under
the action of a constant force is related to the time t by the equation
t~ (A + 3 , where ~ is in metres and t in seconds.
Find:
(i) the displacement of the particle when its velocity is zero, and
(ii) the work done by the force in first 6 seoonds. (lIT 1979)

A ball falls under gravity from a height of 10 metres with an initial


downward velocity vo'It collides with ground loses 50%ofits energy
in collision and then rises back to the sam" height. Find:
198 PRACTICEPROBLEMS

(i) the initial velocity Vo and


(ii) the height to which the ball would .rise after collision, if the
initial velocity Vo was directed upward instead of downward.
(g = 9.8 mls'). (lIT 1979)
1541~,_'~ __ ~_~ ~~~
A 40 kg mass, hanging at the end of a rope oflength I, oscillates in
a vertical plane with an angular amplitude Go'What is the tension'
in the rope when it makes an angle G with the vertical? If
the breaking strength of the rope is 80 kg, what is the
maximum amplitude with 'which the mass can oscillate with the
rope breaking? (lIT 1978)
1551_'_' -------------------~. -'' -'_..._.._.~_.~-".
A particle of mass m is moving in a horizontal circle of radius r .

,.
under centripetal force - ~ ' where h is a constant. What is the
total energy of the particle?
(lIT 1977)

~_._:-_-------~-----
Two springs have their force constants as hI and h. (hi> h.). In
which spring is more work done:
(i) when their lengths are increased by the same amount
(ii) when they are stretched by the same force. (lIT 1976)
1571 _
A nail is located at a certain distance vertically below the point of
suspension of a simple 'pendulum. The pendulum bob is released
.from a position where the string makes an angle of 60. with
the vertical. Calculate the distance of the nail from the point of
suspension such that the bob will just perform revolutions
with the nail as centre. Assume the length of the pendulum to be
1 metre. (lIT 1975)
1581 _
A man weighing 60 kg climbs up a staircase carrying a 20 kg load
on his head. The stair case has 20 steps and each step has a height
20 em. If he takes 10 .seconds to climb, calculate the power.
(g =.9.8 mls') (lIT 1974)
PRACTICE PROBLEMS 199

1591 _

The length of a simple pendulum is 1 m. The bob of the pendulum


of mass 10 gm is released when the string is horizontal. When itis
at the lowest point of the path,
(i) what is its kinetic energy?
(ii) what is tension in the string?
(g = 980 crn/s2). (lIT 1973)

~-------------
The pulley system shown in the figure is used to
lift a weight W with uniform velocity by applying
a force F at the free end of the string.
F
(i) When the free end of the string is pulled
through a distance x by what distance does
Wmoveup?
(ii) What is corresponding change in the energy
ofW?
(iii) What is the magnitude of F and the work done by it?
(iv) What is mechanical advantage of the system?
(Neglect the friction and the mass of the pulley and strings).
(lIT 1973)

~~-------~
lliJ _
']\vo masses ml and m2 connected by a light spring of natu ral length
10 is compressed completely and tied by a string. This system while
moving with a velocity Vo along + ve x-axis pass through the origin
at t = O. At this position the string snaps. Position of mass m, at
time t is given by the equation
XI (t) = Vo t - A (1- cos", t)
Calculate ;
(a) position of the particle m2 as a function of time.
(b) 10 in terms of A. (lIT 2003)

[ill -------------------------
A particle of mass m, moving in a circular path of radius R with a
constant speed v2 is located at point (2R, 0) at time t = 0 and a man
200 PRACTICE PROBLEMS

starts moving with a velocity v I along the + ve y-axis from origin


at time t = O. Calculate the linear momentum of the particle with
respect to the man as a function oftime. (liT 2003)
y
!

~
....

~-------------
A car P is moving with a
unifordm speed of 5J3 3mls .rL_: r
towar s a camage 0 mass f p Ij

9 kg at rest kept on the


A
rails at a point B as shown
in figure. The heightAC is
120 m. Cannon balls of 1 kg are fired from the car with an initial
velocity 100 mls at an angle 30° with the horizozntal. The first
cannon ball hits the stationary carriage after a time to and sticks
to it. Determine to-
At to' the second cannon ball is fired. Assume that the resistive
force between the rails and the carriage is constant and ignore the
vertical motion of the carriage throughout. If the second ball also
hits and sticks to the carriage, what will be the horizontal velocity
of the carriage just after the second impact? (liT 2001)

[E]-------------
Two blocks of mass 2 kg
and M are at rest on an
inclined plane and are
separated by a distance
of 6.0 m as shown. The
coefficient of friction
between each of the
blocks and the inclined
PRACTICE PROBLEMS 201

plane is 0.25. The 2 kg block is given a velocity of 10.0 mls np


the inclined plane. It collide with M, comes back and has a velocity
of 1.0 mls when it reaches its initial position. The other block
M after the collision moves 0.5 m up and comes to rest. Calculate
the coefficient of restitution between the blocks and. the mass
ofthe block M. [Take sin e '" tan e = 0.05 and g = 10 mls2)
(lIT 1999)

[MJ~ir~~~~~~~~~~~~
A cart is moving along + x direction with a velocity of 4 mls. A
person on the cart throws a stone with a velocity of 6 mls relative
to himself. In the frame of reference of the cart the stone is thrown
in y-z plane making an angle of 30° with vertical z-axis. At the
highest point of its trajectory, the stone hits an object of equal
mass hung vertically from branch of a tree by means of a string of
length L. A completely inelastic collision occurs, in which the stone
gets embedded in the object. Determine:
(i) the speed of the combined mass immediately after the collision
with respect to an observer on the ground.
(ii) the length L of the string such that the tension in the
string becomes zero when the string becomes horizontal
during the subsequent motion of the combined mass ..
(lIT 1997, July)

~~.,
.~~~-----~~---
A small sphere of radius R is held y
against the inner surface of a
larger sphere ofradius 6R.(Fig.).
The masses of large and small
sphres are 4M and M, respectively. 0 x
This arrangement is placed on a Origin R

horizontal table. There is no


friction between any surfaces of contact. The small sphere is noy;
released. Find the coordinates of the centre of the larger sphere'
when the smaller sphere reaches the other extreme position.
'(lIT 1996)

.Two towers AB and CD are situated a distance 'd' apart as shown


.' in the figure. AB'is 20 m high and CD is 30 m high from the ground.
.202 PRACTICEPROBLEMS

An object of mass m is thrown


from the top ofAB horizontally
with a velocity of .~Oms- I
towards CD. Simultaneously m
A
another object of mass 2m is
thrown from the top of CD at
an angle of 60°to the horizontal . d
B D
towards AB with. the same
magnitude ofinitial velocity as
that of the first object. The two objects move in the same vertical
plane, collide in mid-air and stick to each other.
(i) Calculate the distance d between the towers and
(ii) Find the position where the objects hit the ground.
(lIT 1994)

A cylindrical solid of mass 1O-2kg and cross sectional ara 10-4 m2


is moving paralle to its axis (the x-axis) with a uniform speed of
103 mls in the positive direction. At t = 0, its front face passes the
plane x = O. The region to the right of this plane is filled with
stationary dust particles'of uniform density 10-3 kg/m3• When a
dust particle collides with the face of the cylinder, it sticks to its
surface. Assuming that the dimensions of the cylinder remain
practically unchanged, and that the dust sticks only to the front
face ofthe cylinder, find the x-coordinate ofthe front of the cylinder
at t = 150 sec. (lIT 1993)
1691 ~ ~ __ '_' '_-"_"'_'
A uniform thin rod of mass M and length L is standing vertically
along the Y-axison a smooth horizontal surface with its lower end
at the origin (0, 0). A slight disturbance at i = 0 causes the lower
end to slip on smooth suiface along the positive X-axis, and the
rod starts falling, .....
(i) What is the path followed by the centre of mass of the rod
during its fall? .
(ii) Find the equation to the trajectory ofa point on the rod located
at a.distance r from the lower end, What is the shape of the
path of this point? (lIT 1993)
PRACTICE PROBLEMS 203

12Q]--------------~
A block 'A' of mass 2 m
is placed on another ;. A 2m
block 'B' of mass 4 m
which in turn is placed III
on a fixed table. The ~v 2<1
two blocks have a same y--~
length 4d and they are __ .i..-~I.==:::;::=~.I-t.---
placed as shown in the 4,1
figure. The coefficient offriction (both static and kinetic) between
the block 'E' and table is fl. There is no friction between the two
blocks. A small object of mass m moving horizontally along a line
passing through the centre of mass (em) of the block Band
perpendicular to its face with a speed u collides elastically with
the block B at a height d above the table.
(a) What is the minimum value of u (call it volrequired to make
the block A topple? .
(b) Ifu = 2 vO' find the distance (from the point P in the figure) at
which the mass m falls on the table after collision.
(Ignore the role offriction during the collision.] (llT 1991)
@J -' -'......:-__ -'-- __
An object of mass 5 kg is projected with a velocity of 20'mls at an
angle of 60° to the horizontal. At the highest point ofits path the
I!rojectile explodes and breaks up into two fragments of masses 1
kg and 4 kg. The fragments separate horizontal1y after the
explosion. The explosion releases internal energy. such that the
kinetic energy of the system at the highest point is doubled.
Calculate the separation between the two fragments when they
reach the ground. (lIT 1990)
1121 ~_~_--'-'-_'__ _
A bul1et of mass M is fired with a velocity 50 m/s at an angle with
the horizontal. At the highest point ofits trajectory, it collideshead-
on with a bob ofmass 3 M suspended by a massless string oflength
10/3 metres and gets embedded in the bob. After the collision, the
string moves through an angle of 120°. Find:
(i) the angle e ;
(ii) the vertical and horizontal coordinates of the initial position
of the bob with respect to the point of firing ofthe bullet.
(Take g = 10 mls"). (lIT 1988)
204 '. PRACTICE PROBLEMS

,.,

A simple pendulum is "Suspendedfrom:a Peg ion a vett1ca1l'wll'll,


The;pendulum'is:pulled aWa!,lfinom the wa]],toa''I1orizontaliPosition.
an.d released. 'The ball hits :the ,wa!lI,the coefficient of•.estitution

being 'Js . Wihat is the mimmummumber:o'f'co'l'Iisionsafter ~hidh


the amplitude of oscillations,becomes 'less than 60:degnees ?
n!IT,r987~

A ball of 'mass TOO.gm is ,projectedvertica:Ily upwards from the


ground with a 'velocity of 49 'm'lsec. At the same time another
identical ball is dropped from a height of 98 m to falJ.freely along
the same path as that followed by the first ball. After some time
the two balls collideand stick together and finally fall to the ground.
Find the time of flight of the masses.(g = 9.8 mls2) (lIT 1985)

Two bodies A and B of masses m


and 2m respectively are placed
on a smooth floor. They y,
are connected by a spring. A.third 77ID;;;n}n~
body C of mass m moves with
velocity Vo along the line joining
A and B and collides elastically with A as shown in Fig. At a certain
instant of time to aftey collision, it is found that the instantaneous
velocities of A and B are the same. Further at this instant
'the compression of the spring is found to be xo' Determine
(i) the common velocity of A "nd B at time to; and
(ii) the spring constant. (lIT 1984)

A block of mass Mwith a semi-


circular track ofraGius R, rests on

L!~~
a horizontal frictionless surface. A ill
uniform cylinder of radius rand
M
mass til is released from rest at the B
top point A'(see figure). The
cylinder slips on'the semicircular
frictionless track ..How far has the
PRACTICE PROBLEMS 205

blockmo;v,ed when the'cylinderreaches the ,bottom (point B)'Qfthe


,traCk ',? How fast is .the block moving w>hen'the Icylinder,reaooes
the :J)nttomnf'the track ? ;(UT1983,

'Partic1es'p and 'Q of mass


.20 gm:and.40gm respec-
p
tivelyare'simultaneously
projectedfr.om !points A
and 'Bon.fhe;g1'ound,'The
initial vElloCitiesof l' and
Q 'make 45 .and 135
0 0
A
,L
angles respectively with the horizontal AB as shown in the figure,
Each particle has an intial speed of 49 mls. The separation AB is
245 m Both particles travelin the same vertical plane and undergo
a collision, After the collision l' retraces its path.
Determine the position of Q when it hits the ground. How much
time after the collision does the particle Q take to reach the ground .?"
(g = 9.8 mls2)
(lIT 1982)

A circular plate of uniform thickness has a diameter of'56 em. A


circular portion of diameter 42 em is removed from one'edge ofthe
plate as shown in the figure.

Find the position.ofthe centre of,mass oftheremaining,portion.


'(ilN' 1980)
206 . PRACTICE PROBLEMS

1791__ '_"_.. _.._- __ ~ __ . _-_"_' -_"__ ~_

A 20 'gm bullet pierces through a plate


of mass M, = 1 kg and then comes to
rest inside a second plate of mass M2 =
2.98 kg, as shown in the figure. It is
---~-- ~ ---->---
found that the two plates, initially at
rest now move with equal velociti~s.
Find the percentage loss in the initial.
velocity ofthe bullet when it is between
M, and M2. Neglect any loss ofmaterial M,
ofthe pla.tes, due to action ofthe bullet.
(lIT 1979)

A body of mass m moving with velocity V in the X.direction


collides with another body of mass M moving in Y.direction with
velocity u. They coalesce into one body during collision. Calculate ;
(i) the direction and magnitude of the momentum of the final
body.
(ii) the fraction of initial kinetic energy transformed into heat
during the collision in terms of the two masses. (lIT 1978)
[ill:-...-.---.
'.1

A bullet is fired from a rifle. If the rifle recoils freely, determine


whether the kinetic energy of the rifle is greater than, equal or
less than that of the bullet.
(lIT 1978)
1821, --.--
Aplastic ball is dropped from a heightofl M and rebounds several
times from the floor. If 1.3 seconds elapse from the moment it is
dropped to the second impact with the floor,what is the coefficient
of restitution ? (g = 9.8 mls2) (lIT 1977)
@J_'_'''_''''_'''_'_'' '_'''_''-_''_''' '"_,,,_
A mass of2.9kgis suspended from a string oflength 50 cm, and is
at rest. Another mass of 100gm, which is moving horizontally
with a velocity of 150 mls strikes and sticks to it.
(i) What is the tension in the string when it makes 60 with the 0

vertical ?
(ii) Will it complete a vertical circle? (g = 9.8 mls2) (lIT 1976)
PRACTICE PROBLEMS 207

A ball moving with a speed of


9m1s strikes an identical ball such
that after the collision the direction
of each ball makes an angle 30 0 O-;efo~ --------A~ft-- i~:--
with the original line of motion C Ii" . er
o lSlon Collision
(see figure).
Find speeds of the two balls after
collision. Is the kinetic energy
conserved in this collision process? (liT 1975)

1851-
A cricket ball of mass 150 grams is moving with a velocity of
12 mls and is hitted by a bat so that the ball is turned back with a
velocity of 20 mls. The force of the blow acts on the ball for
0.01 seconds. Find the average force exerted on the ball by the
bat. (liT 1974)

A wooden block of mass 10 grams is dropped from the top of a cliff


100 metres high. Simultaneously a bullet of mass 10 grams is fired
from the foot of the cliff vertically upwards with a velocity of
lOOmis.
(i) Where and after what time will they meet?
(ii) If the bullet after striking the block gets embeded in it, how
high will it rise above the cliff before it starts falling?
(g = 9.8 mls'). (lIT 1973)

A projectile of mass 50 kg is thrown vertically upward with an


initial velocity of 100 mls. After 5 seconds it explodes into two
fragments, one of which having mass 20 kg travels vertically up
with a velocity of 150 mls.
(i) What is the velocity of the other fragment of this instant?
(ii) Calculate the sum of momenta of the two fragments 3 seconds
after the explosion. What would have been the momentum of
the projectile at this instant if there had been no explosion?
(g = 9.8 mls'). (liT 1973)
208 PRACTICE PROBLEMS
r88w~' .
'~'-' ------------------------
A ba1l moving on a horizontal frictionless plane hits an ide'ntical
ball at rest .witha velocity of 50 cm/s. If the collision is elastic,
calculate the speed impaIted to the target ball if the speed of the
projectile ball after the collision is 30 cm/s. Show that, two balls
will move at right angles to each other after the collision.
(lIT 1972)

,1mmimI~ _
1891_: ~ ~
Three particles A, B and C each of mass
m, are connected to each other by three
massless rigid rods to form a rigid,
equilateral triangular body of side I.
This body is placed on a horizontal
frictionless table (xcY plane) and is
hinged to it at the point A so that it
can move without friction about the .E.
vertical axis through A (see figure). B c
The body is set into rotational motion
on the table about A with a constant angular velocity OJ.

(a) Find the magnitude of the horizontal force exerted by the


hinge on the body.
(b) At time T, when the side BC is parallel to the x-axis, a force F
is applied on B along BC (as shown). Obtain the x-component
and the y-component of the force exerted by the hinge on the
body, immediately after time T. (lIT 2002)
:190 1,~;-_. ._.. ==_'_7_._'_.._"_' __ -",
Two heavy metallic plates are A
joined together at 90. to each
other, a .laminar sheet of mass
30 kg is hinged at the line AB
joining the two heavy metallic
plates. The hinges are
frictionless.
The moment of inertia of the laminar sheet about an axis parallel
to AB and passing through its centre of mass is 1.2 kgm2. Two
rubber obstacles P and Q are fIXed, one on each metallic plate at a
distance 0.5 m from the line AB this distance is chosen so that the
PRAPICE PROBLEMS 209

reaction due to the hinges onthe laminar sheet is zero during the
impact. Initially theJaminar sheet hits one oUhe.obstacle with an
angularvelocity.1 rad/s and turns back. If the impulse ofthe sheet
due to each obstacle is 6 N-s, "
(a) find the location of the centre of mass of theJaminar sheet
fromAB.
(b) at what angular velocity does the laminar sheet come back
after the first impact?
(c) after how many impacts, does the laminar sheets cometo rest?
. (lIT 2001)

A rod AIl ofmass M and length L is lying on a horizontal frictionless


surface. A particle of mass m travelling along the surface hits the
end'~ of the rod with a velocity Vo in a direction perpendicular to
AB. The collision is completely elastic. After the collision the
particle comes to rest.
(a) Find the ratio mIM.
~
, (b) A point P on the rod is at rest immediately after the collision.
Find the distance AP.

(c) Find the linear speed ofthe point P at a time ( ;~) after the
collision. . (lIT 2000)

m,
F

A uniform circular disc has radius R and mass m. A parti,c1eah;o (If


mass m, is fixed at a point A on the edge ofthe disc as shown in the
figure. The disc can rotate freely about a fixed horizontal chord
210 PRACTICE PROBLEMS

". R
PQ that is at a distance 4" from A
the centre C ofthe disc. The line
AC is perpendicular to PQ.
Initially, the disc is held vertical
_____-1
Rl4
with the point A at its highest
position. It is then allowed to Q t
fall so that it starts rotating
aboutPQ. Find the linear speed
ofthe particle as it reaches its
lowest position. (lIT 1998)
eEl -------------------------
A wedge of triangular cross section (AB = BC = CA = 2R) is moving
with a constant velocity - vi towards a sphere of radius R fIxed
on a smooth horizontal table as shown in the fIgure. The wedge
makes an elastic collision with the fIxed sphere and returns along
the same path without any
rotation. Neglect all friction z
and suppose that the wedge
remains in contact with the
sphere for a very short time
tit during which the sphere x.
exerts a constant force F on
the wedge;
~ ~
(a) Find the force F and also the normal force N exerted by the
table on the wedge during the time tot.
(b) Let h denote the perpendicular distance between the centre
~
.of mass of the wedge and the line of action of F . Find tha
~
magnitude of the torque due to the normal force N about the
centre of the wedge, during the interval tot. (lIT 1998)
ill]
. Two thin circular. disks of mass 2 kg and
raidus 10 cm each are joined by a rigid
massless rod of length 20 cm. The axis
of the rod is along the perpendicular to
the planes ofthe disk through their centres <.- - --->
20 em
PRACTICEPROBLEMS 211

(see fig.). This object is kept on a truck in such a way that the axis
of the object is horizontal and perpendicular to the direction ofthe
motion of the truck.
Its friction with the floor of the truck is large enough so that the
object can roll on the truck without slipping. Take x axis as the
direction ofmotion ofthe truck and z axis as the vertically upwards
direction. If the truck has an acceleration of9 mls2, calculate
(i) the force offriction on each disk.
(ii) the magnitude and the direction oftlle frictional torque acting
on each disk about the centre of mass 0 of the object.
Express the torque in the vector form in terms ofunit vectors
in the i,J and k in the x, )' and z directions.
(liT 1997, July)

~---------------
A uniform disc ofmass m
and radius R is projected
horizontally with
velocity Vo on a rough
horizontal floor sothat it 1.=0 I.=t,
starts off with a purely sliding motion at t ~ O.After to seconds, it
acquires a purely rolling motion as shown in the figure.
(i) Calculate the velocity of the centre of mass of the disc at tao
(ii) Assuming the coefficient of friction to be fl, calculate to. Also
calculate the work done by the frictional force as a function of
time and the total work done by it over a time t much longer
than to' (liT 1997, May)

~---------------
A rectangular rigid fixed block has a
long horizontal edge. A solid
homogeneous cylinder of radius R is
placed horizontally at rest with its
length parallel to the edge such that
the axis of the cylinder and the edge of
the block are in the same vertical plane
as shown in the figure. There is
sufficient friction present at the edge
so that a very small displacement
causes the cylinder to roll off the edge
without slipping. Determine:
212 PRACTICEPROBLEMS

(a) the angle eo through which the cylinder rotates before it leaves
. contact with the edge.
(b) speed of~e centre of mass of the cylinder before leaving
contact with the edge, and
(e) ratio ofthe translational to rotational kinetic energies of the
cylinder when its centre of mass is in horizontal line with the
edge. (ITT 1995)
~Hn>.
A block X of mass 0.5 kg is held by
a long massless string on a
frictionless inclined plane of y
inclination 30° to the horizontal.
The string is wound on a uniform
solid cylindrical drum Y of mass 2
kg and of radius 0.2 m as shown in
the figure. The drum is given an
initial angular velocity such that
the block X starts moving up the
plane.
(i) Find the tension in the string during the motion.
(ii) At a certain instant of time, the magnitude of the angular
velocity of Y is 10 rad s- '. Calculate the distance travelled
by X from that instant of time until it comes to rest.
(lIT 1994)

Two uniform this rods A and B of p


length 0.6 m each and of masses 0.01
kg. and 0.02 kg. respectively are rigidly
joined, end to end. The combination is A
pivoted at the lihgter end P as shown
in the figure, such that it can freely
rotate about the point P in a vertical
plane. A small object of mass 0.05 kg,
moving horizontally hits the lower end B
of the combination and sticks to it.
What should be the velocity of the
object so that the system could just be •
raised to the horizontal position? (JIT 1994)
PRACTICE PROBLEMS , 213

A homogeneous rod AB of
length L = L8 m and mass
M is pivoted at the centre 0
in such a way that it can
rotate freely in the vertical
plane (see figure), The rod
is initially in the horizontal
position, An insect S of the
same mass M falls vertically with speed V on the point C, midway
between the points 0 and R Immediately after falling, the insect
moves towards the end B such that the rod rotates' with a constant
angular velocity 0),
(a) Determine the angular velocity 0) in terms of V and L,
(b) If the insect reaches the end B when the rod has turned
through an angle of 90°, determine V. (IIT 1992)

A carpet of mass M made of inextensible material is rolled along


its length in the form of a cylinder of radius R and is kept on a
rough floor, The carpet starts unrolling without sliding on the floor
when a negligibly small push is given to it, Calculate the horizontal
velocity of the axis of the cylinderical part of the carpet wnen its
radius reduces to R!2, (IIT 1990)

I~Q~I)
A thin uniform bar lies on a
frictionless horizontal surface and
10 ntIs ". A
is free to move in any way on the ~---~-----------
surface. Its mass is 0,16 kg and
length metres. Two particles, each
of mass 0,08 kg, are moving on the ~-----------B
same surface and towards the, ~ar 6m1;-;
in a direction perpendicular to the
bar, one with a velocity of 10 mls
and the other with 6 mis, as shown
in the figure, The first particle strikes the bar at point A and the
other at point B, Points A and B are at a distance of 0,5 m from the
centre of ,he bar, The particles strike the bar at the same instant
oftime and stick to the bar on collision: Calculate the loss of kinetic
energy of the system in the above collision process. (IIT 1989)
214 PRACTICE PROBLEMS

11031
------------------------
A small sphere rolls down without
slipping from the top of a track in T
a vertical plane. The track has an
elevated section and a horizontal .; A
part. The horizontal part is 1.0 ~; 1.0111
metre above the ground level and ~ +
the top of the track is 2.4 metres !J
above the ground. Find the distance on the ground with respect to
the point B (which is vertically below the end ofthe track as shown
in the figure) where the sphere lands. During its flight as a
projectile, does the sphere continue to rotate about its centre of
mass? Explain. (liT 1987)

11041

A particle is projected at time t = 0 from a point P on the ground


with a speed vo' at an angle of 45° to the horizontal. Find the
magnitude and direction ofthe angular momentum of the particle
Vo
aboutP at time t =-. (IlT 1984)
g

ApuIley arrangement is shown


in the figure. The cylinder Ahas
a diameter of 30 cm and the
cylinder B has a diameter of20
cm. The working handle has an
arm of 50 cm. The direction of
winding of the rope on A is
opposite to that on B.
Calculate the mechanicaladvantage ofthis arrangemel)t.(IIT 1980)
11061
A thin bar AB, whose weight p
can be neglected, is suspended
by strings from the two points 6
Gent lOclIl 4cm
A and B (see fJgure). The bar -A
carries masses of 10kg and 5 kg
as shown. Find the tensions in 10 5

the strings and the angle 8,


if the system is the equilibrium. (IlT 1977)
PRACTICE PROBLEMS 215

11071 _

A wheel of radius 40 ems rests


against a step of height 20 ems as
shown in the figure. What is the
minimum horizontal force which,
if applied perpendicular to the
axle, will make the wheel climbthe
step ? The mass of the wheel is
2 kg. (g ~ 9.8 m/s2) (lIT 1976)
[ioiJ _
A table has a heavy circular top of radius 1 metre and mass 20 kg.
It has four light legs oflength 1 metre fixed symmetrically on its
circumference.
(i) What is the maximum mass that may be placed any where on
this table without toppling the table?
(ii) What is the area ofthe table top over which any weight may
be placed without toppling it ? (lIT J 97'1)

~ --------------------
11091 _

R
There is a crater of depth 100 on the surface of the moon (radius
R). A projectile is fired vertically upward from the crater with a
velocity, which is equal to the escape velocity v from the surface of
the moon. Find the maximum height attained by the projectile.
(lIT 2003)
IllOI _
Distance between the centres of two stars is lOa. The masses of
these stars are M and 16 M and their radii a and 2a, respectively.
A body of mass m is fired straight from the surface of the larger
star towards the smaller star. What should be its minimum initial
speed to reach the surface of the smaller star? Obtain the
expression 'n terms of GM and a. (lIT 1996)
11111 _

An artificial satellite is moving in a circular orbit around the earth


with a speed equal to half the magnitude of escape velocity from
the earth.
216 PRACTICEPROBLEMS

(i) Determine the height of the satellite above the earth's


surface.
(ii) If the satellite is stopped suddenly in its orbit and allowed to
fall freely onto the earth, find the speed with which it hits the
surface of the earth. (lIT 1991)
11121,~ _
Three particles, each of mass m, are situated at the vertices of
an equilateral triangle of side length a. The only forces acting
on the particles are their mutual gravitational forces. It is desired
that each particle moves in a circle while maintaining the original
mutual separation a. Find the initial velocity that should be
given to each particle and also the time period of the circular
motion. (lIT 1988)

Twosatellites 8, and 82 revolve round a planet in coplanar circular


orbits in the same sense, Their periods of revolution are 1 hour
and 8 hours resppctively. The radius of the orbit of 8, is 10' km.
When 82 is closest to 8" find
(i) the speed of 82 relative to 81' \
(ii) the angular speed of 82 as actually observed by an astronaut
in 8" (lIT 1986)

Consider an earth satellite so positioned that it appears stationary


to an observer on the earth and serves the purpose of a fixed
relay station for intercontinental transmission of television
and other communication, What would be the height at
which satellite be positioned and what would be the direction of
its motion?
(Radius earth = 6.4 x lOs cm, g = 980 cm/s2) (lIT 1973)

The mass and diameter of a planet are twice those of earth, What
will be the period of oscillation of a pendulum on this planet ifit is
a seconds pendulum ofthe earth?
(lIT 1973)
PRACTICE PROBLEMS • 217

11161
~-------------
_

In a Searle's experiment, the diameter of the wire as measured by


a screw gauge of least count 0.001 em is 0.050 em. The length,
measured by a scale of least count 0.1 em, is 110.0 em. When a
weight of 50 N is suspended from the wire, the extension is
measured to be 0.125 em by a micrometer ofleast count 0.001 em.
Find the maximum error in the measurement ofYoung's Ifiodulus
of the material of the wire from these data. (IIT 2004)
11171, _
A 5 m long cylindrical steel wire with radius 2 x 10-3m is suspended
vertically from a rigid support and carnes a bob of mass 100 kg at
the other end. If the bob gets snapped; calculate the change in
temperature of the wire ignoring radiation losses.
(For the steel wire: Young's Modulus = 2.1 x lOll Pa; Density =
7860 kg/m"; Specific heat = 420 J/kg-K). (JIT 2001)
11181 . -"-"-'-'-' ._-------- -' -

A light rod of length 200 em is suspended from the ceiling


horizontally by means oftwo vertical wires of equal length tied to
its ends. One of the wires is made of steel and is of cross-section
0.1 sq em and the other is of bra~l"s,of' cross-section
0.2 sq em. Find out the position along the rod at which a
weight may be hung to produce
(i) equal stress in both wires
(ii) equal strains in bob wires.
(Yb""",= 1 x 101" dyne/em", Y."",r= 2 x l'Ol"dyne/cm")
(IIT 1974)
11191 _
A sphere of radius 10 em and' mass 25 kg is attached to the
lower end of a steel wire which is suspended from the ceiling of a
room, the point of support is 521 cm above the floor. When the
sphere is set swinging as a.simple pendulum, its lowest point just
grates the floor. Calculate the velocity of the ball at its lowest
position.
(Young'smodulus of steel = 20x lOll dyne/em". Unstretched length
of wire = 500 cm, Radius of steel wire =.0~05cm) g = 980 cm/s".
(lIT 1972)
218 '. ' PRACTICE PROBLEMS

~:'U???'I
I)~plun> .
A uniform solid cylinder of density
0,8 g/cm3 floats in equilibrium in a
combination of two non mixing
liquids A and B with its axis
vertical. The densities of the A
liquids A and Bare 0,7 g/cm3 and
1.2 g/cm3, respectively, The height
ofliquid A is hA = 1.2 cm the length B
of the part of the cylinder
immersed in liquid B is hB = 0,8
cm.
(a) Find the total force exerted by liquid A on the cylinder.
(b) Find h, the length of the part of the cylinder in air.
(c) The cylinder is depressed in such a way that its top surface is
just below the upper sm-face of liquid A and is then released.
Find the acceleration of the cylinder immediately after it is
released." (lIT 2002)
Ijg~I:::':::i'H,:::,
A wooden stick of length L, radius R and density p has a small
metal piece of mass ill (of negligible volume) attached to its one
end. Find the minimum value for the mass m (in terms of given
parameters) that would make the stick float vertically in
equilibrium in a liquid of density a (> p), (lIT 1999)

Ij~I'>'
A,ball of density d is dropped on to a horizontal solid surface. It
bounces elastically from the surface and returns to its original
position in a time't1. Next, the ball is released and it falls through
the same height before striking the surface ofa liquid of density dL.
(a) If d < du obtain an expression (in terms of d, t" and dL) for
the time 1, the ball takes to come back to the position from
which it was 'released.
(b) Is the motion of the ball simple harmonic?
(c) If d = dL,how does the speed of the ball depend on its depth
" inside the liquid? Negleot all frictional and other dissipative
. forces. Assume the depth of the liquid to be large.
(lIT 1992)
PRACTICE PROBLEMS 219

11231 _

A wooden plank of length 1 m and


uniform cross-section is hinged at one
end to the bottom of a tank as shown in
the figure. The tank is filled with water
upto a height 0.5 m. The specificgravity
ofthe plank is 0.5. Find the angle e that
the plank makes with the vertical in the
equilibrium position. (lIT 1984)
(Exclude the case e = 0°)

Acubical block of wood 10 cm along each side floats at the interface


between an oil and water with its lower surface 2 cm below the
interface. The heights of the oil and water columns are 10 cm each.
The density of oil is 0.8 gmlc.c.
(i) What is the mass of the block?
(ii) What is the pressure at the lower surface ofthe block?
(g = 10 mls2) (lIT 1977)
11251 _
A beaker containing water is placed on the pan of a balance which
shows a reading of M gms. A lump of sugar of mass m gms and
volume V cc. is now suspended by a thread in such a way that it is
completely immersed in water without touching the beaker and
without any overflow of water. What will be the reading of the
balance just when the lump of sugar is immersed? How will the
reading change as the time passes on ? (lIT 1978)
11261_-'-'- _
Acube of wood supporting 200 gm massjustfioats in water. When
the mass is removed, the cube rioes by 2 cm. What is the size ofthe
cube? (lIT 1978)
11271_' _
To what J:!eighta cylindrical vessel be filled with a homogeneous
liquid to make the force with which the liquid presses on the sides
of vessel equal to the force exerted by the liquid on the bottom of
the vessel? (lIT 1976)
220 PRACTICE PROBLEMS

112Sli
-----------------------~
Apiece ofcork is embeded inside an ice block which floats in water.
What will happen to the level of water when all the ice melts?
(lIT 1976)
1291
1
------------------------
A rod of length 6 metres has a mass of 12 kg. It is hinged at one
end at a distance of 3 metres below a water surface.
(i) What weight must be attached to the other end ofthe rod so
that 5 metres of the rod are submerged?
(ii) Find the magnitude and direction of the force exerted by the
hinge on the rod. .
(specific gravity ofthe material of the rod is 0.5)
(lIT 1976)

A large block ofice 5 metre thick has a vertical hole drilled through
and is floating in the middle oflake. What is the minimum length
of a rope required to scoop up a bucket full of water through the
hole? (density of ice = 0.9 gm/c.c.)- (lIT 1975)

A cubical block of iron 5 cm on each side is floating on mercury in


av'esseI.
(i) What is the height of the block above the mercury level?
-(iiF Water is poured into the vessel until it just covers the iron
block. What is the height of the water column?
(density of mercury = 13.6 gm/c.c., density of iron = 7.2 gm/c.c.)
(lIT 1973)
PRACTICE PROBLEMS 221

11341 _

A piece of ice with a stone frozen in it floats on water kept in a


beaker. Will the level of water increase, decrease as remain the
same when the ice completely melts? (lIT 1973)
11351 _

A vertical U tube ofuniform inner


cross-section contains mercury in Oil H~ Glycerine
both its arms. A glycerine (density
1.3 gmIc.c.) column oflength 10em
is introduced into one of its
il!
arms. Oil of density (0.8 gm/c.c.) is
poured in the other arm until the
upper surface of oil and glycerine
are in the same horizontal level as Mercury
shown in the figure. Find the length of the oil column.
Density of mercury is 13.6 gmIc.c. (lIT 1972)
11361 _

A piston of cross sectional area 100 cm2 is used in a hydraulic


press to exert a force of 107 dynes on the water. What is the cross
sectional area of the other pistall which supports a truck having a
mass of 2000 kg ?
(g = 980 cm/s2). (lIT 1972)

~~--------
11371, _
Consider a horizontally oriented syringe containing water located
at a height ofl.25 m above the ground. The diameter ofthe plunger
is 8 mm and the diameter of the nozzle is 2 mm. The plunger is
pushed with a constant speed of 0.25 m/s. !"ind the horizontal range
of water stream on the ground. Take g = 10 mls2• (lIT 2004)
11381 _

A nonviscous liquid of constant Q


density 1000 kg/m3 flows in a
streamline motion along a tube of
variable cross section. The tu be is p 5m
kept inclined in the vertical plane
as shown in the figure. The area 2m
of cross section of the' tube at
two points P and Q at heights of
222 . PRACTICE PROBLEMS

2 meters and 5 meters are respectively Ix 10-3 m2 and 8x 10-3 m2•


The velocity of the liquid at point Pis 1 mls. Find the work done
per unit volume by the pressure and the gravity forces as the fluid
flows from point P to Q. (liT 1997, July)
11391__ ~ _

A large open top container of negligible mass and uniform cross-

sectional area A has a small hole of cross-sectional area l~O in its


side wall near the bottom. The container is kept on a smooth
horizontal floor and contains a liquid of density p and mass mo'
Assuming that the liquid starts flowing out horizontally through
the hole at t = 0, calculate
(i) the acceleration of the container, and
(ii) its velocity when 75% of the liquid has drained out
(lIT 1997, May)

A container oflarge uniform cross-


sectional area A resting on a
horizontal surface, holds two . :0.
immiscible, non-viscous

d and 2d, each of height


and
incompressible liquids of densities
H
"2 as
Jitl

HI2
I
~

oOdOO

-----
0

-------------
0

2d------
0

-------~-~--~--~-
------
shown in the figure. The lower
density liquid is open to the
atmosphere having pressure Po'
(a) A homogeneous solid cylinder of length L (L < H/2), cross-
A
section area '5 is immersed such that it floats with its axis

vertical at the liquid-liquid interface with length ~ in the


denser liquid. Determine.
(i) the density D of the solid and,
(ii) the total pressure at the bottom of the container.
(b) The cylinder is removed and the original arrangement is
restored. A tiny hole of area s (8 « A) is punched on the
vertical side of the container at a height h (h < H/2).
Determine:
PRACTICEPROBLEMS 223

(i) the initial speed of efflux of the liquid at the hole,


(ii) the horizontal distance x travelled by the liquid initially,
and
(iii) the height hm at which the hole should be punched so
that the liquid travels the maximum distance x", initially.
Also calculate x",.
(Neglect the air resistance in these calculations). (lIT 1995)
mmmmm _
11411 _

A small sphere falls from rest in a viscous liquid. Due to friction,


heat is produced. Find the relation between the rate of production
of heat and the radius ofthe sphere at terminal velocity. (lIT 2004)
11421 _

A liquid of density 900 kg/m3 is filled in


a cylinderical tank of upper radius 0.9
m and lower radius 0.03 m. A capillary .....-
tu be of length 1is attached at the bottom
of the tank as shown in the figure. The
capillary has outer radius 0.002 m and 11
inner radius a. When pressure P is
applied at the top of the tank volume a.3m
flow rate of the liquid is 8 x 10- 0 m3/s
and if capillary tube is detached, the
liquid comes out from the tank with a
velocity 10 m/s. Determine the
coefficient of viscosity of the liquid.
a2
[Given: 1t a2 = 10-0 m2 and -1- = 2 x 10-0 m) (lIT 2003)

~----------
11431 _

A container of width 2a is filled with a liquid. A


thin wire of weight per unit length A, is gently ~2a~
placed over the liquid surface in the middle of
the surface as shown in the figure. As a result,
the liquid surface is depressed by a distance
y(y« a). Determine the surface tension of the
liquid.
W
===- " .:==
=====--====
-----------
-----------
===========
-----------

(lIT 2004)
224 , PRACTICE PROBLEMS

II~I
II,'.>'
A bubble haviug surface tension T and
radius R is formed on a ring of radius
b (b« R).Air is blown inside the tube
with velocity v as shown in the figure. C--+
" V~:
The air molecule collides. --'-+ ....
perpendicularly with the wall of the
bubble and stops. Calculate the radius
at which the bubble separates from
the ring. (lIT 2003)

mm~~~~~~~~==~L
I,N-til,,"::",,",'"
==============
A solid sphere of radius R is floating in a liquid of density p with
half ofits volume submerged. Ifthe sphere is slightly pushed and
released, it starts performing simple harmonic motion. Find the
frequency ofthese oscillations. . (lIT 2004)

A thin rod of length L and area of cross section S is pivoted at its


lowest point P inside a stationary, homogeneous and non viscous
liquid (see figure).
The rod is free to rotate in a
vertical plane about a horizontal
axis passing through P. The
density dJ ofthe material ofthe rod
is smaller than the density d2 of
the liquid. The rod is displaced by
a small angle e from its
equilibrium position and then ----- :':Y. •••
___ c.__ p
----------
------,-_._- _
released. Show that the motion
ofthe rod is simple harmonic and
determine its angular frequency in
terms of the given parameters. (lIT 1996)

Two identical balls A and. B each of mass 0.1 kg are attached to


two identical massless springs. The springcmass. system is
constrained to move inside a rigid smooth pipe bent in the form.of
PRACTICE PROBLEMS 225

a circle as shown in the figure. The


pipe is fixed in a horizontal plane.
The centres of the balls can move
in a circle, of radius 0.06 metre.
Each spring has a natural length
0.06 1t metre and spring constant Q
0.1 N/m. Initially, both the balls are
n
displaced by an angle e = '6 radian
with respect to the diameter PQ of
the circle (as shown in the figure)
and released from rest.
(i) Calculate the frequency of oscillation of ball B.
(ii) Find the speed of ball A when A and B are at the two ends of
the diameter PQ.
(iii) What is the total energy of the system? (lIT 1993)

Two non-viscous, incompressible and


immiscible liquids of densities p and 1.5 p
are poured into the two limbs of a circular
tube of radius R and small cross-section
kept fixed in it. vertical plane as shown in
the figure. Each liquid occupies one-forth
the circumference of the tube.
(a) Find the angle e that the radius to the
interface makes with the vertical in
equilibrium position.
(b) Ifthe whole is given a small displacement from its equilibrium
position, show that the resulting oscillations are simple
harmonic. Find the time period of these oscillations.
(lIT 1991)

Twolight springs of force constants


h, and h2 and a block oflpass m
are in one line AB on a smooth
horizontal table such that on'e end
of 'each spring is 'fixed" on
rigid" supports and the other
end is free as shown in the figure.
226 PRACTICE PROBLEMS

The distance distance CD between the free ends of the springs is


60 ems. If the block moves along AB with a velocity 120 em/sec in
between the springs, calculate the period of oscillation of the
block.(k, = 1.8 N/m, k2 = 3.2 N/m, m = 200 gm) (lIT 1985)
11501
------------------------
A point mass m is suspended at the end of a massless wire oflength
I and cross section A. IfY is the Young's modulus for the wire,
obtain the frequency of oscillation for the simple harmonic motion
along the vertical line. (lIT 1978)

11511
------------------------
A particle of mass 10 gm is describing a simple harmonic motion
along a straight line with a period of 2 seconds and amplit\lde of
10 em. What is the kinetic energy when it is
(i) 2 em from equilibrium position?
(ii) 5 em from equilibrium position?
How would you account for the difference between these two values?
(lIT 1972)
ISmD I
11521 _

A string tied between x = 0 and x = Ivibrates in fundamental mode.


The amplitude A, tension T and mass per unit length II is given.
Find the total energy of the string. (lIT 2003)
15
1 31_.- ----------------
In a resonance tube experiment to determine the speed of sound
in air, a pipe of diameter 5 em is used. The air column in pipe
resonates with a tuning fork of frequency 480 Hz when the
minimum length of the air column is 16 em. Find the speed of
sound in air at room temperature. (lIT 2003)
11541-
----------------~--~---~
Two narrow cylindrical pipes A and B have the same length. Pipe
A is open at both ends and is filled with a monoatomic gas of molar
mass MA• Pipe B is open at one end and closed at the other end,
and is filled with a diatomic gas of molar mass MB• Both gases are
at the same temperature.
PRACTICE PROBLEMS 227

(a) If the frequency to the second harmonic of the fundamental


mode in pipe A is equal ofthe frequency ofthe third harmonic
of the fundamental mode in pipe B, determine the value of
MA
Ma'
(b) Now the open end of pipe B is also closed (so that the pipe is
closed at both ends). Find the ratio of the fundamental
frequency in pipe A to that in pipe B. (liT 2002)
11551 _
A 3.6 m long vertical pipe resonates with a source of frequency
212.5 Hz when water level is at certain heights in the pipe. Find
the heights of water level (from the bottom of the pipe) at which
resonances occur. Neglect end correction. Now,the pipe is filled to
a height H (", 3.6 m). A small hole is drilled very close to its bottom
and water is allowed to leak obtain an expression for the rate of
fall of water level in the pipe as a function ofH. Ifthe radii of the
pipe and the hole are 2 x ]0-" m and 1 x ]0- 3 m respectively,
calculate the time inten>al between the occurrence of first two
resonances. Speed of sound in air is 340 mls and g'" 10 mls".
(lIT 2000)
11561 _

A long wire PQR is made by joining two wires PQ and QR of equal


radii. PQ has length 4.8 m and mass 0.06 kg. QRhas length 2.56 m
and mass 0.2 kg. The wire PQR is under a tension of 80 N. A
sinusoidal wave-pulse of amplitude 3.5 em is sent along the wire
PQ from the end P. No power is dissipated during the propagation
ofthe wave-pulse. Calculate
(a) the time taken by the wave-pulse to reach the other end R of
the wire, and
(b) the amplitude of the reflected and transmitted wave-pulses
after the incident wave-pulse crosses the joint Q. (liT 1999)
11571 ~ _
The air column in a pipe closed at one end is made to vibrate in ita
second overtone by a tuning fork offrequency 440 Hz. The speed
of sound in air 330 ms-I. End corrections may be neglected. Let Po
denote the mean pressure at any point in the pipe, and l>P v the
maximum amplitude of pressure variation.
228 PRACTICEPROBLEMS

(a) Find the length L of the air column.


(b) What is the amplitude of pressure variation at the middle of
the column ?
(c) What are the maximum and minimum pressures at the open
end of the pipe?
(d) What are the maximum and minimum pressures at the closed
end of the pipe? (lIT 1998)

It~I)'
The first overtone of an open organ pipe beats with the first
overtone of a closed organ pipe with a beat frequency of 2.2 Hz.
The fundamental frequency of the closed organ pipe is 110 Hz.
Find the lengths of the pipes. (IIT 1997, May)

11~~1> .
A metallic rod of length 1 m is rigidly clamped at its mid-point.
Longitudinal stationary waves are set up in the rod in such a way
that there are two nodes on either side of the mid-point. The
amplitude of an antinode is 2 x 10-6 m. Write the equation of motion
at a point 2 cm from the mid-point and those of the constituent
waves in the rod. (Young's modulus = 2 x 1011Nm- 2, density =
8000 kg m- 3) (IIT 1994)

I~~l(>
Tworadio stations broadcast their programs at the same amplitude
A, and at slightly different frequencies 00, and 002 respectively.
002 - 00, = 103 Hz. A detector receives the signals from the two
stations simultaneously. It can only detect signals of intensity"
2A2.
(i) Filld the time interval between successive maxima of the
intensity of the signal received by the detector.
(ii) Find the time for which the detector remains idle in each cycle
of the intensity of the signal. (IIT 1993)
I~~tl> .
The displacement of the medium ,n a sound wave is given by the
equation
y, = A cos (ax + btl where A, a and b are'positive constants.
The wave is reflected by an obstacle situated" at x = O.The intensity
dfthe reflected wave is 0.64 time that of the incident wave.
(a) What are the wavelength and frequency of incident wave?
(b) Write the equation for the reflected wave.
PRACTICE PROBlEr.IS 229

(e) In the resultant wave formed after reflection, find the


maximum and minimum values of the partide speeas in the
medium.
(d) Express the resultant wave as a superposition of a standing
wave and a travelling wave. What are the positions of the
antinodes of the standing wave? What is the direction of
propagation of travelling wave? (IIT 1991)
11621 _

The following equations represent transverse waves:


z, = A cos (kx - rot)

z. = A cos (kx + rot)


Z3 = A cos (ky - rot)
Identify the combination(s) of the waves which "ill produce
(i) standing wave(s),
(ii) a wave travelling in the direction making an angle of 45
degrees with the positive x and positive y axes. In each case,
find the positions at which the resultant intensity is always
zero. (lIT 1987)

o
The vibrations of a string of length 60 cm fixed at both ends are
represented by the equation:

y= 4sin(~:)cOS(96"t)

where x and y are in cm and t in seconds.


(i) What is the maximum displacement of a point at x = 5 cm ?
(ii) Where are the nodes located along the string?
(iii) What is the velocity of the particle at x = 7.5 cm at t = 0.25
sec. ?
(io) Write down the equations of the component waves whose
superposition gives the above wave. (IIT 1985)
11641 ~_
A uniform rope of length 12 m and mass 6 kg hangs vertically
from a rigid support. A block of 2 kg is attached to the free end of
the rope. A transverse pulse of wavelength 0.06 m is produced at
the lower end of the rope. Wb.at is the wavelength of the pulse
when it reaches the top of the rope? (lIT 198'\)
23.0 PRACTICE PROBLEMS
~
- t-, .•.•• -.-."
.';.
". ,." .'--,.,- , ..'. '" .. '""'- . ,~.~;

,,-.-. .' . .'

A steel wi,e aflength 1 m, mass 0.1 kg and unifatm crass-sectianal


area 10- 6 m2 is rigidly fixed at bath ends. The temperature .ofthe .
wire is lawered by 20°C. If transverse waves are set up by plucking
the string in the middle, calculate.the frequency .offundamental
made .ofvibratian. (lIT 1984)

A sanameter wire under tensian .of64 Newtons vibrating in its


fundamental made is in resanance with a vibrating tuning fark.
The vibrating partian .ofthe sa'nameter wire has a length .of10 cm
and a mass .of1 gm. The vibrating tuning fark is naw maved away
fram the vibrating wire with a canstant speed and an .obServer
standing near the sanameter hears .one' beat per secand.
Calculate the speed with which the tuning fark is maved if the
speed .ofsaund in air is 300 mls. (lIT J983)
~'-""-."" ,-
•. :.__ .~' _.'U_~" __ C' •• _~ •• 0_ •• .~._. _ :'. ~ , •• ~ ~ "

~i'.' ~'~ "

A string 25 cm lang and having a mass .of2.5 gm is under tensian.


A pipe c1ased at .oneend is 40 cm lang. When the string is set
vibrating in its first avertane and the air in the pipe in its
fundamental frequency, 8 beats per secand are heard. It is .observed
that decreasing the tensian in the string decreases the beat
frequency. If the speed .ofsaund in air is 320 mis, find the tensian
in the string. (lIT 1982)
.. "... _._V~ ..-:-~~ .__ ..
11681" .
A tube .ofa certain diameter and .oflength 48 cm is .openat bath \
ends. Its fundamental frequency .ofresanance is faund to be 320
Hz. The vel.ocity.ofsaund in air is 320 mlsec. Estimate the diameter
.ofthe tube.
One end .ofthe tube is naw c1ased.Calculate the lawestfrequency
afresanance far the tube. (lIT 1980)

A metal wire .ofdiameter 1 mm is held an twa knife edges separated


by a distance af50 cm. The tensian in the wire is 100 N. The wire,
vibrating with its fundamental frequency, and a vibrating tuning
fark together praduce 5 beats/sec. The tensian in the wire is then
reducted to 81 N. When the twa are excited beats are heared again
at the same rate, calculate.
PRACTICE PROBLEMS 231

(i) frequency of the fork.


(ii) density of the material of the wire. (lIT 1980)
11701 _

ABis a cylinder oflength 1m fitted


with a thin flexible diaphragm C
at the middle and other thin
flexible diaphragms A and B at the
ends. The portions AC and BC
contain hydrogen and oxygen
gases respectively.The diaphragms
A and B are set into vibrations of same frequency. What is the
minimum frequency of these vibrations for which diaphragm C is
a node? (Under the conditions of experiment vE2 = 1100 mIs,
v02 = 300 mls). (lIT 1978)

11711 _

A column of air at 5loC and a tuning fork produces 4 beats per


second when sounded together. As the temperature of the air
column is decreased, the number of beats per second tends to
decrease and when the temperature is !G°C, the two produce 1
beat per second. 'Find the frequency of the tuning fork. (lIT 1977)
11721 _

Velocity of sound in a tube containing air at 20° C and a pressure


of 76 cm of mercury is 330 metres/sec. What will be its velocity
when the pressure is increased to 100 cm of mercury and the
temperature is kept constant? (lIT 1976)
11731 _

1\\'0 wire of radii I' and 21' respectively are welded together'end to
end. The combination is used as a sonometer wire and is kept under
tension T. The welded point is midway between the two bridges.
What would be the ratio ofthe number ofloops formed in the wires
such th:~tthe joint is a node when stationary vibrations are set up
in the wires? (lIT 1976)
11741 ~ ~
A tuning fork having a frequency of 340 vibrations/sec is vibrated
just above a cylindrical tube. The height of the tube is' 120 cr.l.
232 PRACTICE PROBLEMS

Water is slowly poured in it. What is the minimum height of water


required for resonance?
(Velocityof sound in air = 340 mls) (liT 1975)
11751 _
A wire of density 9 gmlc.c. is stretched between two clamps 100 cm
apart while subjected to an extension of0.05 cm. What is the lowest
frequency of transverse vibrations in the wire assuming Young's
modulus of the material to be 9 x 10" dyne/cm2? (lIT 1975)

11761
A tuning fork of unknown frequency when sounded with another
offrequency 256 Hz gives a 4 beats and when loaded with a certain
amount ofwax, it is again fdund to give 4 beats. Find the unknown
frequency. (liT 1974)

A man standing in front of a mountain at a certain distance beats


a drum at regular intervals. The druming rate is gradually
increased and he finds that the echo is not heard distinctly when
the rate becomes 40 per minute. He then moves nearer to the
mountain by 90 metres and finds that echo is not heard when the
druming rate becomes 60 per minute. Calculate :
(i) distance between the mountain and the initial position of the
man,
(ii) velocity of the sound. (liT 1974)

11781
A pipe oflength 1.4 m closed at one end is filled with a gas and it
resonates in its fundamental tone with a tuning fork. Another pipe
of the same length but open at both ends is filled with air and is
resonates in its fundamental tone with the same tuning fork.
Calculate the velocity of sound at 0° C in the gas, given that the
velocity of sound in air is 360 mls at 30°C where the experiment is
performed. (liT 1974)

11791
A sonometer wire fixed at one end has a solid mass M hanging
from its other end to produce tension in it. It is found that 70 cm
length ofthe wire produces a certain fundamental frequency when
plucked. When the same mass M is hanging in water, completely
submerged in it, itis found that the length ofwire has to be changed
PRACTICE PROBLEMS • 233

by 5 cm in order that it will produce the same fundamental


frequency. Calculate the density of the material of the mass M
hanging from the wire. (lIT 1972)

~~----------------
11801 _

A boat is travelling in a river with a speed 10 mls along the stream


flowing with a speed 2 m/s. From this boat, a sound transmitter is
lowered into the river through a rigid support. The wavelength of
the sound emitted from the transmitter inside the water is 14.45
mm.
Assume that attenuation of sound in water and air is negligible.
(a) What will be the frequency detected by a receiver kept inside
the river downstream?
(b) The transmitter and the receiver are now pulled up into air.
The air is blowing with a speed 5 mls in the direction opposite
the river stream. Determine the frequency of the sound
detected by the receiver. Given
Thmperature of the air and water = 20°C
Density of river water = 103 kg/ms
Bulk modulus ofthe water = 2.088 x 109 Pa
Gas constant R = 8.31 J/mol-K
Mean molecular mass of water = 28.8 x 10- 3 kg/mol

~p for air = 1.4 (lIT 2001)


v
11811 _

A band playing music atfrequency fis moving towards a wall at a


speed Vb' a motorist is following the band with a speed Vm. If V is
the speed of sound, obtain an expression for the beat frequency
heard by the motorist. (lIT 1997, July)
2
118 1_' --------------'---'-----

A whistle emitting a sound offrequency 440 Hz is tied to a string


of 1.5 m length and rotated with an angular velocity of 20 rad S-I
in the horizontal plane. Calculate the range of frequencies heard
by an observer stationed at a large distance from the whistle.
(lIT 1996)
234 PRACTICEPROBLEMS

li831
------------------------

e
Asource of sound is moving along
a circular orbit ofradius 3 metres
with an angular velocity of 10
rad/s. A sound detector loca~d I_ Gm _I' Gm '1
far away from the source IS ~ ........•• ••
executing linear simpleharmonic '0" ABC J)
motion along the line BD with an .
amplitude BC = CD = 6 metres.
The frequency ofoscillation ofthe
5
detector is -n per second. The source is at the point A when the
detector is at the point B. If the source emits a continuous sound
wave offrequency 340 Hz, find the maximum and the minimum
frequencies recorded by the detector. (I1T 1990)
11841
A train approaching a hill at a speed of 40 km/hr sounds a whistle
offrequency 580 Hz when it is at a distance of 1 km from a hill. A
wind with a speed of40 km/hr is blowiIl':.in the direction ofmotion
of the train. Find
(i) the frequency of the whistle as heard by an observer on the
hill, and
(ii) the distance from the hill at which the echo from the hill is
heard by the driver and its frequency.
(Velocityof sound in air = 1,200 km/hr). (I1T 1988)

Two tuning forks with natural frequencies of 340 Hz each move


relative to a stationary observer. One fork moves away from the
observer, while the other moves towards him at the same speed.
The observer hears beats offrequency 3 Hz. Find the speed of the
tuning fork.
(speed of sound = 340 mls) (lIT 1986)
11861
A source of sound offrequency 256 Hz is moving rapidly towards a
wall with a velocity of 5 mlsec. How many beats per second will be
heard if sound travels at a speed of 330 mlsec ?
(liT 1981)
PRACTICE PROBLEMS 235

~'---------
11871_,. _
A cube of coefficient of linear expansion "'. is floating in a bath
containing a liquid ofcoefficient of volume expansion 1/.When the
temperature is raised by "'T, the depth upto which the cube is
submerged in the liquid remains the same. Find the relation
between "'. and 1" showing all the steps. (lIT 2004)
11881_' '_' _
The apparatus shown in
the figure consists of four
glass columns connected
by horizontal sections. The A B C o
height of two central 95' 5' 95~ s'
columns Band C are 49 em
each. The two outer -
columns A and D are open
to the atmosphere. A and Care maintaned at a temperature of
95°C while the column Band D are maintained at 5°C. The height
of the liquid in A and D measured from the base line are 52.8 cm
and 51 cm respectively. Determine the coefficient of thermal
expansion of the liquid. (lIT 1997,July)
11891 ~ __
A thin rod of negligible mass and area of cross-section 4 x 10-6 m2,
suspended vertically from one end, has a length of 0.5 m at 100°C.
The rod is cooled to O°C, but prevented from contracting by
attaching a mass at the lower end. Find
(i) this mass, and
(ii) the energy stored in the rod.
Given for the rod: Young's modulus = 10" N/m2, Coefficient of
linear expansion 10-5 K-l and g = 10 mls2. (lIT 1997, May)
11901 _

A composite rod is made by joining a copper rod, end to end, with


a second rod of a different material but of the same cross s<.-ction.
AtZ5°C, the composite rod is 1 meter in length, of which the length
of copper rod is 36 em. At 125°C the length of the composite rod
increases by 1.91mm. When the composite rod is allowed to expand
by holding it betweeh two rigid walls, it is found that the length of
the two constituents do not change with the rise of temperature.
236 • PRACTICE PROBLEMS

Find the Young's modulus and the coefficient of linear expansion


of the second rod.
(Forcoppe;: : Y = 1.3 x 1011 Nlm2, a = 1.7 x lO-5rC) (liT 1979)
1 1 ~m-c~-' _~._ O __ ~. __ _,
.! _ ~ • m -:-: " 1

191
A copper wire is held at two ends by rigid supports. At 30°C, the
wire is just taut, with negligible tension. Find the speed of
transverse waves in this wire at 10°C.
(For copper: a = 1.7 x 1O-5rC, Y = 1.3 x 1011N/m2, density = 9 x
10-'kgm3) (liT 1979)
11921 ""m_mo... • ••• O.

A sinker ofweight Wo has an apparent weight w, when weighed in


a liquid at a temperature t, and "'2 when weight in the same liquid
at tempera.ure t2• The coefficient of cubical expansion of the
material of sinker is Il.What is the coefficient ofvolume expansion
of the liquid. (liT 1978)
!1931 ~ _
A clock with an iron pendulum keeps correct time at 20°C. How
much will it lose or gain if the temperature changes to 40°C?
(Coefficient of cubical expansiou of iron = O.000036/"C).
(liT 1977)

The brass scale of a barometer gives correct reading at O°C.The


coefficient of thermal expansion of brass is O.OOOO2l°C. The
barometer reads 75 cm at 27°C. What is the atmospheric pressure
at OOC, (liT 1977)
11951_. -._. ~'_' _-_~"'_- _"-_' ~.. ,_.-_._.~-O"-_"_;. _
What should be the length of a steel and copper rod be at O°Cso
that the length of the steel rod is 5 cm longer than the copper rod
at any temperature? (Coefficient of linear expansion for copper =
1.7 x lo-5rC, coefficient of linear expansion for iron = 1.1 x lo-5rC
(liT 1976)

Two rods of different metals


having the same area ofuoss--
section A, are placed end to
end between two massive walls
F" 0,
I - -':;..

Y.
,

as shown in figure. The first


rod has length 1" ccefficient of
PRACTICE PROSlEr/iS 237
linear expansion «, and Young's modulus Y I. The corresponding
quantities for second rod are /2' «2 and Y2. The temperature of
both the rods is raised by T degrees.
(i) Find the force with which the rods act on each other (at the
higher temperature) in terms of the given quantities.
(ii) Also find the lengths of the rods 'it higher temperature.
Assume that there is no change in the cross-sectional area of the
rods and the rods do not bend. There is no deformation ofthe walls.
(lIT 1975)
11971 _
A piece of metal weighs 46 gm in air. When it is immersed in a
liquid of specific gravity 1.24 at 27°C it weighs 30 gm. When the
temperature of liquid is raised to 42°C, the metal piece weighs
30.5 gm. Specific gravity of the liquid at 42°C is 12.0. Calculate
the coefficient ofJinear expansion of the metal. (lIT 1974)
\i981 _
The difference between the length of a certain brass rod and that
of a steel rod is claimed to be constant at all temperatures. Is this
possible? (lIT 1974)
11991 _
A one litre glass flask contains some mercury. It is found that at
different temperatures the volume of air inside the flask remains
same. What is the volume of mercury in this flask?
(coefficient oflinear expansion of glass = 9 x lO-G/oC,coefficient of
cubical expansion of mercury = 180 x lO-Gi"C). (lIT 1973)

~-------------
A metallic bob weighs 50 grams in air. It is immersed in a liquid at
a temperature of25°C it weighs 45 grams. When the temperature
ofthe liquid is raised to 100°C,it weighs 45.1 grams. Calculate the
coefficient of cubical expansion of the liquid. Assuming the
coefficient ofJinear expansion of the metal to be 12 x lO-G/oC.
(lIT 1973)

12011__ ~ '__ _
2
A steel wire of cross-sectional area 0.5 mm is held between two
fixed supports. If the tension in this wire is negligible and itisjust
taut at a temperature of 20°C, determine the tension when the
238 PRACTICE PROBLEMS

temperature falls to O°C.Assume that the distance between the


supports remains the same.
(a. = 12 x lO--G/oC,
Y = 2.1 x 1012 dyne/cm2) (lIT 1973)
CALORIMETRY

An ice cube of mass 0.1 kg at O°Cis placed in an isolated container


which is at 227°C. The specific heat S of the container varies with
temperature T according to the empirical relation
S =A+ BT,
where A = 100 cal/kg-K and B = 2 x 10--2 callkg-K".
If the final temperature of the container is 27°C, determine the
mass of the container.
(Latent heat of fusion of water = 8 x 10' callkg-K,
Specific heat of water = 10"callkg-K) (lIT 2001)
120.31 k~~~~~'J?i~;t~~~~:~h'~;_t:;f!~{~~~~
The temperature of 100 gm of water is to be raised from 24°C to
90°C by adding steam to it. Calculate the mass of the steam.
required for this purpose. ' (lIT 1996)
12041-"".Jt'..
. ftJ•.•..
j3!'~'.'~'r.l;:!<-*!\~r-~~~~"1;~.,
/?\:l"«'.~ ~c- -, ;.~;.,:
.••
sr :~.q\~~~;,;n~~yt1I;>!"'~!;~-o!<'t;;~'1!'::~';;,
'M"t,t~_.!i ~~~ ";>(".\.f."
ll<"l
'_~
~ .

5 grams of water at 30°C and 5 grams of ice at - 20°C are mixed


together in a calorimeter. Find the temperature of the mixture.
Water equivalent of calorimeter is negligible. .
(Specific heat of ice = 0.5 caVgmcoC,Latent heat of ice = 80 call
gm). ' (lIT 1977)
120"
:_
51 ..,,'~ t,:;~,,1;kf,.'-"
.•I,\ p.!!
.' :;d~~::1'~;,~'ll~{:
t~"".~":::;'I\'I'r~J:"(:"~:>r'"",~~'l'r~",,'!tL'T.';lJl
••(:r't -~~'."::"~)',~.,J:-;.
:.~ -''''- "::.~U::,~.
~':I~;_ ~~~.~):"90:,",'9\r:t:.;t~!",,~.1-;_i1

The temperatures of equal masses of'three different liquids A, B


and C are 12°C,19°Cand 28°C respectively. The temperature when
A and B are mixed is 16°C and when Band C are mixed it is 230C.
What will be temperature when A and C are mixed. (lIT 1976)
12061~f',~fJ;:3~~~r:!,:~f::,;:~~':!\~~~T
A vessel is filled completely with 500 gms of water and 1000 gms
',of mercury. When 21200 calories of heat are given to it water of
mass 3.52 gms overflows. Calculate the coefficient of volume
expansion of mercury. Expansion of vessel may be neglected.
(coefficienfofvolume expansion of water = 1.5 x lO--'/oC,
Density of mercury = 13.6gm/cc, ' .
Density ,ofwater = 1 gm/cc,
Specific heat of mercury = 0.03 caVgm_OC) (lIT 1976)
PRACTICE PROBLEMS ,239

12071 _
A mixture of 250 gm of water and 200 gm of ice at O°Cis kept in a
calorimeter which has a water equivalent of 50 gm, If 200 gm of
stream at 100°Cis passed through this mixture, calculate the final
temperature and weight of the contents of the calorimeter.
(lIT 1974)
!2081 _
A lead bullet strikes against a steel armour plate with a velocity of
300 mls. If the bullet falls dead after the impact, find the rise in
temperature of the bullet assuming that the heat produced is
shared equally between the bullet and the target. (Specificheat of
lead = 0.03 cal/gm_DC). (lIT 1974)
12091 _
An aluminium container ofmass 100 grams contains 200 grams of
ice at - 20°C. Heat is added to the system at the rate of 100calories
per second. What is the temperature ofthe system after 4 minutes.
Draw a rough sketch showing the variation of the temperature of
the system as a function of time ?
(Specific heat of ice =0.5 cal/gm_OC,
Specific heat of aluminium =0.2 cal/gm_OC,
Latent heat of fusion of ice =80 cal/gm) (lIT 1973)
12101 -_-_-_..-_-_-._'_-. _
In an industrial process 10 kg of water per hour is to be heated
from 20°C to BO°C.To do this, steam at 150°C is passed from a
boiler into a copper coil immersed in water. The steam condenses
in the coil and is returned to the boiler as water at 90°C. How
many kg of steam are required per hour? (lIT 1972)
(Specific heat of steam = 1 cal/gm,
Latent heat of steam = 540 cal/gm)

~~------
12111 T' ' - ".':

A uniform rod of length L, conductivity K is connected from


one end to a furnace at temperature T I' The other end of rod is at
temperature Toand is exposed to atmosphere. The temperature of
atmosphere is T•. The lateral part of rod is insulated. If
240 PRACTICE PROBLEMS

T2 - T, < < T" T2 = T, + .1.Tand Insulation


.1.T= (TI - T,), find proportionality
constant of given equation. The
heat loss to atmosphere is through
radiation only and the emissivity
of the rod is E. (lIT 2004)

Hot oil is circulated through an


insulated container with a
wooden lid at the top whose ~
conductivity K= 0.149 J/(m-OC
sec), thickness t = 5 mm,
emissivity = 0.6. Temperature
of the top of the lid is To
maintained at T) = 127°. If the Ta = 27°C
ambient temperature Ta = -Hot Oil
27°C, calculate
(a) rate of heat loss per unit area due to radiation from the lid.

(b) temperature of the oil. (Given IT 17 x 10-8)


= ""3 (I IT 2003)

1~~~I...H> .
A solid body X of heat capacity C is kept in an atmosphere whose
temperature is TA = 300 K At time t = 0 the temperature of X is To
= 400 K It cools according to Newton's law of cooling. At time tl'
its temperature is found to be 350 K
At this time (tl), the body X is connected to a large box Y at
atmospheric temperature TA, through a conducting rod oflength L,
cross-sectional area A and thermal conductivity K. The cross
sectional area A of the connecting rod is small compared to the
surface area of X. The heat capacity of Y is so large that any
variation in its temperature may be neglected. Find the
temperature of X at time t = 3tl" (lIT 1998)
~ ;.;.,.:.:.:.:.:.:.:.:.:.:.:.:-:.:.;.;.;.:.:.:.:.:-:.:.;.:.:.:-:.:.:.:.:.:.:.;.; .
~H>.
,

A double-pane window used for insulating a room thermally from


outside consists of two glass sheets each of area 1 m2 and thickness
0.01 in separated by a 0.05 m thick stagnant air space. In the steady
state, the room-glass interface and the glass-outdooF interface are
at constant temperatures of 27°C and O°Crespectively. Calculate
the rate of heat flow through the window pane. Also find the
temperatures of other interfaces. Given, thermal conductivities of
glass and air as 0.8 and 0.08 Wm- I K- I respectively.
(lIT 1997, May)
PRACTICE PROBLEMS • 241

12151 _

A cylindrical block of length 0.4 m and area of cross-section 0.04


m2is placed coaxially on a thin metal disc of mass 0.4 kg and the
same cross-section. The upper face of the cylinder is maintained
at a constant temperature of 400 K and the initial temperature of
the disc is 300 K. Ifthe thermal conductivity ofthe material ofthe
cylinder is 10 watt/m-K and the specificheat of the material of the
disc is 600 Jlkg.K, how Jongwill it take for the temperature of the
disc to increase to 350 K? Assume, for purposes of calculation, the
thermal conductivity ofthe disc to be very high and the system to
be thermally insulated except for the upper face of the cylinder.
(lIT 1992)
12161 _

An electric heater is used in a room of total wall area 137 m2 to


maintain a temperature of + 20°C inside it, when the outside
temperature is - 10°C. The walls have three different layers
materials. The innermost layer is of wood of thickness 2.5 em, the
middle layer is of cement of thickness 1.0 em and'the outermost
layer is of brick ofthickness 25.0 em. Find the power ofthe electric
heater. Assume that there is no heat loss through the floor •.nd the
ceiling. The thermal conductivities ofwood, cement and brick are
0.125, 1.5 and 1.0 watt/m/°C respectively. (lIT 1986)
12171 _

Three rods of material X and three


rods of material Yare connected as c
shown in the figure. All the rods are
ofidentical length and cross-sectional A B E
area. If the end A is maintained at y
600C and the ju nction E at 10°C.
Calculate the temperature of the D
junctionsB, C and D. The thermal
conductivity ofX is 0.92 caVsec-cm-oC
and that ofY is 0.46 caVsec-cm-oC. (lIT 1978)
12181 _
A room is maintained at 20°C by a heater of resistance of20 ohms
connected to 200 volts mains. The temperature is uniform
throughout the room and the heat is transmitted through a glass
window of area 1 m2 and thickness 0.2 em. Calculate the
242 PRACTICE PROBLEMS

temperature outside. Thermal conductivity of glass is 0.2 callm.


sec.oC and mechanical equivalent of heat is 4.2 Joules/cal.
(lIT 1978)
12191 _

A bar of copper oflength 75 cm and a bar of steel oflength 125 cm


are joined together end to end. Both are of circular cross-section
with diameter 2 cm. The free ends of copper and steel bars are
maintained at 100°C and O°Crespectively. The surfaces of bars
are thermally insulated. What is temperature of the
copper-steel junction. What is the heat transmitted per unit time
across the junction when the steady state has been reached.
(Thermal conductivity of copper is 9.2 x 10- 2 Kcallm-sec-oC and
that of steel is 1.1 x 10- 2 Kcallm-sec-OC) (lIT 1977)
12201,_; --_._ .... _. '-'_.'-' -"-"'--"-' ---_._ ... _-_ .. _.. _.. _--

Two rods A and B are of equal length. Each rod has its ends at
temperature T, and T2' What is the condition that will ensure equal
rates offlow of heat through the rods A and B. (lIT 1976)
12211'- .. .

A 2 metre long wire of resistance 40 and diameter 0.64 mm is


coated with plastic insulation ofthickness 0.06mm. When a current
.of5 amps. flows through the wire find the temperature difference
across the insulation in steady state. (Thermal conductivity of .
plastic = 1.6 x 10.3 callcm.s-OC). (lIT 1974)
222
1 1_' --------~-------~-
A slab of stone of area 3600 sq cm and thickness 10 cm is exposed
on the lower surface to steam at 100 C. A block of ice at O°Crests
D

on the upper surface of the slab. In one hour 4800 gm of ice is


melted. Calculate the thermal conductivity of the stone. (Latent
heat of ice = 80 callgm) . (lIT 1972)
12231 _

A closed cubical box ma,de of a perfectly insulating material has


walls of thickness 8 cm and the only way for heat to enter or leave
the box is through two solid, cylindrical, metallic plugs, each of
cross-sectional area 12 cm2.and length 8 cm fixed in opposite walls
ofthe box (see figure). The outer surface ofA ofone plug is kept at
100°Cwhile the outer surface ofB ofthe other plug is maintained
at 4°C. The thermal conductivity of the material of the plug is
PRACTICE PROBLEMS . 243

0.5 caVern-see-DC.A source of energy generecting 36 calories/sec is


enclosed inside the box. Find the equilibrium temperature of the
inner surface of the box assuming that it is the same at all poiIits
on the inner surface. (lIT 1972)
.
.................... .

........
0 ••••••••••

~---------~
12241--' .....
A cubical box of side 1 meter contains helium gas (atomic weight
4) ata pressure of 100N/m2• During an obserVationtime of! second,
an atom travel1ing with the root-mean-square speed parallel to
one of the edges of the cube, was found to make 500 hits with a
particular wall, without any collision with other atoms. Take
25
R; 3" J/mol-K and k; 1.38 x 10-23 JIK
. (a)Evaluate the temperature of the gas.
(b) Evaluate the average kinetic energy per atom.
(e) Evaluate the total mass of helium gas in the box. (lIT 2002)

12251_' ----------------
A closed container of volume 0.02 m' contains a mixture of neon
and argon gases, at a temperature of27°C and pressure of 1 x lOS
Nm-2. The total mass ofthe mixture is28 gm. If the gram molecular
weights of neon and argon are 20 and 40 respectively, find the
masses ofthe individual gases in the container, assuming them to
be ideal. (Universal gas constant ~ 8.314 J/mol. K) (lIT 1994)
12261_-_. _
A thin tube of uniform cross-section is sealed at both ends. It lies
horizontally, the middle 5 em containing mercury and the two equal
ends containing air at the same pressure P. When the tube is held
at an angle of 600 with the vertical direction, the length of the air
column above and below the mercury column are 46 em and 44.5
em respectively. Calculate the pressure Pin centimetres of mercury.
(The temperature of the system is kept at 30°C). (lIT 1986)
244 PRACTICE PROBLEr~S

1227 1_F__ ._"_.-_. "_. _._----' .__ ._.._------'--'


Two glass bulbs of equal volume are connected by a narrow tube
and are filled with a gas atO"C and a pressure of76 cm of mercury.
One of the bulbs is then placed in melting ice and the other is
placed in a water bath maintained at 62°C. What is the new value
of the pressure inside the bulbs? The volume of the connecting
tube is negligible. (lIT 1985)
12281r- - ~...~. -- --~....-- - - ...
A cyclic process ABCA shown in the V-T diagram (see figure) is
performed with a constant mass of an ideal gas. Show the same
process on a P- V diagram.
(In figure, CA is parallel to the V-axis and BC is parallel to the T-
axis) v (lIT 1981)
. Vt .•••... c ,B

VI ••••••• :A
T, T, T
~ r- --- ---- .__. --_.... --'_.._. j

Ajar contains a gas and a few drops of water at TOK The pressure
in the jar is 830 mm ofHg. The temperatue of the jar is reduced by
1%. The saturated vapour pressures of water at the two
temperatures are 30 and 25 mm ofHg. Calculate the new pressure
in the jar.' (lIT 1980)
r;;;w
~
r------=-,..,..~ ._-_._--~-
.. .' '. ..'
'>r- ----
,j

A column of mercury of 10 cm length is contained in the middle of


a narrow horizontal 1 m long tube which is closed at both the ends.
Both the halveS' of the tube contain air at a pressure of 76 cm of
mercury. By what distance will the column of mercury be displaced
if the tube is held vertically? (lIT 1978)
12311 r---'''~-''
-.-
--.
'---:-'""
-.- ~'l -l-'

Calculate the mass of 1 litre of moist air at 27°C when the barometer
reads 753.6 mm ofHg and the dew point is 16.1°C.
(Saturation vapour pressure of water at 16.1°C is equal to 13.6
mm ofHg, density of air atNTP is equal to 0.001293 glee, density
of saturated water vapour at NTP is equal to 0.000808 glee).
(lIT 1977)
PRACTICE PROBLEMS 2450

12321'_ ---------------
A glass capillary tube, sealed at both ends is 100 cm long, It lies
horizontally with the middle 10 cm containing m~rcury, The two
ends of the tube (which are equal in length) contain air at 27°C
and at a pressure 76 cm ofmercury. The tube is kept in a horizontal
position such that the air column at one end is at O°Cand the
other end is maintained at 127°C. Calculate the length ofthe air
column which is at O°Cand its pressure. Neglect the change in
volume of mercury and glass. (lIT 1975)
12331 _
An electric bulb ofvolume 250 ccwas sealed offduring manufacture
at pressure oflO-' mm ofHg at 27°C. Compute the number of air
molecules contained in the bulb. (lIT 1974)
12341 _
At the top of a mountain a thermometer reads 7°C and a barometer
reads 70 cm ofHg. At the bottom of the mountain they read 27°C
and 76 cm ofHg. Compare the density of air at the top with that at
the bottom. (lIT 1974)
12351 _
An ideal gas is trapped between a mercury column and the closed
lower end of a narrow 'lertical tube of uniform bore. The upper
end ofthe tube is open to the atmosphere (atmospheric pressure =
76 cm of mercury). The lengths of mercury and the trapped gas
columns are 20 cm and 43 cm respectively. What will be the length
of the gas column when the tube is tilted slowly in a ~ertical plan"-
through an angle of 60°. Assume the temperature to be constant.
(lIT 1972)
~i.~M~aiI _
12361 _

An ideal diatomic gas is enclosed h an


insulated chamber at temperature 300 K
Atmosphere
The chamber is closed by a freely movable
massless piston, whose initial height from
the base is 1m. Now the gas is heated such
that its temperature becomes 400 K at
constant pressure. Find the new height ofthe piston from the base.
If the gas is compressed to initial position such that no exchange
ofheat takes place, find the final temperature ofthe gas.(lIT 2004)
246 PRACTICEPROBLEMS

12371 _
An insulated container containing monoatomic gas of molar mass
m is moving with a velocity vo' If the container is suddenly stopped,
find the change in temperature. (lIT 2003)
12381 _
A monatomic ideal gas of two moles is
taken through a cyclicprocess starting
from A as shown in the figure. The

volume ratios are

A
(~:) =2and (~:)=4. 0 T T_ Tn

If the temperature TA at Ais 27°C, Calculate:


(aJ temperature of the gas at point B.
(bJ heat absorbed or released by the gas in each process.
(cJ the total work done by the gas during the complete cycle.
Express your answer in terms of the gas constant R. (lIT 2001)
12391 _

1\vo moles of an ideal monatomic gas p

is taken through a cycle ARCA as 2P, --lSJ" C


shown in the poTdiagram. During the :
process AB, pressure and :
temperature ofthe gas vary such that P, --~------- A
PT = constant. 1fT, = 300 K, calculate T, 21, T

(aJ the work done on the gas in the process AB and


(bJ the heat absorbed or released by the gas in each ofthe process.
Give answers in terms of the gas constant R. (lIT 2000)
12401 _

1\vo moles of an ideal monatomic gas, initially at pressure P, and


volume V" undergo an adiabatic compression until its volume is
V2. Then the gas is given heat Q at constant volume V2.
(aJ Sketch the complete process on ap-V diagram.
(bJ Find the total work done by the gas, the total change in its
internal energy and the final temperature of the gas.
(Give your answer in terms ofp" V" V2, Q and R) (lIT 1999)
PRACTICEPROBLEMS 247

One mole of an ideal monatomic


gas is taken round the cyclic P
process ABCA as shown in the
figure. Calculate
(a) the work done by the gas
(b) the heat rejected by the gas in
the path CA and the heat
absorbed by the gas in the path
Pc,
:~c
S',, ,,
,

ZV, v
AB.
(c) the net heat absorbed by the gas in the path BC.
(d) the maximum temperature attained by the gas during the cycle.
(lIT 1998)
I:M~I~.,'.""~~~~~~~~~~~~~~
One mole of a diatomic ideal gas (y = 1.4) is taken through a cyclic
process starting from point A. The process A -+ B is an adiabatic
compression, B -+ C is isobaric expansion, C -+ D an adiabatic
V
expansion and D -+ A is isochoric, The volume ratios are VA = 16
. B

and ~c = 2 and the temperature at Ais TA = 300 K. Calculate the


0

.tempe:ature ofthe gas at the points Band D and find the efficiency
of the cycle. (lIT 1997, July)

I:M~I
",,'
A --"'---"'---"'--=--"'--=-=--"'---"'--=-=-=-=~
sample of 2 kg of monatomic P .
Helium (assumed ideal) is taken
through the process ABC and
another sample of 2 kg of the
(l0' N/m')
10 BOC
same gas is taken through the 5. Ai Jl
process ADC (see figure). Given
molecular mass of Helium = 4. L~1';"0-~2i",O---V (m.')

(i) What is the temperature of Helium in each of the states A, B,


C andD?
(ii) Is there any way oftelling afterwards which sample ofHelium
went through the process ABC and which went through the
process ADC ? Write Yes or No.
(iii) How much is heat involved in each of the processes ABC and
ADC? (lIT 1997, May)
248 PRACTICE PROBLEMS

At 27' two moles of an ideal mono atomic gas occupy a volume V


The gas expands adiabatically to a volume 2V Calculate:
(i) the final temperature of the gas,
(ii) change in its internal energy, and
(iii) the work done by the gas during this process. (lIT 1996)

A gaseous mixture enclosed in a vessel of volume V consists of one


C 5
gram mole of a gas A with y = Cp = "3and another gas B with y =
v

~ at a certain temperature T. The gram molecular weights of the


gases A and Bare 4 and 32 respectively. The gases Aand B do not
react with each other and are assumed to be ideal. The gaseous
mixture follows the equation PV'9/'3 = constant, in adiabatic
processes.
(a) Find the number of gram moles of the gas B in the gaseous
mixture.
(b) Compute the speed of sound in the gaseous mixture at T =
300K
(c) If T is raised by 1 K from 300 K, find the percentage change
in the speed of sound in the gaseous mixture.
1
(d) The mixture is compressed adiabatically to 5" of its initial

volume V. Find the change in its adiabatic compressibility in


terrus of the given quantities. (lIT 1995)

An ideal gas is taken through a cyclic thermodynamic process


through four steps. The amounts of heat involved in these steps
are: Q, = 5960 J, Q2 = - 5585J, Q3 = -2980 J and Q. = 3645 J
respectively.
The corresponding works involved are:
W, = 2200 J, W2 = - 825 J, W3 = -1100 J, and W. respectively.
(i) Find the value ofW •. ,
(ii) What is the efficiency of the cyclic? (lIT 1994)
PRACTICE PROBLEMS 249

One mole of a monatomic ideal gas is taken through the cycle shown

DucB
in the figure, y
A ----> B : adiabatic expansion : A •
B ----> C : cooling at constant volume ~
C ----> D, adiabatic compression
D ----> A, heating at constant volume
I X
o V._----'JI>

The pressure and temperature at A, B, etc. are denoted by P A' PB,


TA, TB etc. respectively. Given that

TA= 1000 K, PB = (~)PA and Pc = (~)PA'


Calculate the following quantities,
(i) The work done by the gas in process A ----> B
(ii) The heat lost by the gas in process B ----> C

(iii) the temperature TD. [Given: (;)"15 = 0.85] (lIT 1993)

Two moles of helium gas


undergo a cyclic process as 2 atm ... At . ]B

~own t~n th: bfig:r~. .


sume egas 0 eI ea, latm ..... --~~-- '.C
calculate the following D,
quantities in this process: 300K 400K
(aJ The net change in the heat energy.
(b) The network done.
(c) The net change in internal energy. (lIT 1992)
1~4~I)n> .
. 7
Three moles of an Ideal gas (Cp = "2 R) at pressure, P A and
temperature TA is isothermally expanded to twice its initial volume.
It is then compressed at constant pressure to its original volume.
Finally gas is compressed at constant volume to its original
pressure PA.
(a) Sketch P-V and poT diagrams for the complete process.
250 PRACTICEPROBLEMS

(b) Calculate the net work done by the gas, and net heat aupplied
to the gas during the complete process. (lIT 1991)
12501 _
An ideal gas having initial pressure P, volume V and temperature
Tis aJlowedto expand adiabaticaJly until its volume becomes 5.66
T
V while its temperature faJls to 2"
(i). How many degrees offreedom do the gas molecules have?
(ii) Obtain the work done by the gas during the expansion as a
function of the initial pressure P and volume V. (lIT 1990)
12511 _

Twomoles ofhelium gas (y = %) are initiaJly at temperature 27°C


and occupy a volume of 20 litres. The gas is first expanded at
constant pressure until the volume is doubled. Then it undergoes
an adiabatic change until the temperature returns to its initial
value.
(i) Sketch the process on a p-V diagram,
(ii) What are the final volume and pressure of the gas?
(iii) What is the work done by the gas? (lIT 1988)
12521 ~
An ideal monatomic gas is confined in a cylinder by a spring-loaded
piston of cross-section 8.0 x 10-3 m2• InitiaJly the gas is at 300 K
and occupies a volume of 2.4 x 10- 3 m3 and the spring is in its
relaxed (unstretched, uncompressed) state, (see figure). The gas is
heated by a smaJl electric heater until the piston moves out slowly
by 0.1 m. Calculate the final temperature of the gas and the heat

Open at.mosphere

Healer 1 t:i-ft1~m_~g..
~ ....-Rigid
-r sUPllort
PRACTICE PROBLEMS 251

supplied (in joules) by the heater. The force constant of the spring
is 8000 N/m, atmospheric pressure is 1.0 x 105 Nm-2. The cylinder
and the piston are thermally insulated. The piston is massless and
there is no friction between the piston and the cylinder. Neglect
heat loss through the lead wires ofthe heater. The heat capacity of
the heater coil is negligible. Assume the spring to be massless.
(lIT 1989)
12531 _

An ideal gas has a specificheat at constant pressure Cp = 5: . The


gas is kept in a closed vessel ofvolume 0.0083 m3, ata temperature
of300 K and a pressure of 1.6 x 106 N/m2• An amount of 2.49 x 104
Joules of heat energy is supplied to the gas. Calculate the final
temperature and pressure ofthe gas. (lIT 1987)
12541 _

The rectangular box shown in Fig.


has a partition which can slide

~gvJt;]:E@=EI]
(y =~I at a pressure Po, volume
~o a;J temperature To' The chamber on the left is slowly heated

-~
by an electric heater. The walls of the box and the partition are
thermally insulated. Heat loss through the lead wires of the heater
is negligible. The gas in the left chamber expands pushing the
... until the final pressure In
partition . both chambers becomes 32 .
Determine
(i) the final temperature of the gas # each chamber
(ii) the work done by the gas in the right chamber. (lIT 1984)

12551'_ -----'----
One gram mole of oxygen at 27°C and one atmospheric pressure is
enclosed in a vessel.
(i) Assuming the molecules to be moving with V""8' find the
number of collisions per second which the molecules make
with one square metre area of the vessel wall.
252 . • PRACTICE PROBLEMS

(ii) The vessel is next thermally insulated and moved with a


constant speed vo' ;t is then suddenly stopped. The process
results in a rise of the temperature of the gas by l°C. Calculate
the speed vo' (y.;, = 1.41) (lIT 1983)

Calculate the work done when one mole of a perfect gas is


compressed adiabatically. The initial pressure and volume of the
gas are 105 N/m2 and 6litres respectively. The final volume of the
gas is 2litres. Molar ~pecific heat of the gas at constant volume is
3R
2 . (lIT 1982)

12571~r'_"_"'_-_'_'__ "_'_' _"'_''_" ~ '_"'~'_~_"__


An ideal gas is enclosed in a vertical cylindrical container and
supports a freely moving piston of mass M. The piston and the
cylillder have equal cross-sectional area A. Atmospheric pressure
is Po' a:ld when piston is in equilibrium, the volume of the gas is
Yo, The piston is now displaced slightly from its equilibrium
position. Assuming that system is completely isolated from its
surrounding, show that the piston executes simple harmonic motion
and find frequency of oscillations. (lIT 1981)

~--------~
1 1- .~- .--
258
Two infinitely large sheets h3.ving charge densities ", and "2
respectively (" > "2) are placed near each other separated by
distance 'd'. A charge 'Q' i,; placed ill between two'plates such that
there is no effect on charge distribution on plates. Now this charge
is moved at an angle of 45 with the homontal towards plate having
0

charge density "2 by distance 'a' (a < d). Find the work done by
electric field in the process. (lIT 2004)
1 1i' .... ......• - .. ~ .... __ •. u •• --- •.••••• _ •••••••• ; •••. "." •••••• ,

259
A positive POillt charge q is fixed at origin. A dipole with a dipole
~ \'
moment p is placed along the x-axis far away from the origin

with p pointing along positive x.axis. Find:


(a) the kinetic energy of the dipole when it reaches a distance d
from the origin, and
(b) the force experienced by the charge q at this moment. (lIT 2003)
PRACTICE PROBLEMS 253

12601 _

Eight point charges are placed at the corners of a cube of edge a as


shown in the figure. Find the work done in disassembling this
system of charges. (liT 2003)
+q ::'/.

+g
-q ,,
,,
+g'
-q __- - - - - - -.t ...
" -g
+9

1
2611 _

A small ball ofmass 2 x 10-3 kg having a charge of 1 j.lC is suspended


by a string oflength 0.8 m. Another identical ball having the same
charge is kept at the point of suspension. Determine the minimum
horizontal velocit.ywhich should be imparted to the lower ball so
that it can make complete revolution. (lIT 2001)
12621 _
The point charges + 8 j.lC, - 1 j.lC, - 1 j.lC, + 8 !-,C are fixed at the

points - J¥ m, - J% m, J% m and J¥
+ + m respectively on
the y-axis. A particle of mass 6 x 10-. kg and of charge + 0.1 j.lC
moves along the - x direction. Its speed at x = + co is va.
Find the least value ofvofor which the particle will cross the origin.
Find also the kinetic energy of the particle at the origin.
1
Assume that space is gravity free. Given 4" "0 = 9 x 109 Nm'/C'.
. (liT 2000)
[263] _
A non-conducting disc of radius a and uniform positive surface
charge density cris placed on the ground, with its axis vertical. A
particle of mass m and positive charge q is dropped, along the axis
of the disc, from a height H with zero initial velocity. The particle

hasqlm = 4£og .
cr
(a) Find the value ofH ifthe particle just reaches the disc.
(b) Sketch the potential energy ofthe particle as a function of its
height and find its equilibrium position. (lIT 1999)
254 PRACTICE PROBLEMS

12641 _

A conducting sphere 8, of radius r is attached to an insulating


handle. Another conducting sphere 82 of radius R is mounted on
an insulating stand. 82 is initially uncharged.
8 I is given a charge Q, brought into contact with 82, and removed.
81 is recharged such that the charge on it is again Q; and it is
again brought in to contact with 82 and removed. This procedure
is repeated n times.
(a) Find electrostatic energy of82 after n such contacts with 81•
(b) What is limiting value of this energy as n ~ oo? (lIT 1998)
12651 _

'!\vo isolated metallic solid spheres ofradii Rand 2R are charged


such that both of these have same charge density cr.The spheres
are located far away from each other, and connected by a thin
conducting wire. Find the new charge density on the bigger sphere.
(lIT 1996)
12661
------------------------
A circular ring of radius R with uniform positive charge density),.
per unit length is located in the y-z plane with its centre at the
origin 0. A particle of mass m and positive charge q is projected
from the pointP (R.J3, 0, 0) on the positive x-axis directly towards
0, with an initial speed v. Find the smallest (non zero) value ofthe
speed v such that the particle does not return to P. (lIT 1993)
12671.
-------------------------
(a) Acharge ofQ coulombis uniformly distributed over a spherical
volume of radius R metres. Obtain an expression for the
energy of the system.
(b) What will be the corresponding expression for the energy
needed to completely disassemble the planet earth against
the gravitational pull amongst its constituent particles?
Assume the earth to be a sphere of uniform mass density.
Calculate this energy, given the product ofthe mass and the
radius of the earth to be 2.5 x 1031 kg-m.
(e) If the same charge ofQ coul. as in part (a) above is given to a
spherical conductor of the same radius R, what will be the
energy of the system? (lIT 1992)
PRACTICE PROBLEMS 255

Two fixed charges - 2Q and Q are located at the points with


coordinates (- 3a, 0) and (+ 3a, 0) respectively in the x-y plane.
(a) Show that all points in the x-y plane where the electric
potential due to the two charges is zero, lie on a circle. Find
its radius and the location of its centre.
(b) Give the expression V(x) at a general point on the x-axis and
sketch the function V (x) on the whole x-axis.
(c) If a particle of charge + q starts from rest at the centre of the
circle, show by a short quantitative argument that the particle
eventually crosses the circle. Find its speed when it does so.
(lIT 1991)

Three concentric spherical metallic


shells A, Band C ofradii a, band c
(a < b < c) have surface charge
densities cr,- crand crrespectively.
(i) Find the potential ofthe three
shells A, Band C. C
(iiJ If the shells A and C are at
the same potential, obtain the
relation between the mdii a,
band c.
(lIT 1990)

A point particle of mass M is -----------.


attached to one end of a massless
rigid non-conducting rod of ---------. . .----. +'1
length L. Another point particle ...--=5!L • ~ E
of the same mass is attached to "'1 ~
the other end of the rod. The two -------~.
particles carry charges + q and •
_ q respectively.'Ibis arrangement
is held in a region of a uniform electric field E such that the rod
makes a small angle e (say of about 5 degrees) with the field
direction, (see figure). Find an expression for the minimum time
needed for the rod to become parallel to the field after it is set free.
(lIT 1989)
256 PRACTICE PROBLEMS
.. -, .•.. ~----',
Three particles, each of mass 1 gm and carrying a charge q, are
suspended from a common point by insulated massless strings,
each 100em long. If the particles are in equilibrium and are located
at the corners ofan equilateral triangle ofside length 3 em, calculate
the charge q on each particle. (Take g = 10 m/s2). (lIT 1988)
.... - -,_... . ~-,,-~

272
1 1_' ---------~-----~--
Three point charges q, 2q and 8q are to be placed on a 9 em long
straight line. Find the positions where the charges should be placed
such that the potential energy of this system is minimum. In this
situation, what is the electric field at the position of the charge q
due to the other two charges? (lIT 1987)
12731~ _
1\vofixed, equal, positive charges, each of
A +<J
magnitude 5 x 10- 5 coul. are located at
points A and B separated by a distance of o
6 m. An equal and opposite charge moves D c
towards them along the line COD, the
perpendicular bisector of the line AB. The B +<J
moving charge, when 'it reaches the point C at a distance of 4 m
from 0, has a kinetic energy of 4 joules. Calculate the distance of
the farthest point D which the negative charge will reach before
returning towards C. (lIT 1985)
... ~ ,- r- - - - ,~_ ..• ,.

A thin fixed ring of radius 1 metre has a positive charge 1 x 10-5


coulomb uniformly distributed over it. A particle of mass 0.9 gm
and having a negative charge of! x 10- 5 coulomb is placed on the
axis at a distance of 1 em from the centre of the ring. Show that
the motion of the negatively charged particle is approximately
simple harmonic. Calculate the time period of oscillations.
(lIT 1982)
. -." .,
12751
A pendulum of mass 80 mg and carrying a charge of 2 x 10- 8
coulomb is at restin a horizontal uniform electric field of20,000 VI
m. Find the tension in the thread of the pendulum in the thread of
the pendulum and the angle it makes with the verticaL
(g = 10 mls2) (lIT 1979)
PRACTICE PROBLEMS 2S7

12761
A rigid insulated wire frame, in the
form of right triangle ABC is set in a
vertical plane. Two beads of
equal masses m each carrying charges
q I and q. are connected by a chord of
h •
A


length I and can slide without friction B ~c
on the wires. Considering the case
when the beads are stationary,
determine:
(i) the angle n,
(i,i) the tension in the chord, and
(iii) the normal reactions on the beads.
If the chord is now cut, what are the values ofthe charges for
which the beads continue to remain stationary? (lIT 1978)
12771 _
Three charges each of value q, are placed at the corners of an
equilateral triangle. A fourth charge Q is placed at the centre of
the triangle
(i) If Q ~ - q, will the charges at the corners move towards centre
or flyaway from it.
(ii) For what value of Q will the charges remain stationary? In
this situation how much work is done in removing the charges to
infinity? (liT 1978)
12781 _
A simple pendulum consists ofa small sphere ofmass m suspended
by a thread oflength I. The sphere carries a positive charge q. The
pendulum is placed in an uniform electric field of strength E
directed vertically upwards. With what periods will the pendulum
oscillate if the electrostatic force acting on the sphere is less than
the gravitational force? (Assume that the oscillations are small).
(liT 1977)
12791 -'--_
Four charges +q, +q, -q, -q are placed respectively at the corners
A, B, C and D of a square with side a arranged in the given order.
Calculate the electric potential and intensity at 0, the centre of
the square. If E and F are the mid points of the sides BC and CD,
258 PRACTICE PROBLEMS

respectively, what will be work done in carrying a charge e from 0


to E and from 0 to F. (lIT 1977)
12801 ~ _

A particle of mass 9 x 10-31 kg and a negative charge of 1.6 x 10-19


coulomb is projected horizontally with a velocity of lOGmlsec into
a region between two infinite horizontal plates of meta!.
The distance between plates is 0.3 cm and the particle enters 0.1
cm below the top plate. The top and bottom plates are connected
respectively to the positive and negative terminals of a 30 volt
battery. Find the components of velocity of the particle just before
it hits one ofthe plates. (lIT 1977)
1281 1, _

A charge + Q is fixed at a distance d in front of an infinite metal


plate. Draw the lines offorce indicating the directions clearly.
(lIT 1976)
12821 _
1\vo hollo',v conductors are charged positively. The smaller is at 50
V and the bigger is at 100 V potentia!. How should they be arranged
such that the charges flow from the smaller to the bigger conductor
when connected by a wire? (lIT 1976)
12831 _
Two identically charged spheres are suspended by strings of equal
lengths. The strings make an angle of 30° with each other. When
suspended in a liquid of density 0.8 gmlcc the angle remains same,
what is the dielectric constant of the liquid? The density of the
material of the spheres is 1.6 gm/cc. (lIT 1976)
12841 _

A particle having a charge of 1.6 x 10-19 coulombs enter midway


between the plates of a parallel plate condenser. The initial
velocity of the particle is parallel to the plane of the plates. A
potential difference of 300 V is applied to the capacitor plates. If
the length of the condenser plates be 10 cm and the condenser
plates be separated by a distance of 2 cm, calculate the greatest
initial velocity for which the particle will not be able to come
out of the condenser plates. The mass of the charged particle is
12 x 10- 24 kg. (lIT 1976)
PRACTICE PROBLEMS 259

12851 _

Two identical charges + Q each, are at a distance r from each other.


A third charge q is placed on the line joining the above two charges
such that all the three charges are in equilibrium. What is the
magnitude, sign and position of the charge q? (lIT 1975)
12861 _
Two positive charges Q and 4 Q are fIxed at a distance of 12 cm
from each other. Sketch the lines of force and locate the neutral
points if any. (lIT 1975)
12871 ~ _
A particle of mass 40 milligrams and carrying a charge of5 x 10-9
coulombs is moving directly towards a fIxed positive point charge
of magnitude 10- 8 coulombs. When it is at a distance of 10 cm
from the fixed positive chrge it has a velocity of 50 cm/s. At what
distance from the fixed charge will the particle come to momentarily
to rest? Is the acceleration constant during motion? (lIT 1975)
12881__ ~ _
A positive charge + Q is located at a point. What is work done if a
unit positive charge is carried once completely around this charge
along a circle of radius r about this point? (lIT 1974)
12891 _

A spark is produced between two insulated surfaces maintained


at a potential difference of 5 x 10" volts. If the energy output is
10-5 Joules, calculate the charge transferred during the spark.
(lIT 1974)

12901 _

'l\vo point charges of value - 20 eSll and + 20 esu are placed on the
x-axis at x = - 10 cm and x = + 10 cm respectively. Calculate.
(i) potential, and
(ii) electric field at the point P (x = O,y'= 10 cm) and Q (x = 20 cm,
)' = 0).
(iii) Find the work done in carrying a unit positive charge from P
to Q along a straight line.
(iv) Is there any path along which the work done is less than the
above value? (3 x 109 esu = 1 coulomb) (lIT 1974)
260 PRACTICE PROBLEMS

12911 ~ _
A positively charged oil droplet remains stationary in the electric
field between two horizontal parallel plates separated by a distance
ofl em. If the charge on the.drop is 96 x 10-10 esu and the mass of
the droplet is 10- 11 gm, what is the potential difference between
the plates?
Now if the polarity of the plates is reversed, what is the
instantaneous acceleration of the droplet?
(3 x 109 esu = 1 coulomb, g = 9.8 mJs2) (lIT 1974)
12921 _
Two copper spheres of the same radii, one hollow and the other
solid are charged to the same potential. Which, if any, of the two
will hold more charge? (lIT 1974)
12931 _
An infinite number of charges each equal to q are placed along the
x-axis at x:::: a, x:::: 2a x:::: 4a, x:::: 8a, ... and s,oon.
l

(i) Find the potential and electric field at the point x = 0 due to
this set of charges.
(ii) What will be potential and electric field if, in the above set
up, the consecutive charges have opposite sign?
(lIT 1974)
12941 ~ _
A spherical liquid drop has a diameter 2 mm and is given a charge
of 5 x 10- 6 esu.
(i) What is the potential at the surface of the drop?
(ii) Iftwo such drops coalesce to form a single drop, what is the
potential at the surface of the drop so formed?
(3 x 109 esu = 1 coulomb) (lIT 1973)
12951 _

A pith ball carrying a charge of 1 esu is suspended by an insulated


thread oflength 50 em. When a uniform electric field is applied in
a horizontal direction the ball is found to deflect by 2 em from
vertical. If the mass of the ball is 0.5 gram, what is the magnitude
and direction of the electric field?
(g = 980 cmJs2, 3 x 109 esu = 1 coulomb) (lIT 1973)
PRACTICE PROBLEMS 261

12961_ ~--------------
Twopoint charges, one of + 100 esu and other of- 400 esu are kept
30cm apart.
(a) Find the points ofzero potential on the line joining the charges.
(Assume the potential at infinity to be zero).
(b) Are there any other points of zero potential in the
neighbourhood of the charges.
(c) Find the points where the electric field due to these two
charges is zero. (lIT 1972)
12971 _

At the corner A of a square ABCD of side 10 cm is placed a charge


+ 200 esu. Another charge -100 esu is located at the centre of the
square. Find the work done in carrying a charge + 15 esu from the
corner C to the corner B of the square.
(3x 109 esu = 1 coulomb) (lIT 1972)

~~------
12981 _

Two capacitors A and B with capacities 3flf' and 2flf' are charged
to a potential difference of 100Vand 180Vrespectively. The plates
of the capacitors are connected as shown in the figure with one
wire from each capacitor free. The upper plate of A is
positive and that ofB is negative. An uncharged 211Fcapacitor C
with lead wires falls on the free ends to complete the circuit.
Calculate :

(i) the final charge on the three capacitors, and


(ii) the amount ofelectrostatic energy stored in the system before
and after the completion of the circu.it.
(lIT 1997, July)
262. PRACTICEPROBLEMS

12991 _

The capacitance of a parallel plate capacitor with plate area A and


separation d is C. The space between the plates is filled witl, two
wedges of dielectric constants K1 and K" respectively (Figure).
Find the capacitance ofthe resulting capacitor. (lIT 1996)

~III
I T
=K, l' d

1
300
1 1_' ---------------
Two parallel plate capacitors A and B have tile same separation
d = 8.85 x 10-4 m between tile plates. The plate area ofA and Bare
0.04 m2 and 0.02 m2 respectively. A slab of dielectric constant
(relative permittivity) K = 9 has dimensions such tilat it can exactly
fill tile plates of capacitor B.

A 1 A
B

<aJ
T
(b) (e)

(i) The dielectric slab is placed inside A as shown in figure A is


then charged to a potential difference of 110V.Calculate the
capacitance of A and tile energy stored in it.
(ii) The battery is disconnected and tilen tile dielectric slab is
removed from A. Find .tile work done by the external agency
in removing.the slab from A.
(iii) The same dielectric slab is now placed inside B, filling
it completely. The two capacitors A and B are then
connected as shown in tile figure. Calculate tile energy stored
in tile system.'
(lIT 1993)
'.
PRACTICE PROBLEMS . - .263

I*~tl»" '
The Fig. shows two identical
parallel plate capacitors connected
to a battery with the switch S
closed. The switch is now openerl
and the free space between the
plates of the capacitors is filled
with a dielectric of dielectric
constant (or relative permittivity) 3. Find the ratio of the total
electrostatic energy stored in both capacitors before and after the
introduction ofthe dielectric. (lIT 1983)

From the given figure, find the value of capacitance C if the


equivalent capacitance between points A and B to be IJlF. All the
capacitances are in JlF. (lIT 1977)
lJ.ll"
A~h--11I l
c,l 6~FT. 4~F
R~~'
h '-1
12~lF

2~F 2~F
B

I~Q~I?U>
Two metal plates form a parallel plate condenser. The distance
d
between the plates is d. A metal sheet of thickness '2 and'of the
same area is inserted completely between the plates. What is the "
ratio of the capacitances in the two cases. (lIT 1976).""""

mmm:iiI"',':n:nu:nn,/ut
I~Q41>
The two batteries A ant: B.

~~l~;~1f~;~~:~i
~:~~~:~R:~
:~0::::~:
switch K is closed at t = O.
After long time it was K
~-- _
found that terminal potential difference across the battery A is
zero. Find the value of R. (lIT 2004)
264 PRACTICE PROBLEr,IS

13051 ._._.,._.,_. -' --'-' -_.-


A thin uniform wire AB of length 1 m, an unknown resistance X
and a resistance of 12 Q are connected by thick conducting strips,
as shown in the figure. A battery and a galvanometer (with a sliding
jockey connected to it) are also available. Connections are to be
made to measure the unknown resistance X using the principle of
Whetstone bridge. Answer the following questions .
. :¥. .. __ - ::w.7."

B C
(a) Are there positive and negative on the
galvanometer?
(b) Copy the figure in your answer book and show the battery
and the galvanometer (with jockey) connected at appropriate
points.
(e) After appropriate connections are made, it is found that no
deflection takes place in the galvanometer when the sliding
jockey touches the wire at a distance of60 cm. from A Obtain
the value of the resistance X. (lIT 2002)
13061~. _
In the circuit shown in the figure,
the battery is an ideal one, with
r'
VI 5 I! n
1
r
emf V, the capacitor is initially R
uncharged. The switch S is closed
at time t = O. A y"nrc
R R
(a) Find the charge Q on the capacitor at time t.
(b) Find the current in AB at time t. What is its limiting value as
t -> «> ? (lIT 1998)
13071 _

j..
Find the emf (V) and internal
resistance (r) of a single battery "
which is equivalent to a parallel •
combination of two batteries of A

emfs V, and V2 and internal


V,
resistance r I and r 2 respectively.
with polarities as shown in Fig. (lIT 1997, May)
. , ~
'.
PRACTICE PROBLEr.1S 265

13081_.'. _
A leaky parallel plate capacitor is filled completely with a
material having dielectric constant K= 5 and electrical conductivity
Ij = 7.4 x 10-12 n- I m- I, If the charge on the plate at the instant

t = 0 is q = 8.85 ",C,then calculate the leakage current at the instant


t = 12 s. (lIT 1997, May)

An electrical circuit is shown in the figure. Calculate the potential


difference across the resistor of 400 ohm, as will be measured by
the voltmeter V ofresistance 400 ohm, either by applying Kirchoff's
rules or ohtherwise. (lIT 1996)
¥

1000

loon
lOY

131011- U

' --~-,
-~ ....--- -_ .... ~ o~. ~ ----~-~

Two square metallic plates of 1 m side are kept 0.01 m apart, like
a parallel plate capacitor, in air in such Ii way that one of their
edges is perpendicular to an oil surface in a tank filled with an
insulating oiLThe plates are connected to a battery of emf 500 V.
The plates are then lowered vertically into the oil at a speed of
0.001 ms-I. Calculate the current drawn from the battery during
the process. (dielectric constant of oil = 11, "0 = 8.85 x 10- 12 C'
N-I m-") (lIT 1994)
13111,-;-_
In the given circuit,
EI = 3Eo =2E. =6volts.~ = 2R. = 60, Ra=2Ro= 4n, C= 5 ~
Find the current in Ra and the energy stored in the capacitor.
R. E, (lIT 1988)
c


66 '. '. PRACTICE PROBLEMS

1312! _
An infinite ladder network of resistances is constructed with 1
ohm' and 2 ohm resistance, as shown in the figure.
The 6 volt battery bet.ween A and B has negligible internal
resistance.
(~-----

BLUrtm_
(i) Show that the effective resistance between A and B is 2 ohm.
(ii) What is the current that passes through the 2 ohm resistance
nearest to the battery? (lIT 1987)
13131~. _
A part of circuit in a steady state along with the currents flowing
in the branches, the values ofresistances etc. is shown in the figure.
Calculate the energy stored in the capacitor C (4f!F). (liT 1986)
lA
3n
~ 3n 5n
2Alnp
C
411F' In
In
~
2Amp 2n 4n
3n
lAmp

1
314
1_, -------~-------

In the circuit shown in the figure, E, F,


G, H, are cells of emf 2, 1, 3 and 1 volt
Ar: ~'- B
respectively,

respectively.
Calculate:
and their internal
resistances are 2, 1, 3 and 1 ohm
F JH+

-
2n -

(i) the potential difference between B


u (;
and D,and G +

(ii) the potential difference across the terminals of each cells G


and H. (lIT 1984)
PRACTICEPROBLEMS .267

13151-::-:-~---:-_---:- ---:----=-
Calculate the steady state current in the 2-ohm resistor shown
in the circuit in the figure. The internal resistance of the battery
is negligible and the capacitance of the condenser C is
0.2 microfarad. 2n

an
I,e ..4n
I'

,
'-6\1 2.80
(lIT 1982)

13161 _
Two resistors, 400 ohms, and 800 ohms are connected in series
with a 6-volt battery. It is desired to measure the current in the
circuit. An ammeter of 10 ohms resistance is used for this purpose.
What will be the reading in the ammeter? Similarly, if a voltmeter
of 10,000 ohms resistance is used to measure the potential
difference across the 400 ohm resistor, what will be the reading in
the voltmeter.
(lIT 1982)
13171__ ~_~ ~ __ ~_
In the circuit s!'own in Fig. E, = 3 volts, E = 2 volts, E =
2 3 1 volt
and R = r 1 = r 2 = r 3 = 1 ohm.
"
A R
" .~
+ _

E,
B

" + -
E,
(i) Find the potential difference between the points A and Band
. the currents through each branch.
(ii) Ifr2 is short circuited and the point A is connected to point B,
find ilie currents through E" E2, E3 and the resistor R. >-

(IIT 1981)
268

13181
(i) State Ohm's Law.
PRACTICEPROBLEMS

..
_
........
I
3000 400n
(ii) In the circuit shown in figure,
a voltmeter reads 30 volts when

----~
it is connected across 400 ohm
resistance. Calculate what the GOV
same voltmeter will read when '-----
it is connected across the 300 ohm resistance. (lIT 1980)
13191 _

A battery of emf 2 volts and internal resistance 0.1 ohm is being


charged with a current of 5 amps.
In what direction will the current flow inside the battery?
What is the potential difference between the two terminals of the
battery? (lIT 1980)
13201 _

A copper wire having cross-sectional area 0.5 mm' and a length of


0.1 m is initially at 25°C and is thermally insulated from
surrounding. If a current of 10 amperes is set up in this wire.
(i) Find the time in which the wire will start melting. The change
of resistance with the temperature of the wire may be
neglected.
(ii) What will this time be, if the length of wire is doubled.
(For copper: resistivity = 16 x 10-9 nom, density = 8.9gm/cc,
specific heat = 390 J/kg-K)
Melting point = 1l00°C (lIT 1979)
13211 _

In the diagram shown below,find the potential difference between

state. TT
the points A and B and between the points Band C in steady
I F (lIT 1979)

3~F I~F

tOn

zon 100V c
PRACTICE PROBLEMS 269

13221 _

All the resistances in the diagram are in ohms.


Find the effective resistance between the points A and B.
3n

3n
liT 1979)
13231 _
A heater is designed to operate
with a power oflOOOwatts in a
:;:~
"tfAWh\
Ion
100 volt line. It is connected in
a combination with a resistance ••IN."
of 10 ohms and a resistance R
to a 100 volts mains as shown
-----~. ~
100V
-

in the figure. What should be


the value of R so that the heater operates with a power of 62.5
watts. (lIT 1978)
13241 _
If a copper wire is stretched to make it 0.1% longer what is the
percentage change in its resistance? (liT 1978)
13251 _
Two electric bulbs, each designed to operate with a power of 500
watts in a 220 volt line are put in a series in a llu volt line. What
will be the power generated by each bulb. (liT 1977)
13261 --'~ _
A potential difference of 220 volts is

fb
maintained across a 12000 O-rheostat
ab (see figure). The voltmeter V
has a resistance of 6000 0 and point C
is at one fourth of the distance from a •
to b. What is the reading in the
270 . PRACTICE PROBLEMS

voltmeter. (lIT 1977)


13271_. _
A potentiometer wire oflength 100cms has a resistance of 10 ohms.
It is connected in series with a resistance R and a cell of emf 2
volts and of negligible internal resistance. A source of emf 10
millivolts is balanced against a length of 40 cms of potentiometer
wire. What is the value of external resistance R? (lIT 1976)
13281_. ~ _
Five resistances are connected as shown in the diagram. Find the
equivalent resistance between points A and B. (lIT 1976)
m

3Q SQ

A 13

13291 _

Five resistances are connected as shown in the diagram. What is


effective resistance betwen the points A and B? (lIT 1976)

A 13

13301 .._.,_

12 cells each having the same emf are connected in series and are
kept in a closed box. Some ofthe cells are wrongly connected. This
battery is connected in series with an ammeter and two cells
identical with the others. The current is 3 amperes when the cells
and battery aid each other and is 2 amperes when the cells ahd
battery oppose each other. How many cells in battery are wrongly
connected?
(liT 1976)

13311
A fuse made of lead wire has an area ot-cross-section 0.2 sq. inm.
on short circuiting, the current in the fuse wire reaches 30 amps.
PRACTICE PROBLEMS 271

How long after short circuiting will the fuse begin to melt?
Initial temperature of the wire is 20'C. Neglect the heat losses.
(For lead: specific heat = 0.032 caVgm.'C, melting point = 327 'C,
density = 11.34 gm/cc, resistivity = 22 x 10- 6 ohm-em) (lIT 1976)

1~~~I>' .
A battery of emf 1.4 volts and internal resistance 2 Q is connected
to a resistor of 100 Q resistance through an ammeter. The resistance
of ammeter is (4/3) ohm. A voltmeter has also been connected to
find the Fotential difference across the resistance.
(i) Draw the circuit diagram.
(ii) The ammeter reads 0.02 amp. What is the resistance of
voltmeter?
(iii) The voltmeter reads 1.10 volts. What is the error in this
reading? (lIT 1975)
I~~~I
,...»,., ....
An electric tea kettle has two heating coils. When one of the coils
is switched Oel the kettle begins to boil in 6 minutes, and when the
other is switched on the boiling begins in 8 minutes. In what time
will the boiling begin if both coils are switched on simultaneously
(i) in series
(ii) in parallel? (lIT 1975)

An electric current of 5 amps is divided into three branches forming


a parallel combination. The lengths of the wire in the three
branches are in the ratio 2, 3 and 4; their diameters are in the
ratio 3, 4 and 5. Find the currents in each branch if the wires are
of the same material. (IIT 1975)
I~~~I.,'..
•••.••. ,','".,
.....
In the circuit shown the 511resistance develops 10.24 calories!
second due to current flowing through it. Calculate
(i) the heat developed per second in 2 Q resistor and
(ii) the potential difference across the 6 Q resistance. (lIT 1974)

50
272. PRACTICE PROBlEr,IS

Two heater coils made of the same material are connected in


parallel across the mains. The length and diameter of the wire of
one of the coils is double that of the other. Which one of them will
produce more heat? (lIT 1973)
1337It-~J;:;~~~~~~M~~~!~~!]~?K~Q~ti
A standard 50 watt electric bulb in series with a room heater is
connected across the mains. IT 50 watt bulb is replaced by 100
watt bulb, will the heater output be larger, smaller or remain the
same? (lIT 1973)

In the circuit shown in figure, the cells E. and E2 have emfs 4V


and BV and internal resistances 0.5 nand 1 n respectively.
Calculate the current in each resistor and the potential difference
across each cell.

ao
" (lIT 1973)

A galvanometer together with an unknown resistance in series is


connected across two identical batteries each of 1.5 volts. When
the batteries are connected in series the galvanometer records a
current of I' ampere and when the battereies aTe in parallel the
current is 0.6 ampere, what is the internal resistance ofthe battery?
• • (lIT 1973)
r~ I
•7't<""',""'!l~~~'~.:l~.;;~?~:t\ro"lf.~'i\;J~(.$'~~t
1340 l;.~ .•
~,,.,..Ijii;~'-<1;.:_'.T.1f'_~_,~
.
_.,,,.,,
.••JJ.1~.~
.
L-.~- .o\;.'£~.l\o,.•~~ --\!trjt:-'O'l.Jf~~Ii'..£:.l<~, ~~t::'\';..;J:
~lili",

An electric kettle 500 watts raises the temperature of one litre of


water from 25.C to 100.C in 15 minutes. What percimtage of
electrical energy supplied is utilised in heatingthewater?(lIT 1973)
13411~W~4~~~~~~1,~~~1mil~~1
Three equal resistors connected in series across a source of emf
together dissipate 10 watts of power. What would be the power
dissipated if the same resistors are connected in parallel across
the same source of emf? (lIT 1972)
PRACTICE PROSlEr,IS 273

~-------------
A galvanometer having a coil resistance of 100 n gives a full
scale deflectionwhen a current of 1milliampere is passed through
it.
(i) What is the value of the resistance which can convert this
galvanometer into an ammeter giving a full scale deflection
of 10 amperes?
(ii) A resistance ofthe required value is available but it will get
burnt if the energy dissipated in it is greater than 1 watt.
Can it be used for the above described conversion of the
galvanometer?
(iii) When this modified galvanometer is connected across the
terminals of a battery, it shows a current of 4 amperes. The
current drops to 1 ampere, when a resistance of 1.5 n is
connected in series with the modifiedgalvanometer. Find the
emf and the internal resistance ofthe battery. (lIT 1972)

~----------
13431_. _

A protGDand an alpha particle, after being accelerated through


same potential difference, enter a uniform magnetic field the
direction of which is peipendicu lar to their velocities. Find
the ratio ofradii of the circular paths of the two particles.
(lIT 2004)
13441 _
A ring of radius R having
uniformly distributed charge Q
is mounted on a rod suspended D
by two identical strings. The
tension in strings in
equilibrium is To. Now a
vertical magnetic field is
switched on and ring is rotated
at constant angular velocity (i).

Find the maximum (i)with


which the ring can be rotated if the strings can withstand a
. . 3To
maxnnum tensIOnof 2 . (lIT 2003)
274 PRACTICE PROBLEMS

3451_.
1 -----------------------
A rectangular loop PQRS made
from a uniform wire has length a, z
width b and mass m. It is free to
rotate about the arm PQ, which
remains hinged along a horizontal p y
Q
line taken as the y-axis (seefigure).
Take the vertically upward
direction as the Z-axis. A uniform
magnetic field B
= (3;+4k) Bo
Y
exists in the region. The loop is
held in the x-y plane and a current I is passed through it. The loop
is now released and is found to stay in the horizontal position in
equilibrium. (lIT 2002)
(a) What is the direction of the current I in PQ ?
(b) Find the magnetic force on the arm RS.
(c) Find the expression for I in terms ofBo' a, band m.

13461 ~ __
Acurrent of 10Aflowsaround
a closed path in a circuit
which is in the horizontal
plane as shown in the figure.
The circuit consists of eight
alternating arcs of radii
r, = 0.08 m and r2 = 0.12 m.
Each arc subtends the same
angle at the centre.
(a) Find the magnetic field
produced by this circuit
at the centre.
(b) An infinitely long straight wire carrying a current of 10 A is
passing through the centre ofthe above circuit vertically with
the direction of the current being into the plane of the
circuit.
What is the force acting on the wire at the centre due to the current
in the circuit? What is the force acting on the arc AC and the
straight segment CD due to the current at the centre?
(liT 2001)
PRACTICE PROBLEMS 275

A circular loop of radius R is bent y


along a diameter and given a shape
as shown in the figure. One the
semicircle (KNM) lies in the x-z ---
plane and the other one (KLM) in .-/ x
the y-z plane with their centres at
the origin. Current I is flowing K
through each of the semicircles as
shown in figure.
(a) A particle of charge q is released at the origin with a velocity
v = - Vo i. Find the instantaneous force 1 on the particle.
Assume that space is gravity free.
(b) If an external uniform magnetic field Bj is applied, determine
the forces F, and F2 on the semicircles KLM and KNM due

to this field and the net force F on the loop. (lIT 2000)

The region between x = 0 and x = L is' filled with uniform, steady


magnetic field Bo k . A particle of mass m, positive charge q and
velocity Vo i travels along x-axis and enters the region of the
magnetic field. Neglect the gravity throughout the question.
(a) Find the value ofL if the particle emerges from the region of
magnetic field with its final velocity at an angle 30° to its
initial velocity.
(b) Find the final velocity of the particle and the time spent by it
in the magnetic field, if the magnetic field now extends upto
Z.IL. (IIT 1999)
134~1.;i
A particle of mass m and charge q is moving in a region where
~ ~
uniform, constant electric and magnetic. fields E and Bare
~' ~
present. E and B are parallel to each other. At time t = 0 the
~ ~
velocity Vo of the particle is perpendicular to E . (Assume that its
speed is always« c, the speed oflight in vacuum.) Find the velocity
276 PRACTICE PROBLEMS

-.
V of the particle at time t. You must express your answer in terms
-> -> ->
of t, q, m. the vectors Vo , E and B , and their magnitudes vo' E
and B. (lIT 1998)

A uniform, constant magnetic


-> y
field B is directed at an angle of / 10
45° to the x-axis in the xy-plane. S R

PQRS is a rigid, square wire frame


carrying a steady current 10,
/ / /
with its centre at the origin O.
x
At time t = 0, the frame is at rest / / /
in the position shown in the P Q
figure, with its sides parallel
to the x and y axes. Each side of
the frame is of mass M and
length L.

(a) What is the torque 1about 0 acting on the frame due to the
magnetic field?
(b) Find the angle by which the frame rotates under the action of
this torque in a short interval oftime I'>.t, and the axis about
which this rotation occurs. (I'>.
t is so short that any variation
in the torque during this interval may be neglected.)
Given: moment of inertia ofthe frame about an axis through
4
its centre perpendicular to its plane is "3 MI:? (lIT 1998)

Three infinitely long thin wires, each carrying current I in the


same direction are inx-y plane of a gravity free space. The central
wire is along the y-axis while the other two are along x = :t d.
(i) Find the locus of the points for which the magnetic field B is
zero.
(ii) If the central wire is displaced along the z-direction by a small
amount and released, show that it will execute simple
harmonic motion. If the linear density of the wires is ", find
the frequency of oscillation. (lIT 1997, July)

,.
PRACTICE PROBLEMS . 277

13521 --;- _

A long horizontal wire AB, which is free to move in a vertical


plane and carries a steady current of 20 A, is in equilibrium at a
height of 0.01 m over another parallel long wire CD, which is
fixed in a horizontal plane and carries a steady current of 30A, as
shown in the figure. Show that when AB is slightly depressed, it
executes simple harmonic motion. Find the period of oscillations.
A B
\
C D (lIT 1994)
13531_- . "

An electron gun G emits electrons


of energy 2 keV travelling in the
positive x-direction. The electrons
are required to hit the spot S where
GS = 0.1 m, and the line GS makes
and angle of 60 with the x-axis,
0

as shown in the figure. A uniform


-,
magnetic field B parallel to GS
exists in the region outside the
electron gun. Find the minimum
value of B needed to make the electrons hit S. (lIT 1993)
13541 _

A straight segment OC (oflength L metre) of a circuit carrying a


current 1 amp is placed along the x-axis see figure. Two infinitely
along straight wires A and B, each extending from z = - "" to + "",
are fixed at y = - a metre and y = + a metre respectively, as shown
in the figure. Ifthe wires A and B each carry a current I amp into
the plane of the pa))er, obtain the expression for the force acting
on the-segment OC. What will be the force on OC if the current in
the wire B is reversed?
y

c x

z
(lIT 1992)

I-zrl" - ...-
278 PRACTICE PROBL'EMS

A wire loop carrying a current I is placed in the x-y plane as shown

YL
in the figure.

o x

(a) If a particle with charge + Q and mass m is placed at the


~
centre P and given a velocity v along NP (see figure), find its
instantaneous acceleration.
~ A

(b) If an external uniform magnetic induction field B = Bi is


applied, find the force and the torque acting on the loop due
to this field.
(lIT 1991)

Two long parallel wires


carrying currents 2.5 amperes
and I ampere in the same p Q
x
direction (directed into the
2.5 Am' ~I Amp ~
plane of the paper) are held : :+- 2m--:
at P and Q respectively such ~:f:---5Ill ":
that they are perpendicular
to the plane of. paper.
The point P and Q are located at a distance of 5 metres
and 2 metres respectively from a collinear point R (see figure).
(i) An electron moving with a velocity of 4 x 105 mls along the
positive x-direction experiences a force of magnitude 3.2 x
10- 20 N at the point R. Find the value of 1.
(ii) Find all the positions at which a third long parallel
wire carrying a current of magnitude 2.5 amperes may
be placed so that the magnetic induction at R is zero.
(lIT 1990)

. '"
PRACTICE PROBLEMS . . 279

13571 _

A pair of stationary and infinitely long bent wires are placed in


the XYplane ofas shown in the figure. The wires carry currents of
i = 10 amperes each as shown. The segments Land M are along
the X-axis. The segments P and Q are parallel to the Y-axis such
that OS = OR = 0.02 m. Find the magnitude and direction of the
magnetic induction at the origin O.
w~
,, T',.
,
, I

,,
:
, Q
,
", LR: 1\1 t~

i 0: s----"-----X
,,
p
,,
,,
,
,,
i , :,
-~)
(111' 1989)

13581 _

'!\vo long straight parallel wires


A
are 2 metres a part, perpendicular
to the plane of the paper see
figure).
The wire A carries a current of9.6
amps, directed into the plane ofthe B
paper. The wire B carries a current !J!M
11
such that the magnetic field of
p
induction at the point P, at a
distance of U10 m from +th.e wire B

is zero. Find
(i) the magnitude and direction of current in B.
(ii) the magnitude oftlie magnetic field of induction at the point
S.
(iii) the force per unit length on the wire B. (lIT 1987)
13591 _
A beam of protons with a velocity 4 x 105 mlsec. enters a uniform
magnetic field of 0.3 tesla at an angle of 60° to the magnetic field.
Find the radius ofthe helical path taken by the proton beam. Also
280 PRACTICE PROBLEMS

find the pitch of the helix (which is the distance travelled by a


proton in the beam parallel to the magnetic field during one period
of rotation). (lIT 1986)
: ':..... .• . ....
l
13601
A particle of mass m = 1.6 x 10- 27 x x
x x.
kg and charge q = 1.6 x 10- 19 C x
enters a region of uniform x x
x x
magnetic field of strength 1 Tesla
along the direction shown in the
E
x
• x x
figure. The speed of the particle is x x
x x
7
10 m/s
(i) The magnetic field is directed along the inward normal to the
plane of the paper. The particle leaves the region of
the field at the point F. Find the distance EF and the
angle 8.
(ii) If the direction of the field is along the outward normal to
the plane of the paper, find the time spent by the particle in
the region of the magnetic field after entering it at E.
(lIT 1984)

A particle of mass 1 x 10-26 kg and charge + 1.6 x 10-19 coulomb


travelling with a velocity 1.28 x 106 mls in the +X direction enters
a region in which a uniform electric field E and a uniform magnetic
field of induction B are present such that
Ex= Ey= 0, E, =-102.4kV/mand B.=B, = 0, By=8 x 1O-2weber/
m'. The particle enters this region at the origin at time t = o.
Determinethelocation (x, y and z coordinates) of the particle at
t = 5 x 10- 6 s. If the electric field is switehed off at this instant
(with the magnetic field still present), what will be the position of
the particle at t = 7.45 x 10- 6 s ? (lIT 1982)
".
13621 '1

A bar magnet with poles 25 cm apart and of strength 14.4Amp-m


rests with centre on a frictionless pivot. It is held in equilibrium at
an angle of60. with respect to a uniform magnetic field ofinduction
0.25 Wb/m2, by "applying a force F at right angle to its axis at a
point 12 cm from pivot. Calculate F. What will happen ifthe force
F is removed?
(lIT 1978)
PRACTICE PROBLEMS 281

13631_: __ "_' '-_"_"_' _"_'._"'_"'_'_" . _


Twoimmitely long parallel wires carry equal currents in the sanie
direction.
(i) What is the direction of the magnetic field due to one of the
wires at any point along the other wire?
(ii) What is the direction offorce on one wire due to the other?
(iii) By what factor does this force change if the current in each
wire is doubled?
(iv) What is the direction of magnetic field at a point midway
between the two wires? (lIT 1973)

~---------
13641_" .. _.-_. '-"-' -----
A metal bar AB can slide on
two parallel thick metallic
rails separated by a distance A
1. A resistance R and an
inductance L are connected to Ii
the rails as shown in the ,
figure. A long straight wire
carrying a constant current 10 ~L
"
~
is placed in the plane of the
rails and perpendicular to B
them as shown. The bar AB
is held at rest at a distance "0
from the long wire. At t = 0, it is made to slide on the rails away
from the wire. Answer the following question.
')
,a F'mare
d Ia t'Ion among I,. dt
di l\n d dcI> h
dt' were .. th e curren t
lIS
in the circuit and cI> is the flux of the magnetic field due to the
long wire through the circuit.
(b) It is observed that at time t = T, the metal bar AB is at a
distance of2xo from the long wire and the resistance R carries
a current il. Obtain an expression for the net charge that has
flown through resistance R froni t = 0 to t = T.
(e) The bar is suddenly stopped at time T. The current through
. L
resistance R is found to be ~ at time 2T. Find the value of R
in terms of the other given quantities. (lIT 20(2)
282. PRACTICE PROBLEMS

13651_" --------------
An inductor of inductance
L = 400 mH and resistors of
resistances R, = 20 and
R2 = 20 are connected to a
T
E
R,
battery of e.m.f. E = 12 V as
S
shown in the figure. The
internal resistance of the
battery is negligible. The switch S is closed at time t = O.
What is the potential drop across L as a function of time?
After the steady staw is reached, the switch is opened. What is the
direction and magnitude of current through R, as a function of
time? (liT 2001)

A thermocole vessel contains 0.5 kg of distilled water at 30°C. A


metal coil of area 5 x 10- 3 m2, number of turns 100, mass 0.06 kg
and reaistance 1.6Ois lying hori20ntally at the bottom ofthe vessel.
A uniform, time varying magnetic field is set up to pass vertically
thro:lgh the coil at time t = O. The field is first increased from zero
to 0.8 T at a constant rate between 0 and 0.2 s and then decreased
to zero at the same rate between 0.2 and 0.4 s. This cycleis repeated
12000time. Make sketches ofthe current through the coil and the
power dissipated in the coil as functions of time for the first two
cycles.Clearly indicate the magnitude ofthe quantities on the axes.
Assume that no heat is lost to the vessel, or the surroundings.
Determine the final temperature of the water under thermal
equilibrium. Specific heat of the metal = 500 jkg- , K-' and the
specificheat Jfwater = 4200 Jkg- I K-I. Neglect the inductance of
the coil. (liT 2000)
~7l"
A magnetic field B = (B~YJ k is 0 x
xExFx
into the paper in the + z direction.
Bo and 5. are positive constants. A
square loop EFGH of side a, mass
x
x H x
GJ G
x
x
!g
m and resistance R, in x-y plane,
x x x
starts falling under the influence
y
of gravity. Note the direction of x
and y axes in the figure. Find:
PRACTICE PROBLEMS • 283

(a) the induced current in the loop and indicate its direction.
(b) the total Lorentz force acting on the loop and indicate its
direction, and
(c) an expression for the speed of the loop, v (t) and its terminal
value. (lIT 1999)
13681_. ~ _
An inductor of inductance 2.0 mH is connected across a charged
capacitor of capacitance 5.0 "F, and the resulting LC circuit is set
oscillating at its natural frequency. Let Q denote the instantaneous
charge on the capacitor, and I the current in the circuit. It is found
that the maximum value of Q is 200 "C

(a) When Q = 100" C,what is the value of I~~I ?

(b) When Q = 200 " C, what is the value of! ?


(c) Find the maximum value of 1.
(d) When I is equal to one half its maximum value, find the value
of I Q I ? (lIT 1998)
13691~ _
A pair of parallel horizontal
conducting rails of negligible
resistance shorted at one end is
fixed on a table. The distance
between the rails is L. A
conducting massless rod of
resistance R can slide on the
rails frictionless rod ofresistance
R can slide on the rails
frictionlessly. The rod is tied to a
massless string which passes
over a pulley fixed to the edge ofthe table. A mass m, tied to the
other end ofthe string, hangs vertically. Aconstant magnetic field
B exists perpendi-cular to the table. If the system is released from
rest, calculate?
(i) the terminal velocity achieved by the rod, and
(ii) acceleration of the mass at the instant when the velocity of
the rod is halfthe terminal velocity.
(lIT 1997, July)
284 . PRACTICE PROBLEMS

13701 -------------------------~
An infinitesimally small bar magnet of dipole moment M is
pointing and moving with the speed v in the x-direction. A small
closed circular conducting loop of radius a and negligible self
inductance lies in the y-z plane with it centre at x = 0, and its axis
coinciding with the x-axis. Find the force opposing the motion of
the magnet, if the resistance of the loop is R. Assume that the
distance x of the magnet from the centre of the loop is much greater
than a.
(lIT 1997, May)

13711 ._. -_ .. _---------

A solenoid has an inductance of 10 henry and a resistance of


2 ohm. It is connected to a 10 volt battery. How long will it take
1
for the magnetic energy to reach 4
of its maximum value?
(liT 1996)
13721 _

A metal rod OA of mass m and length r is kept rotating with a


constant angular speed (i) in a vertical plane about a horizontal
axis at the end O. The. free end A is arranged to slide without
friction along a fixed conducting circular ring in the same plane as
~
that of rotation. A uniform and constant magnetic induction B is
applied perpendicular and into the plane of rotation as shown in
figure. An inductor L and an external resistance R are connected
through a switch S between the point 0 and a point C on the ring
to form an electrical circuit. Neglect the resistance of the ring and
the rod. Initially, the switch is open.

R x
L x
PRACTICE PROBLEMS 285

(a) What is the induced emf across the terminals ofthe switch?
(b) The switch S is closed at time t = O.
(i) Obtain an expression for the current as a function oftime.

(ii) In the steady state, obtain the time dependence of the


torque required to maintain the constant angular speed,
given that the rod OA was along the positive X-axis at
t = O. (lIT 1995)

Twoparallel vertical metallic rails AB and


CD are separated by 1 m. They are
connected at the two ends by resistances A c
R, and R2 as shown in the figure. A
horizontal metallic bar L of mass 0.2 kg
slides without friction, vertically down the ll!
L
rails under the action ofgravity. There is a
uniform horizontal magnetic field of 0.6 T
perpendicular to the plane of the rails. It n D
is observed that when the terminal velocity
is attained, the powers dissipated in R, and
R2are 0.76 Wand 1.2 W respectively. Find the terminal velocity of
the bar L and the values of R, and R2• (lIT 1994)

A rectangular frame ABCD made of A E B


uniform metal wire has a straight x x x x xx x
connection between E and F made of XXXXXX x x
x x xii x x x x x x
the same wire, as shown in the figure.
AEFD is a square of side 1m, and EB = XXXxXXXX
xXXXXXxX
x x
x X
FC = 0.5 m. The entire circuit is placed xxxxx~: X X

in a steadily in.creasing, uniform D F C


magnetic field directed into the plane of the paper and normal to
it. The rate of change ofthemagnetic field is 1 Tis. The resistance
per unit length of the wire is 101m. Find the magnitudes and
directions of the currents in the segments AE, BE and EF.
(lIT 1993)

A circuit containing a two position switch S is shown in figure.


(a) The switch S is in position '1'. Find the potential difference
VA - VB and the rate ofproductiort ofjoule heat in R,.
286 , PRACTICE PROBLEMS

(b) If now the switch S is put in position '2' at t = 0, find


R, C

2!l
R, E,
2!l 12V
1
A , R, n
2 S 3V 211
30 R,
L

lOmH

(i) steady current in R. and


(ii) the time when current in R. is half the steady value.
Also calculate the energy stored in the inductor L at
that time.
(lIT 1991)

Two long parallel horizontal rails, a distance d apart and


each having a resistance A per unit length, are joined at one end
by a resistance R. A perfectly conducting rod MN ofmass m is free
to slide along the rails without friction (see figure). There is a
uniform magnetic field of induction B normal to the plane of the
paper and directed into the paper. A variable force F is applied to
th~ rod MN such that, as the rod moves, a constant current flows
throughR.

x x xM, x x x
~ x
x x x :: x x x
x
R x x x F x d
B x
x x x x x x
x
x x xN x x x

(i) Find the velocity ofthe rod and the applied force F as functions
of the distance x of the rod from R.
(ii) What fraction of the work done per second by F is converted
into heat?
(lIT 1988)
PRACTICE PROBLEMS 287

Space is divided by the line AD into two regions. Region I is field


free and the Region II has a uniform magnetic field Bdirected into
the plane of the paper. ACD is a semicircular conducting loop of
radius r with center at 0, the plane of the loop being in the plane
offue paper. The loopis now made to rotate with a constant angular
velocity OJ about an axis passing through 0 and perpendicular to
the plane of the paper. The effective resistance of the loop is R.
Region I x X Region II X X

xxxxxx
xxxxxx
x x x x x x
B
x X X X X X

X X X X X X

D X X X X X X

X X X X X X

(i) Obtain an expression for the magnitude 6ffue induced current


in the loop.
(ii) Show the direction of the current when the loop is entering
into the Region II.
(iii) Plot a graph between the induced e.m.f. and the time of
rotation for two periods of rotation. (liT 1985)
1~78r., .
A square metal wire loop of side 10 cm and resistance 1 D is moved
wifu a constant velocity Vo in a uniform magnetic field ofinduction
B = 2 Wh/m2 as shown in the figure. The magnetic field lines are
perpendicular to the plane of fue loop (directed into the paper).
The loop is connectd to a network ofresistors each ofvalue 3 ohms.
The resistances of the lead wires OS and PQ are negligible. What
should be fue speed of the loop so as to have a steady current of 1
milliampere in the loop? Give the direction ofcurrent in the loop.
•.. --------~
,
: x x ,
xv"X~
: XliX
,
'x x .C
'x x
•,x x
L x__ ~_!,~ (liT 1983)' (
288 • -. PRACTICE PROBLEMS

r.;;;;;"]-
~ ~,~t~..
••••~~~;r.~
••,- ~-;'~.,
~". I'-;'~~
";;l"~"~il'~7.~-:'J~"'"
'.•_!~,"",f~;'~~"~~t~
.~.", ',~';r..~~'~
~q~'.ltl:}j:.~ :.'~~.~if~':'i;.i.":~~j~
. .
The two rails ofa railway track, insulated from each other and the
ground, are connected to a millivolt-meter.What is the reading of
the millivoltmeter when a train travels at a speed of 180km/hour
aong the track given that the horizontal components of earth's
magnetic field is 0.2 x 10-' weberslm2 and the rails are separated
by 1 metre? (liT 1981)

ALTERNATING CURRENT
1380I .,..po---.~
.\.'~1~.?~;w~1X~~~7~~~tf\ii'~¥.ttU~\~,-(.::r:Rf
'f-
_ ~ .••, I."~;
!'.
U'.Jf., .'..1S: ~."_~. _'I."'" :~4fI'J1-•••••_,';;:M".",-l.; ..,~~~
'.,(~.i'. :' .•
l~-~;~
__"'h'_"".~/.~LJ
In an LRseries circuit, a sinusoidalvoltageV=Vosin (j) t is applied.
(j)
Itisgiven thatL=35 mH, R= nn, Vnns = 220 V, -2 = 50Hz and
• 1t
22
It= 7',Find the amplitude of current in the steady state and
obtain the phase difference between the current and the voltage.
Alsoplot the variation ofcurrent for one cycleo-nthe given graph.
(liT 2004)
13811 r""!"i ~'l"''!~(Ji{Jjf$~-~!1N~-\n~:k~~h1i:..~~.
f,' •...•-_. ¥'!~.'"•.•.••.•'1'''' _",."i~r:r-,,,,,.-.- ". Wo'-, '~"~~,.,,:
J;;~!.! •.tl.~_~~l1!\~~~
•..•- •• ..,_." •••• ..,. ••. '.,. " .•.-.>.:-~,.•~., .•• ~~

A square loop of side 'a' with a


capacitor of capacitance C is
located between two current
carrying long parallel wires as
shown. The value ofI in the wires a
is given as I = 10 sin (j) t.
(a) Calculate maximum current
in the square loop.
(b) Draw a graph between charges on the upper plate of the
capacitor vs time. f (lIT 2003)
RAYOPTICS
13821 M.tf ..,ti' .!fliF!'~~~~{~~~
•. A light ray is incident on an
'.. : irregular shaped. slab of
, ~efracti~e'inde~ ,J2 at an angle "

of 45° with the normal on the --- •..


1-'3=1.514
E Q
--
incline face as shown in the
figure. The ray finally emerges
from the curved surface in the R =OAm
j ~.,
PRACTICE PROBLEMS 289

medium of the refractive index •.•= 1.514and passes through point


E. If the radius ofcurved surface is equal to 0.4 m, find the distance
OE correct up to two decimal places. (lIT 2004)
13831_' o'_-._""_-
An object is approaching at thin convex lens. of focal length 0.3 m
with a speed of0.01 mls. Find the magnitudes ofthe rates ofchange
ofposition and lateral magnification ofimage when the object is at
a distance of 0.4 m from the lens,
f~O.3m

O.Olm!.
,,
,,
,,
,,
•• OAm (lIT 2004)
13841; . -- .

A prism of refracting angle 30° is coated with a thin film of


transparent mat.erial ofrefractive index 2.2 on face ACofthe prism.
A light ofwavelength 5500 A is incident on face AB such that angle
of incidence is 60°, find
(a) the angle of emergence,
[Given refractive index ofthe material
of the prism is ,f3].
(b) the minimum value of thickness of the
coated film on the face AC for which
C
the light .emerging from the face has B
maximum intensity, (lIT 2003)
~-_. __ ._. -~----
Find the focal length of the lens shown in the figure. The radii of
curVature of both the surfaces are equal to R.

(lIT 2003)
290 PRACTICE PROBLEMS

','1
The refractive indices of the crown glass for blue and red lights
are 1.51 and 1.49 respectively and those of the flint glass are 1.77
and 1.73 respectively. An isosceles prism of angle 6° is made"of
crown glass. A beam of white light is incident at a small angle of
this pri.sm. The other flint glass isosceles prism is combined with
the crown glass prism such that there is no deviation ofthe incident
light. Determine the angle of the flint glass prism. Calculate the
net dispersion of the combined system. (lIT 2001)
13871 c "' _. - I
A thin biconvex lens ofrefractive index ~ is placed on a horizontal
plane mirror as shown in the figure. The space between the lens ,
4
and the mirror is then filled with water ofrefractive index "3' It is
found that when a point object is placed 15 cm above the lens on
its principle axis, the object coincides with its own image.

On repeating with another liquid, the object and the image again
coincide at a distance 25 cm from the lens. Calculate the refractive
index of the liquid. (lIT 2001)
13881_; .~-_-._._.- -_'-__ ""'_- - -_'_ .. __ '_
A convex lens offocallength
15 em and a concave mirror
offocal length 30 em are kept
with their optic axes PQ and Q
RS parallel but separated in R
vertical direction by 0.6 em as s
shown. The distance between
the lens and mirror is 30 em.
An upright objectABofheight 3~ 20crn
1.2 cm is placed on the optic
axis PQ of the lens at a distance of20 em from the lens. If A'B' is
the image after refraction from the lens and reflection from the
mirror, find the distance of A'B' from the pole of the mirror and
obtain its magnification. Also locate position of A' and B' with
respect to the optic axis RS. ilT 2000)
PRACTICE PROBLEMS 291

13891 _

The x-y plane is the boundary between two transparent media.


Medium - 1 with z;;, 0 has a refractive index ,J2 and medium - 2
with z ,;;o. 0, has a refractive index./3 .
A ray oflight in medium
- 1 given by the vector A = 6./3 i + 8./3 J- 10 k is incident on
the plane of separation. Find the unit vector in the direction of the
refracted ray in medium-2. (lIT 1999)
13901 _
A quarter cylinder of
radius R and refractive
index 1.5 is placed on a
table. A point object P is
kept at a distance of mR
from it. Find the value of P
m for which a ray from P
will emerge parallel to the ~.--,-n~R-
....•
+'--=R-- •••1

table as shown in the figure. (lIT 1999)


13911 _

A prism of refractive index n, and another prism of refractive index


nz are stuck together without a gap as shown i'1 the figure. The
angles of the prisms are as shown. n, and nz depend on A, the
wavelength of light, according to

n, = 1.20 + 10.8: 10' and nz = 1 45 10.8 x 10'


), . + )!'
where, A is in nm.

-
(a) Calculate the wavelength 1..0 for which rays incident at any
. angle on the interface BC pass through without bending at
that interface.
(b) For light of wavelength 1.0, find the angle ofincidencei on the
face AC such that the deviation produced by the combination
of prisms is minimum. (lIT 1998)
292 PRACTICE PROBLEMS

13921_" ----' ------_._--------~

A thin equiconvex lens of glass of refractive index f! = and of %


focal length 0.3 m in air is sealed into an opening at one end of a

. tank filled with water (f! =il On the opposite side of the lens, a
mirror is placed inside the tank on the tank wall perpendicular to
the lens axis, as shown in figure. The separation between the lens
and the mirror is 0.8 m. A small object is placed outside the tank
in front of the lens at a distance of 0.9 m from the lens along its
axis. Find the position (relative to the lens) of the image of the
object formed by the system.
1<---- 0.9 m ----+ ..-.0.8 m •. _.'"1.

•.................. MirroT

(lIT 1997, May)

13931_: ~ _
A right angle prism (450 - 900 - 450) of
refractive index n has a plate of
refractive index n, (n, < n) cemented to
its diagonal face. The assembly is in air.
A ray is incident on AB (see figure),
(i) Calculate the angle ofincidence at
AB for which the ray strikes the
diagonal face at the critical angle. B C

(ii) Assuming n = 1:352, calculate the angle of incidence at AB


for which the refracted ray passes through the diagonal face
undeviated. (lIT 1996)
13941 _
r
A thin plano-convex lens offocallength is split in to two halves:
one of the halves is shifted along the optical axis (see figure).

1.8m
PRACTICEPROBLEMS 293

The separation between object and image planes is 1.8 m. t,


The magnification of the image formed by one of the half-lenses is
2. Find the focal-length of the lens and separation between
the two halves. Draw the ray diagram for image formation.
(lIT 1996)

A ray of light travelling in air is incident at grazing angle (incident


angle = 90°) on a long rectangular slab ofa transparent medium of
thickness t = 1.0 m (see figure). The point ofincidence is the origin
A (0,0). The medium has a variable index of refraction n 0') given
by
n (y) = [Ky312 + 1]112
where K = 1.0 (metre)- 312

y
P(x"y,) AIR
,,
,,
,,,
t =l.Om , MEDIUM
,,
B(x, y)
,
,
,,
, ,,
X
A(o,o) AIR

The refractive index of air is 1.0.,


.. (a) Obtain a relation between the slope of the trajectory of the
ray at a point B (x,y) in the medium and the incident angle at
that point.
(b) Obtain an equation for the trajectory y (x) of the ray in the
medium.
(c) Determine the coordinates (x" y,) of the point P, where
the ray intersects the upper surface .of the slab-air
boundary. .
(d) Iildicate the path of the ray subsequently. (lIT 1995)
I~lllll~'
~~~_~~~~~~~~~~~~~~
Light is iricident at an angle a on one planar end ofa transparent
cylindrical rod of refractive index n. Determine the least value of
n sothat the light entering the rod does not emerge from the curved
surface of the rod irrespective of the value of a . (lIT 1992)
294 PRACTICE PROBLEMS

A parallel beam oflight travelling in water (refractive index = "3)


is refracted by a spherical air bubble of radius 2 mm situated in
water. Assuming the light rays to be paraxial,
(i) find the position of the image due to refraction at the first
surface and the position of the final image.
(ii) draw a ray diagram showing the positions ofboth the images.
(lIT 1988)

A right angled prism is to be


made by selecting a proper
material and the angles A and
B (B ,;;A), as shown in Figure.
It is desired that a ray of light
incident on the face AB
emerges parallel to the incident A B
direction after two internal
reflections.
(i) What should be the
minimum refractive index C
n for this to be possible?
5 '
(ii) For n ="3 is it possible to achieve this with the angle B equal
to 30 degrees? (lIT 1987)

Monochromatic light is incident on a plane interface AB between


two media of refractive indices n1 and nz (nz > nl) at an angle of
incidence' e as shown in the figure: The angle e is infinitesimally
greater than the critical angle for the tW9 media so that total
internal reflection takes place. Now if a transparent slab DEFG of
.'., uniform thickness and of refractive index n3 is introduced on the
Medium I
(n,) ,
D.- E
: Medium III
G: (us) F
A B
Medium II
(n,)
PRACTICE PROBLEMS' . 295

interface (as shown in the figure), show thatfor any value ofna all
light will ultimately be reflected back again into medium II.
Consider separately the cases.
(i) na < n, and (ii) na > n,. (lIT 1986)

A plano convex lens has a thickness of 4 cm. When placed on a


horizontal table with the curved surface in contact with it,
the apparent depth of the bottom most point ofthe lens is found to
be 3 cm. If the lens is inverted such that the plane face is in
contact with the table, the apparent depth of the centre of the
plane face is found to be 25/8 cm. Find the focal length of the lens.
(lIT 1984)

The convex surface of a thin


concavo-convex lens of glass of

refractive index 1.5 has a radius ..•
of curvature 20 cm. The concave ,

surface has a radius of curvature ,,


60 cm. The convex side is •,


,
silvered and placed on a horizontal
surface.
Where should a pin be placed
on the optic axis such that its
image is formed at the same
place?
's;::Jz
. . .. 4
(ii) If the concave part is filled with water of refractive index "3'
find the distance through which the pin should be moved so
that the image of the pin agmn coincide with the pin.
(lIT 1981)

An object is placed 21 cm in front of a concave mirror ofradius of


curvature 10 cm. A glass slab of thickness' 3 cm and refractive
index 1.5 is then placed close to the mirror in the space between
the object and the mirror.
Find the position r~ the final image formed.
(Youmay take the distance of the near surface ofthe slab from the
mirrortobe 1 cm).
296 PRACTICE PROBLEMS

140~1__ "'_' _"-_"~_-_"'_' "_"'_"_' _.-_-_-_-_~_--_.-_'-_ -_ -_ -_--.-_-_-_"-_ ..

A telescope has an objective offocallength 50 cm and an eye piece


offocallength 5 cm. The least distance of distinct vision is 25cm.
The telescope is focussed for distinct vision on a scale 200 cm away
from the objective. Calculate :
(i) the separation between the objective and the eye-piece.
(ii) the magnification produced. (lIT 1980)
14041_, :_" --"-'' -' ---"-' -' "-' -----"----'-' ._"------_._._"._.,,_._-_._,._.-.

A rectangular block ofglass is placed on a printed page lying on a


hori..zontalsurface. Find the value of the refractive index of glass
for which the letters on the page are not visible from any of the
vertical faces of the block. (lIT 1979)
1 4051_" -_._, "-' ._-_ .. _ ... _.... _. -----'---"_ ... _... _ .. _-_. __ .-_._' '-'-

The radius ofcurvature ofthe convex face of a plano convex lens is


12 cm and its refractive index is 1.5.
(i) Find the foca1length of this lens.
(ii) The plane surface ofthe lens is now silvered. At what distance
from the lens will parallel rays incident on the convex face
converge. I

(iii) Sketch the ray diagram to locate the image, when a point
object is placed on the axis, 20 cm from the lens (polished).
(iv) Calculate the image distance when the object is placed as in
(iii). (lIT 1979)
14061, ",'-' -",-
A ray of light is incident at an angle of 60 on one face of prism 0

which has an angle of30°, The ray emerging out ofthe prism makes
an angle of 300 with the incident ray. Show that the emergent ray
is perpendicular to the face through which it emerges and ca!culate
the refractive index of the material of the prism. (lIT 1978)
14071~,'_' ~_~ ~'
A pin is placed 10 cm in front of a convex lens offoca1length 20 cm,
made of a material of refractive index 1.5, The surface of the lens
farther away from the pin is silvered and has a radius ofcurvature
are 22'cm, Determine the position of the final image, Is the image
real as virtual? (lIT 1978)
PRACTICE PROBLEMS 297

1~~~IH';EEHEUHH
A glass lens has focal length 5 cm in air. What will be its focal
length in water. (Refractive index of glass is 1.51 and that of water
is 1.33). (lIT 1977)

A person looking through the telescope T


just sees the point A on the rim at the
bottom of a cylindrical vessel when the
vessel is empty (see figure). When the
. vessel is completely filled with a liquid of
refractive index 1.5, he observes a mark
at the centre B of the bottom, without
moving the telescope on the vessel. What A B c
is the height ofthe vessel if the diameter
of the cross-section is 10 cm? (lIT 1977)

A ray of light is travelling from diamond to glass. Calculate the


minimum angle of incidence of the rayon the diamond glass
interface such that no light is refracted into glass. What will happen
if the angle of incidence exceeds the angle? (refractive index of
glass is 1.51 and that of diamond is 2.47) (lIT 1977)

What is the velocity oflight in glass of refractive index 1.5?(Velocity


of light in air = 3 X 1010 cm/sec.) (lIT 1976)
- -.-.-.;.-. . ; ; . . "".,.<",.
1mI
412' ' ' ,.....•.....•...
'............•............•.....•.
'.'"..•...
'.'."."..:..'..,..'..'..'" '.'.'
..' '.' , '..'..'.' '.'.'.'.'.' .( <,'
':':':':':':'.:::':':':"
';;::::::::::::::.:-:.:.;.;.:.:::::::::::::::::::::::::

The refractive index of the material of a prism of refraCting angle


45° is 1.6 for a certain monochromatic ray. What should be
minimum angle of incidence of this rayon the prism so that no
total internal reflection takes place as the ray comes out of the
prism. (lIT 1976)

Photographs of the ground are taken from an aircraft flying at an


altitude of 2000 metres by a camera with a lens offocallength 50
ems. The size of the film in the camera is 18 em x 18 em. What
area of the ground can be photographed by this camera at anyone
c_ ?
Lnne.
(lIT 1976)
298 PRACTICE PROBLEMS

A point object '0 is placetl at a distance of 12 cms on the axis of a


convex lens offocallength Hkms. On the other side of the lens, a
convex mirror is placed at a distance of 10 cms from the lens such
that the image formed by the combination coincides with the object
itself. What is the focallength of the convex mirror? (liT 1976)

A rectanguIarglass block of thickness 10 cm and refractive index


1.5 is 'placed . over ,a small ,coin. A beaker filled with water ,of
4
refractive index ':3 to ,a height of.16 ,cmand is ,placed :.over the
glass block.
(a) Find the apparent'position of.theobject when it iswiewed at
near normal incidence.
(b) Draw a neat,ray diagram.
(c) If the eye is slowly moved away from.the normal at a certain
position the object is found to disappear due to total internal
,reflection. At which surface does this happen and why?
.(HT1975)

.A projector lens has a'foca:lJength 10.cm. iItthrows animage.of,a2


cm x 2 cmslide on a screen 5 metre from the lens. Find::
(i) the size of the ,picture on the screen and
(ii) ratio of illuminations of the slide and of the picture on the
screen. (IIII' <1975)
1~171"
An object isplaced at 20 cm left of the convex lens offocal'Iength
10 cm. If a concave mirror offocal length 5 cm is placed at 30 cm to
the right of the 'len. find the magnification and'the nature of the
final image. 'Draw:theray diagram and locate the ,position of the
final image.

,0-0 )-
I' .• .- •

20cm 30cm '(IIII' 1971&')


PRACTICE PROBLEMS 299

14181 _
An object is placed in front of a convex mirror at a distance of 50
ems. A plane mirror is introduced covering lower half ofthe convex
mirror. If the distance between the object and the plane mirror is
30 ems, it is found that there is no parallux between the images
formed by two mirrors. What is the radius of curvature of the convex
mirror?
(lIT 1973)

14191 _

A ray of light incident


normally on one of the faces
of a right angled isosceles
prism is found to be totally
reflected as shown in the
figure. What is the minimum
value of the refractive index
of the material ofthe prism?
When the prism is immersed in water, trace the path of
theemergent rays for the same incident ray, indicating the values
4
of all the angles. (J.I" = "3)
(lIT 1973)

142°1 _
An object of height 4 cm is kept to the left of and on the axis of a
converging lens of focal length 10 cm as shown in figure. A plane
mirror is placed inclined at 45° to the lens axis 10 cm to the right
of the lens (see figure). Find the position and size of the image
formed by the lens and mirror combination. Trace the rays forming
the image.

to-l5cm
,,
• Object

(lIT 1972)
300 PRACTICEPROBLEMS

~~----------
14211 ~ __ ~ _
In a Young's double slit experiment light consisting of two
wavelengths )" = 500 nm and )'2 = 700 nm is incident normally on
the slits. Find the distance from the central maxima where the
maximas due to two wavelengths coincide for the first time after
D
central maxima. (given d = 1000) where D is the distance between
the slits and the screen and d is the separation between the slits.
(lIT 2004)

~_. ------------
Apoint source 8 emitting light of wavelength 600 nm is placed at
a very small height h above a flat reflecting surface AB (see figure).
The intensity of the reflected light is 36% of the incident intensity.
Interference fringes are observed on a screen placed parallel to
the reflecting surface at a very large distance D from it.

Screen

(a) What is the shape of the interference fringes on the screen?


(b) Calculate the ratio of minimum to the maximum intensities
in the interference fringes formed near the point P (shown in
the figure).
(c) If the intensity at point P corresponds to a maximum, calculate
the minimum distance through which the reflecting surface
AB should be shifted so that the intensity at P again becomes
maximum. (UT 2002)

~---------~-----
!" vessel ABCD o£1O cm width has two small slits 8, and 8 sealed 2
with identical glass plates of equal thickness. The distance between
the.stlits is 0.8mm. POQ is the line perpendicular to the plane AB
and passing through 0, the middle point of 8] and 82, A
monochromatic ight source is kept at 8, 40 cm below P and 2m
PRACTICE PROBLEMS 301

from the vessel, to illuminate the slits as shown in the figure below.
Calculate the position of the central b.right fringe on the other
wall CD with respect to the line OQ. Now, a liquid is poured into
the vessel and filled up to OQ.
A D
P SI
.'."'."""' .. "'., ... "., ..••...•.••.... ' •.•..•• Q
: 0
!40cm 82
!S .__
..
?!!1:._. __ .
C
The central bright fringe is found to be at Q. Calculate the refractive
index of the liquid. (lIT 2001)

A glass plate of refractive index 1.5 is coated with a thin layer of


thickness t and refractive index 1.8. Light of wavelength I.
travelling in air is incident normally on the layer. J:t is partly
reflected at the upper and the lower surfaces ofthe layer and the
two reflected rays interfere. Write the condition for their
constructive interference. If I. = 648 nm, obtain least value oft for
which rays interfere constructively. (lIT 2000)

The Young'sdouble slit experiment is done in a medium ofrefractive


index ~. A light of600 nm wavelength is falling on the slits having
0.45 mm separation. The lower slit 82 is covered by a thin glass
sheet of thickness 10.4 I-'m and refractive index 1.5. The
interference pattern is observed on a screen placed 1.5 m from the
slits as shown.
y

J
'*~~----------------
"
(aJ . Find the location ofthe central maximum (bright fringe with
zero path difference) on the y-axis.
(bJ Find the light intensity at point 0 relative to the maximum
fringe intensity.
302 ' PRACTICE PROBLEMS

(e) Now, if600 nm light is replaced by white light ofrange 400 to


700 nm, find the wavelengths of the light that form maxima
exactly at point O.
[Allwavelengths in this problem are for the given medium of
4
refractive index "3' Ignore dispersion] (IIT 1999)

A coherent parallel beam of microwave of wavelength 'A. ; 0.5 mm


falls on a Young's double apparatus. The separation between the
slits is 1.0mm. The intensity ofmicrowaves is measured on a screen
placed parallel to the plane of the slits at a distance of 1.0 m from
it as shown in the figure. .
1
d =1.0 mm
x
D=l.Om

Screen

(a) If the incident beam falis normally on the double slit


apparatus, find the coordinates ofall the interference minima
on the screen.
(b) If the incident beam makes an angle of30° with the x-axis (as
in the dotted arrow shown in the figure), find the y-coordinates
of the first minima on either side of the central maximum.
(IIT 1998)

In a Young's experiment, the upper slit is covered by a thin glass


plate of refractive index 1.4 while the lower slit is covered by
another glass plate, having the same thickness as the first one but
having refractive index 1.7. Interference pattern is observed using
light of wavelength 5400 A. It is found that the. point P on the
screen where the central maximum (n ; 0) fell before the glass
3. .
plates were inserted now has '4 the original intensity. It is further
observed that what used to be the fifth maximum earlier, lies below
the point P while the sixth minimum lies above P. Calculate the
thickness ofthe glass plate. (Absorption oflight by glass plate may
be neglected). (IIT 1997, July)
PRACTICE PROBLEMS 303

14281~, _
In Young's experiment, the source is red light of wavelength 7 x
10- 7 m. When a thin glass plate of refractive index 1.5 at this
wavelength is put in the path of one of the interfering beams, the
central bright fringe shifts by 10- 3 m to the position previously
occupied by the 5th bright fringe. Find the thickness of the plate.
When the source is now changed to green light of wavelength 5 x
10-7 m, the central fringe shifts to a position initially occupied by
the 6th bright fringe due to red light. Find the refractive index of
glass for the green light. Also estimate the change in fringe width
due to the change in wavelength. (lIT 1997, May)
14291-,~,._----------------------
.

A double-slit apparatus is immersed in a liquid of refractive index


1.33.It-has slit se;>arationof 1rom, and distance between the plane
of slits and screen is 1.33 m. The slits are \lluminated by a ,parallel
beam of light whose wave-length in air is 6300 A.
m Calculate the fringe-width.
(ii) One of the slits of the apparatus is covered by .a thin
glass sheet ofrefractive index 1.53.Find the smallest thickness
of the sheet to bring the adjacent minimum to ,the,axis.
.(lIT 1996)
14301_: ' ...._.. -_ ...._...._-_.
In the figure shown, 8 is a monochromatic point source ,emitting
light of wavelength A = 500 nm. A thin lens of cireular shape and
focal length 0.10 m is cut into two identical halves L.and ~ by a
plane passing 'through a diameter. The two halves are placed
symmetrically about the central axis SO with .a gass of 0.5 mm.
The distance along ,the axis from 8 to L. and L2 is '0.15 m, while
that .from L, and :~ to 0 is 1.30 m. The screen at Q is normal to
80.

O.15m 1.3Om
-, ,0
304 PRACTICE PROBLEMS

(i) If the third intensity maximum occurs at point A on the screen,


find distance OA.
(ii) Ifthe gap between L] and L2 is reduced from its original value
of 0.5 mm, will the distance OAincrease, decrease or remain
the same? (lIT 1993)

I~tli'
Two parallel beams of light P and Q (separation d) containing
radiations ofwavelengths 4000 A and 5000 A (which are mutually
coherent in each waveleugth separately) are incident normally on
a prism as shown in Figure. The refractive index of the prism as a

function of wavelength is given by the relation, I' (A) = 1.20 +-;-


where A is in A and b is positive constant. The value of b is su~h
that the condition for total reflection at the face AC is just satisfied
for one wave length and is not satisfied for the other.

(a) Find the value ofb.


(b) Find the deviation ofthe beams transmitted through the face
AC.
(e) A convergent lens is used to bring these transmitted beams
into focus. Ifthe intensities of the upper and the lower beams
immediately after transmission from the face AC, are 41 and
I respectively, find the resultant intensity at the focus.
(lIT 1991)

A narrow monochromatic beam of light


of intensity I is incident on a glass plate
as shown in figure. Another identical
glass plate is 'kept close to the first one
and parallel to it. Each glass plate reflects
25 per cent ofthe light incident on it and
transmits the remaining. Find the ratio (2)
PRACTICE PROBLEMS ' 305

of the minimum and the maximum intensities in the interference


pattern formed by the two beams obtained after one reflection at
each plate. (lIT 1990)
14331_' __ ' '_'_' ._.. ._.. _.=_.._. _
In a modified Young's double slit
experiment, a mono-chromatic
uniform and parallel beam oflight
ofwavelength 6000 A and intensity

(~O)Wm- 2 is incident normally


F

on two circular apertures A and B


of radii 0.001 m and 0.002 m
respectively. A perfectly trans-
parent film of thickness 2000 A
and refractive index 1.5 for the wavelength of 6000 A is placed in
front of aperture A, (see figure). Calculate the power (in watts)
received at the focal spot F of the lens. The lens is symmetrically
placed with respect to the apertures. Assume that 10%ofthe power
received by each aperture goes in the original direction and is
brought to the focal spot. (lIT 1989)
14341_, ----------_._ .. _.,,_ .. _.,-- .._----

A beam of light consisting oftwo wavelengths, 6500 A and 5200 A,


is used to obtain interference fringe in a Young's double slit
experiment:
(i) Find the distance ofthe third bright fringe on the screen from
the central maximum for wavelength 6500 A.
(ii) What is the least distance from the central maximum where
the bright fringes due to both the wavelengths coincide?
The distance between the slits is 2 mm and the distance between
the plane of the slits and the screen is 120 em. (lIT 1985)

14351._ --------------
In Young's double slit experiment using monochromatic light the
. fringe pattern shifts by a certain distance on the screen when a
mica sheet of refractive index 1.6 and thickness 1.964 microns is
introduced in the path of one of the interfering waves. The mica
sheet is then removed and the distance between the slits and the
screen is doubled. It is found that the distance between successive
maxima (or minima) now is the same as the observed fringe shift
upon the introduction ofthe mica sheet. Calculate the wave length
of the monomchromatic light used in the experiment. (lIT 1983)
306 PRACTICE PROBLEMS

1436 1- . -.
Screen S is illuminated by two point
sources A and B. Another source C sends
a parallel beam of light towards point P on
the screen (see figure). Line
AP is normal to the screen and the lines A P
AP, BP and CP are in one plane The
distances AP, BP and CP are 3 m, 1.5 m
and 1.5 m respectively. The
radiant powers of sources A and Bare 90
watts and 180 watts respectively. The beam S
from C is of intensity 20 wattslm2. Calculate the intensity at P on
the screen. (liT 1982)

~
14371: - _ ••. ----- -_. ~ .-

Two metallic plates A and B, each of area 5 x 10-4 m2, are placed
parallel to each other at a separation of 1 cm. Plate B carries a
positive charge of 33.7 x 10- 12C. A monochromatic beam oflight
with photons of energy 5 eVeach, starts falling on plate A at t = 0
so that 101Gphotons fallon it per square mls. Assume that one
photoelectron is emitted for every lOG incident photons. Also
assume that all the emitted photoelectrons are collected by plate
B and the work function of plate A remains .constant at the value 2
eV. Determine:
(a) the number of photoelectrons emitted up to t = 10 s.
(b) the magnitude of the electric field between the plates A and
B at t = 10 sand
(c) the kinetic energy of the most energetic photoelectron emitted
at t = lOs when it reaches plate B.
Neglect the time taken by the photoelectron to reach plate B.
Take So = 8.85 x 10-12 C2/N_m2 (liT 2002)
..
'

14381. ,
Assume that the de Broglie wave associated with an electron can
form a standing wave between the atoms arranged in a one
dimensional array with nodes at each of the atomic sites. It is found
that one such standing wave is formed if the distance d between
the atoms of the array is 2 A . A similar standing wave is again
formed if d is increased to 2.5 A but not for any intermediate
PRACTICEPROBLEMS 307

value of d. Find the energy of the electrons in electron volts and


the least value of d for which the standing wave of the type
described above can form. (lIT 1997, July)
1439 1 _

In a photoelectric effect set-up, a point source of light of power


3.2 x 10-3 W emits monoenergetic photons of energy 5.0 eV.The
source is located at a distance of 0.8 m from the centre of a
stationary metallic sphere of work function 3.0 eV and of radius
8.0 x 10- 3 m. The efficiency of photoelectron emission is one for
every lOGincident photons. Assume that the sphere is isolated
and initially neutral and that photoelectrons are instantly swept
away after emission.
(a) Calculate the number of photoelectrons emitted per second.
(b) Find the ratio of the wavelength of incident light to the de
Broglie wavelength of the fastest photoelectrons emitted.
(c) It is observed that the photoelectron emission stops at a certain
time t after the light source is switched on. Why ?
(d) Evaluate the time t. (lIT 1995)

14401_' --------------
A monochromatic point SC
source S radiating
wavelength 6000 A, with
power 2 watt, an aperture
A of diameter 0,1 m and a D
large screen SC are placed
as shown in figure. A
photoemissive detector D -----6"'
of surface area 0.5 cm2 is placed at the centre of the screen. The
efficiency of the detector for the photoelectron generation per
incident photon is 0.9.
(a) Calculate the photon flux at the centre of the screen and the
photocurrent in the detector.
(b) If the concave lens L of focal length 0.6 m is inserted in the
aperture as shown, find the new values of photon flux and
photocurrent. Assume a uniform average transmission of80%
from the lens.
(c) If the workfunction of the photoemissive surface is 1 eV,
calculate the vlauM of the stopping potential in the two cases
(without and with the lens in the aperture). (lIT 1991)
308 PRACTICE PROBLEMS
1
441
1'.. '" .-;-.-.,_. :- ... - -'.
A beam of light has three wavelengths 4144 A, 4972 A and 6216 A
with a total intensity of 3.6 x 10- 3 W m- " equally distributed
amongst the three wavelengths. The beam falls normally on an
area 1.0 cm" of a clean metallic surface of work function 2.3 ev'
Assume that there is no loss oflight by reflection and that each
energetically capable photon ejects one electron. Calculate the
number of photoelectrons liberated in two seconds. (lIT 1989)

~
1
442
1' .._-..... ------------------
..... ... .. . ... .. ... ... .. ....-

In a photoelectric setup, the radiations from the Balmer seriesof


hydrogen atom are incident on a metal surface of work function 2
eV.The wavelength of incident radiations lies between 450 nm to
700 nm. Find the maximum kinetic energy of photoelectron
emitted. (Given hc/e = 1242eV -nm) (lIT 2004)
14431-.' ..
Characteristic X-rays offrequency 4.2x 1018 Hz are produced when
transitions from L shell to K shell take place in a certain target
material. Use Mosley's law to determine the atomic number of the
target material. Given Rydberg constant R = 1.1 x 107 m-'.
(lIT 2003)
14441 _.
" .. "

A hydrogen-like atom (described by the Bohr model is observed to


emit six wavelengths, 'originating from all possible transitions
between a group of levels. These levels have energies between
-0.85 eVand -0.544 eV (including both these values).
(a) Find the atomic number of the atom.
(b) Calculate the smallest wavelength emitted in these
transitions.
Take hc = 1240 eV-nm, and ground state energy ofhydrogen
atom = - 13.6 ev' (lIT 2002)
14451-' ... _.
(a) A hydrogen-like atom of atomic number Z is in an excited
state of quantm number 2n. It can emit a maximum energy
photon of 204 eV.If it makes a transition to quantum state n,
a photon ofenergy 40.8eV is emitted. Find n, Z and the ground
state energy (in eV)for this atom. Also,calculate the minimum
PRACTICE PROBLEMS 309

energy (in eV) that can be emitted by this atom during de-
excitation. Ground state energy of hydrogen atom is -13.6
eV
(b) When a beam of 10.6 eV photons of intensity 2.0 W/m2 falls
on a platinum surface of area 1.0 x 10-4m2 and work function
5.6 eV, 0.53% of incident photons eject photoelectrons. Find
the number of ph9toelectrons emitted per second and their
minimum and maximum energies (in eV).
Take 1 eV = 1.6 x 10-19 J (lIT 2000)
14461 _

Photoelectrons are emitted when 400 nm radiation is incident on


a surface of work function 1.9 eV These photoeclectrons pass
through a region containing a- particles. A maximum energy
electron combines with an a-particle to form a He+ ion, emitting a
single photon in this process. He+ ions thus formed are in their
fourth excited state. Find the energies in eV of the photons, lying
in the 2 to 4 eV range, that are likely to be emitted during and
after the combination.
[Take, h = 4.14 X 10-15 eV -s] (lIT 1999)
14471 _

An electron, in a hydrogen-like atom, is in an excited state, It has


a total energy of = -3.4 eV. Calculate :
(i) the kinetic energy, and
(ii) the de Broglie wavelength of the electron. (lIT 1996)
14481 _
An electron in the ground state of hydrogen
atom is revolving in anticlock-wise
direction in a circular orbit of redius R.
(i) Obtain an expression for the orbital
magnetic dipole momentum of the
electron.
(ii) The atom is placed in a uniform
magnetic induction B such that the
plane-normal of the electron orbit makes an angle of 30° with
the mlfgnetic induction. Find the torque experienced by the
orbiting electron. (lIT 1996)
14491__ ~ - _
A hydrogen-like atom (atomic number Z) is in a higher excited
state of quantum number n. This excited atom can make a
310 PRACTICE PROBLEMS

transition to the first excited state by successively emitting two


photons of energies 10.20 eVand 17.00 eV respectively.
Alternatively, the atom from the same excited state can make a
transition to the second excited state by successively emitting two
photons of energies 4.25 eV and 5.95 eV respectively. Determine
the values of nand Z. (lIT 1994)
«(Ionization energy of hydrogen atom = 13.6 eV)
14501_, . _"'_"'_"_" '0

A neutron of kinetric energy 65 eV collides inelastically with a


singly ionized helium atom at rest. It is scattered at an angle of
90° with respect of its original direction.
(i) Find the allowed values of the energy ofthe neutron and that
of the atom after the collision.
(ii) If the atom gets de-excited subsequently, by emitting
radiation, find the frequencies of the emitted radiation.
[Given: (mass ofthe atom) = 4 (mass of neutron), (Ionisation energy
ofH atom) = 13.6 eV] (lIT 1993)
14511 '.'

Light from a discharge tube containing hydrogen atoms falls on


the surface of a piece of sodium. The kinetic energy of the fastest
photoelectrons emitted from sodium is 0.73 eY The work function
for sodium is 1.82 eY Find .
(a) the energy of the photons causing the photoelectric emission.
(b) tj:>e quantum numbers of the two levels involved in the
emission of these photons,
(c) the change in the angular momentum of the electron in the
hydrogen atom in the above transition, and
(d) the recoil speed of the emitting atom assuming it to be at rest
before the transition.
(Ionization potential of hydrogen is'13.6 eV) (lIT 1992)
r:;;;;l
~ .. '
. ~ '1
.. > ~.
~,.
"':""';: :~-. ".--:-'--"".':>--:''1
_ ""',' ~~'. .: 'r •. '

Electrons in hydrogen-like atoms (Z = 3) make transitions from


the fifth to the fourth orbit and from the foutth to the third orbit.
The resulting radiations are incident normally on a metal plate
and eject photoelectrons. The stopping potential for the
photoelectrons ejected by the shorter wavelength is 3.95 volts.
Calculate the work function of the metal, and the stopping potential
for the photoelectrons ejected by the longer wavelength.
(Rydberg constant '= 1.094 x 10' mol) (lIT 1990)
PRACTICE PROBLEMS ", 311

A gas ofidentical hydrogen-like atoms has some atoms in the lowest


(ground) energy level A and some atoms in a particular upper
(excited) energy levelB and there are no atoms in any other energy
level. The atoms of the gas make transition to a higher energy
level by absorbing monochromatic light of photon energy 2.7 eV.
Subsequently, the atoms emit radiation of only six different photon
energies. Some of the emitted photons hav" energy 2.7 eV, some
have energy more and some have less than 2.7 eV.
(i) Find the principal quantum number of the initiaIly excited
level B.
(ii) Find the ionization energy for the gas atoms.

(iii) Find the maximum and the minimum energies ofthe emitted
photons. (liT 1989)

A particle of charge equal to that of an electron, --<J, and mass 208


times the mas~ of the electron (caIled a mu-meson) moves in a
circular orbit around a nucleus of charge +3e. (Take the mass of
the nucleus to be infinite). Assuming that the Bohr model of the
atom is applicable to this system,
(i) derive an expression for the radius ofthe nth Bohr orbit.
(ii) find the value of n for which the radius of the orbit is
approximately the same as that ofthe first Bohr orbit for the
hydrogen atom.
(iii) find the wavelength of the radiation emitted when the
mumeson jumps from the third orbii to the first orbit.
(Rydberg's constant = 1.097 x 107 m-I). (liT 1988)

A doubly ionised Lithium atom is hydrogen-like with atomic


number 3.
(i) Find the wavelength of the radiation required to excite
the electron in Li++ from the first to the third Bohr
orbit. (Ionisation energy cf the hydrogen atom equals
13.6 eV.)
(ii) How many spectral lines are observed in the emission
spectrum of the above excited system? .
(liT 1985)
312 PRACTICE PROBLEMS

The ionization energy of a hydrogen like Bohr atom is 4 rydbergs.


(i) What.is the wavelength of the radiation emitted when the
electron jumps from the first excited state to the ground state?
(ii) What is the radius of the first orbit for this atom?
(llT 1984)

IDtraviolet light ofwavelengtbs 800 A and 700 A when allowed to


fall on hydrogen atoms in their ground state is found to liberate
electrons with kinetic energy 1.8 eV and 4.0 eV respectively. Find
the value of Planck's constant. (llT 1983)

Hydrogen atom in its ground state is excited by means of


monochromatic radiation of wavelength 975 A. How many different
lines are possible in the resulting spectrum? Calculate the longest
wavelength amongst them. Youmay assume the ionization energy
for hydrogen atom as 13.6 eY. (lIT 1982)

A single electron orbits around a stationary nucleus of charge


+ Ze, where Z is a constant and e is the magnitude of the electronic
charge. It reqnires 47.2 eV to excite the el2ctron from the second
Bohr orbit to the third Bohr orbit.
Find:
(i) The value of Z.
(ii) The energy reqnired to excite the electron from the third to
the fourth Bohr orbit.
(iii) The wavelength of the electromagnetic radiation required to
r(,move the electron from the first Bohr orbit to infinity.
(iv) The" kinetic energy, potential energy and the angular
momentum of the electron in the first Bohr orbit.
(v) The radius of the first Bohr orbit.
(The ionization energy of hydrogen atom = 13.6 eY, Bohr radius =
5.3 x 10-11 metre, velcoiy of light = 3 x 108 mlsec. Planck's constant
= 6.6 x 10- 34 joul't-sec).
(llT 1981)
PRACTICE PROBLEMS 313

The age of a rock containing lead and uranium is equal to 1.5 x 10"
. yrs. The uranium is decaying into lead with half life equal to 4.5 x
10" yrs. Find the ratio of lead to uranium present in the rock.
Assuming initially no lead was present in the rock
(Given 2113 = 1.259)
r.:::-W -_.--- -~ ..
~I
. -- - ...._ _._-~----.~
..
(IIT2OO4)

A radioactive element decays by II emission. A detector records


n beta particles in 2 seconds and in next 2 seconds it records
0.75 n beta particles. Find mean life correct to nearest whole
number.
Given: ln2 = 0.6931,ln3 = 1.0986 (TIT 2003)
r;;;;;]-
~l.' -------~-. , - ..'---
.
A radioactive nucleus X decays to a nucleus Y with a decay constant
Ax = 0.1 sec.l. Y further decays to a stable nucleus Z with a decay
constant A.y = 1130sec-I. Initially, there are only X nuclei and their
number is No = 1()20: Set up the rate equations for the populations
ofX, Y and Z. The population of the Y nucleus as a function of time
is given by

Ny(t) = C,"~~A;J(e-Ar _e-Azt)


Find the time at which Ny is maximum and determine the
population of X and Z at that instant. (TIT 20(1)
~;-,... -.--l
Nuclei of a radicactive element A are being produced at a constant
rate a. The element has a decay constant)" At time t = 0, there are
No nuclei of the element.
(0) Calculate the number N of nuclei of A at time t.
(b) If a = 2 No A, calculate the number of nuclei of A after
one half-life of A, and also the limiting value of N as
t -+ co.
(IITl998)

L
314 PRACTICE PROBLEMS

~Hi'"
In an ore containing Uranium, the ratio oftJ238to Pb206nuclei is 3.
Calculate the age ofthe ore, assuming that all the lead present in
the ore is the final stable product oftJ238.Take the half-life ofD'38
to be 4.5 x 109 years. (lIT 1997 May)

At a given instant there are 25% undecayed radio-active nuclei in


a sample. Mter20 seconds the number ofun decayed nuclei reduces
to 12.5%.Calculate (i)mean-life ofthe nuclei, and the time in which
the number of undecayed nuclei will further reduce to 6.25% of
the reduced number.
(lIT 1996)

A small quantity of solution containing 24Naradio-nuclide (half


life 15 hours) of activity 1.0micro-curie is injected into the blood of
a person. A sample of the blood of volume 1 cm3 taken after 5
hours shows an activity of 296 disintegrations per minute.
Determine the total volume of blood in the body of the person.
Assume that the radioactive solution mixes uniformly in the blood
of the person.
(1 Curie = 3.7 x 1010disintegrations per second) (lIT 1994)

There is a stream of neutrons with a kinetic energy of0.0327 eV.If


the half life of neutrons is 700 sec0nds, what fraction of neutrons
will decay before they travel a.distance of 10 m ?
(lIT 1986)

~~. ~~~~~=c.

~
In a nuclearreactcr undergoes fission liberating 200
energy. The reactor has a 10%efficiency and produces 1000 MW
power. If the reactor is to function for 10 years, find the total mass
of uranium required.
(lIT 20(1)
PRACTICE PROBLEMS 315

A nucleus at rest undergoes a decay emitting an a particle of de-


Broglie wavelength /..= 5.76 x 1O-15m. If the mass of the daughter
nucleus is 223.610 a.m.u. and that ofthe a particle is 4.002 a.m.u.,
determine the total kinetic energy in the final state. Hence, obtain
the mass of the parent nucleus in a.m.u.
<i a.m.u. = 931.470 MeV/c2) (lIT 2001)

The element Curium ~~8cm had a mean life of 10'3 seconds. Its
primary decay modes are spontaneous fission and a decay, the
former with a probability of 8% and the latter with a probability of
92%. Each fission releases 200 MeV of energy. The masses involved
in a decay are as follows:
~:8Cm = 248.072220 u,

~:4pu = 244.064100 u and ;He = 4.002603 u.


Calculate the power output from a sample of 1020 em atoms.
(lu = 931 MeV/c"). (lIT 1997)

A nucleus X, initially at rest, undergoes alpha-decay according to


the equation,

~2X-+ ~28y+ a
(a) Find the values of A and Z in the above process.
(b) The alpha particle produced in the above process is found to
move in a circular track of radius 0.11 m in a uniform magnetic
field of3 Testla. Find the energy (in Mev) released during the
process and the binding energy of the parent nucleus X.
Given that

m(Y) = 228.03 u; m (iH) = 1.008 u;

m{~n) =1.009u;m{;He)=4.003u;
(lIT 1991)
316 , ,
PRACTICE PROBLEMS

It is proposed to use the nuclear fusion reaction


2H+
112
2H4 4He
In a nuclear reactor of200 MWrating. If the energy from the above
reaction is used with a 25 per cent efficiency in the reactor, how
many grams of deuterium fuel will be needed per day. (The masses
of rH and ~He are 2.0141 atomic mass units and 4.0026 atomic
mass units respectively). (UT 1990)

••
ANSWERS
.
',~


.'
PRACTICE PROBLEMS . 317

GrmDD.....
-------------
1. (a) 450
(b) 2 mfs.

2. initial velocity = 6 mfs


acceleration = 4 mfs2 downward along the plane

u~ ) t
4. ( u+~ '

(ii) ~(~)t2
2 u+~

6. 4S krnlh.
7. 36km.
8. 48krnlh.
9. (i) The block that falls freely will strike the ground earlier.
(ii) Same speed.
10. Same time.
11. (i) Velocity = + X-axis, acceleration = 0
(ii) Velocity = 0, acceleration = 0
(iii) Velocity = - X-axis, acceleration = 0
12. 10m.

13. 9 metres.

14. U = 7.2 mfs, t = 1 sec.

u2 sin 2a.
15. (a)
g cose
ucos(u+e)
(b)
cose
16. (i) M = 1 second
(ii) (sJ3m, Sm)
h 1
17. H = 2"
318 PRACTICEPROBLE~.lS
18. (i) 747.1 metres
(ii) 85.55 mls
19. (a) - 1mls
(b) 1.48 sec:

20. (ii) aT = (5:) g

N= (~) mg

21. (i) 20 N
(ii) 50 N

22. Velocity = 6.54 mls


Time taken by m is 2.8 second.

Time taken by 2 m is ~ second.

23. F(l-D.
m,. sin a + Ill:! sin fl)
24. f = ( m,. cos a + Ill:! cosfl g

m,.~gsin (a - fl)
T = (m,. cos c'1+ ~ cos fl)

25. 3 K
26. (a) (i) 17400 N (ii) 14700 N (iii) 1200 N
(b) 36m
(e) Average velocity = 3 mls
Average acceleration =0
27. 25 crnJs2 Eastward.
28. 110 em from the pulley.
230 emls towards right.
29. (i) remains a8me
(ii) increase
30. (a) t = 2 sec
(b) Sa = 7./2 m
SA= 8./2 m
PRACTICE PROBLEMS 319

31. (b) fl = 30 N
f. = 15N
F =60N
Tension = 18 N
(Acceleration ofM) = (Acceleration ofml) = 0.6 mls2
(ml is at rest w.r.t. M).
32. (i) zero

ai) (2;Z) mg

(iii) ::12
33. FMlN= ~
1+ fl2
a = tan-I fl with horizontal
34. Ms!= 10 kg, fl = 0.098 J
35. (i) M) = 4.2 kg
(ii) 9.8 N

36. VC=VF= ~2g(y-lJX)

37. (i) 1.31 mls2


(ii) 5.2 N
38. 78.4 N
39. 4.43 x 10' N

40. (i) 98(Jz -1) N


ai) 98(Jz +1) N
3 -.J2)
(iii) ( 3 +.J2

41. (i) 36N


(ii) 11.67 rad/s
(iii) m) at a distance of 0.1 m from 0 and m. at a distance of 0.2 m
fromO.
320 PRACTICE PROBLEMS

42. (i) h = R _ .L
2
00

00 min = Jf = 9.9 radls

(ii) Ihglmin = 9.8 x 10- 3 mls2

. 1 ~Mg
43. 2rrmL
44. To apply brakes.

45. (i) e = cos-1 (:~) = 52.20

(ii) 1.6 N
(iii) 3.16 mls

46. (i) 2.2 second


(ii) 2.123 N

47. NA
__f1 mg (3 coOSe - 2) e:s:coS-l~
1
e '" cos- ~

t
'e:s:cos-l~
B
N = (mg(2-:coSe) e",cos-1

48. u= gL(2 + 3
2
./3)
49. 3.36 mls.
50. 4.24m

51. A:g (hI _ h,)2 .

52. (i) Zero


(ii) 18 m, where m is the mass of the body.

53. (i) 14 mls


(ii) 10 m.

';I","~ .' • ,"'""'""'


PRACTICE PROBLEMS 321

54. 40 (3 cos 8 - 2 cos 80) kg wt, 60 0

k
55.
2r
56. (i) First
. (ii) Second

57. 80cm.
58. 313.6 Watt.
59. (i) K.E. = 0.098 J
(ii) T = 0.294 N
. x
60. (i) "2

.. Wx
(n) 2
W Wx
\.'iii) F = -2 ' work done = -2
(iv) 2

61. (a) x = vot + ~ A (1- cos Olt)


2
"'"
(b) 1 =
o
(1 + ~ ) A
m2

100..[3
63. to = 12 seconds, 11 mls.

26
64. e =0.84, M =_ kg
.f3
65. (a) 2.5 mls
(b) L= 0.32m

66. (L + 2R, 0).


67. (i) d = 10J3 m
20
(ii) J3 m from B.
68. 105 m.
322 . PRACTICE PROBLEMS

69. (i) Vertical straight line.


x2 y2
(ii) ---+ 1
(~-rr r
2
=
(y-axis is a vertical line through C.M.)
The path of the point is a part of an ellipse.

70. (a) 5)% ~gd


(b) 6d.j3i.
71.44.18m

72. (i) 8= cos-J (~) =37.

(ii) (120 m; 45 m)

73. n = 4.
74. 6.53 seconds

75. (i) u=-


Uo
3
2mu02
(ii) k---
- 3x 8
76. Distance = meR - r)
(m+M)

Speed = m 2g (R - r)
M(m+M)

77. The particle Q hits the ground at the mid point of AB.
5
Time = J2 second.

78. 9 cm left from.the centre of plate.


79. 25%.

80. (i) J(mV)2 + (MU)2 ,8 = tan-


1
(~;) with x-axis.
2 2
'ii) Mm ( V +u )
" M+m Mu2+mV2
PRACTICE PROBLEMS " • 323

81. Less.
82. 0.94
83. (i) 135.3 N
(ii) Yes.
84. 3,Ja mis, No.
85. 480N.
86. (i) t = 1 second, 4.9 M below the top ofthe cliff.
(ii) 77.55 metres above the top of the cliff.
87. (i) 15 mls downward
(ii) 1080 kgm/s, 1080 kgmls

88. 40cm/s.
89. (a) ,Ja mro2[

(b) Fy = ,Ja mro2[

F
F =--
x 4
90. (a) 0.1 M
(b) 1 rad/s
(c) Infmite
m 1
91. (a) M = 4"
2
(b)AP="3L
DO
(c) Vp = 2./2
8F
92. (a) a -----
plank - 3m, + 8m"
4F
acylinder = 3m, + 8m2
3m,F
(b) fcPlank & Cylinder) = 3m, + 8m"

m,F .
fcCylinde. & ",onnd) = 3m, + 8m"
324 ' PRACTICE PROBLEMS

93. ~5gR

94. (a) F =
-> 2mv
M (~
1-
k) ; N-> = ( mg+ J3M
J3 2mv ) ,
k

(b) T- 4mvh
- J3M
95. (i) 6 N
(ii) ~I = 0.6 (k - J) Nm

~2 = 0.6(-k-J) Nm

I~ll = 1~21= (0.6).12 Nm


.2
96. (i) "3vo

(ii)t-~
0- 3J.1g

3 2 1 2
W = - mVoMt + 2m (Me) for t > to' W= --mvo
6

97. (a) Be = cos-


1
(t)
(b) V = ~tgR

(c) KT = 6
KR
98. (i) T=4.9N
(ii) S = 1.224 m

99. 6.3m/s
12V
100. (a) 7L

(b) :2,j2gL = 3.5 m/s

101. P; gR

102. 2.72 J.
PRACTICE PROBLEMS 325

103. 2 m, sphere continues to rotate.

104.
mv3
2,/2 g
105. 20.

106. TAP = ~113 kg wt


TBQ = 7,/2 kgwt

e =tan-1 m
107. 34N.
108. (i) 48.3 kg
(ii) 2 m2

109. 100R

110. V = ~J5GM
2 .a
t
111. (i) ; h = R = 6400 km
(ii) v = ..fiR = 7.92 km/s

112. v = JG:;
3
T=21tJa
3Gm
113. (i) 1t x 10"' km/hr

(ii) ~ radians/hr

114. 35940 km, above the earth's surface.


115. 2,/2 second
"
116. 4.89%
117. 0.00457 K.
118. (i) 133.3 em from steel wire
! :

(ii) 100 em from steel wire


326 PRACTICE PROBLEMS

119. 3.75 m/s


120. (a) zero
(b) 0.25 em
g
(c)
6
121. (nR2L)(v'Pcr" -p)

122. (a) (dLd:d}l


(b) Periodic but not SHM.
(c) Ball will move with constant velocity within the liquid.
123. 8 = 45°
124. (i) 840 gm
(ii) P aUn + 103 N/m2
125. In the beginning eM + PwV)
where Pw = density of water
Then the reading increases and at the end it becomes eM+ in).
)
126. Side length = 10 em.
127. h=R.
~
•..
128. No change.
7
129. (i) - kg
3
(ii) 1; kgwt, vertically downward.
130. 0.5 metre.
131. 3.33 litres.
132. Liquid level will rise.
133. (i) 2.35 em
(ii) 2.54cm
134. Decrease ..
135. 9.61 cm
PRACTICE PROBLEMS 327

136. 1.96m2
137. 2m.

138. W(gravity) = - 2.94 x 104 Jim"


W(p,",",u,e) = 2.9025 x 104 Jim"

139. (i) :0
(ii) ~2gmo
Ap

140. (a) (i) :!..d


4

(ii) Po {T+~)dg
(b) (i) v= ~f(3H - 4,';)
(ii) x = .J h(3H - 4h)

(iii) h = ~H
m 8
3
x =-H
m 4
dQ oc-;
141. dt r

142. lAlx 10-3 NS/m2

143. 'Aag
2y

4T
144.- 2
pv

145. ..!-r
2"
g
2R

143.

l
328 PRACTICE PROBLEMS

1
147. (i) - Hz
"
(ii) " mls
50
(iii) 4,,2 x 10- 5 J

148.. (a) s=tan-1m

(b) T = 2"
~2g cosS
~8.l
149. (1+ ~;) sec.

150. 1
2"
JYA
ml
151. (i) 4.8 x 10- ~ J
(ii) 3.75x10-4J

153. 336 mls.


MA _ 400
154. (a) -
MB 189

(b) nA 3
nB ="4
155. Heights of water level = 3.2 m, 2.4 m, 1.6 m, 0.8 m and
dH -JH
0; dt = 179
Time interval= 42.9 sec.
156. (a) 0.14 sec.
(b) Ar=2.1cm, ~
.. '

Ii, = 1.4 em
PRACTICE PROBLEMS ,329

157. (a)

(b)

(c) Pmin:PmlU:Po
(d) Pmin:PO-&'o
P mlU: Po + l>Po
158. LclO8ed: 0.75 m
Lopen: 1.0067 m or 0.9934 m

159. Equation of motion, y: (2 X 10-5 m) sin (O.lm) cos (25000 It t)


Constituent waves, Yl: (l X 10-5 m) sin (5ltx - 25000 It t)
Y2: (l X 10- 5, m) sin (51tx + 25000, It t)
where x is in metre and t is in second.

160. (i) 21t X 10-3 s.

(ii) " 10-3


"2x' S.

21t b'
161. (a) A: -,V:-
a 2"
(b) Y : - 0.8 A cos (ax - btl'
(c) Maximum speed: 1.8M'
Minimum speed: 0
(d) y ; Y travelling + Y stationary"
: 0.2A COS (ax + btl - lL6;A sin ax&inlbt
Antinodes of stationary' wave,

x: (n+i)~ with n: 0.1,2., .

The direction of propagatiollJ of wavelling:wave'i:il' negative


X-axis. . .

162. (i) Z 1 and z2" Resultant iittel1'Sitywill1lle' zero,at,,».:: ~m' + ~) i


where m' = O~1, ~, ..... ..

(ii) Zl and:z3. Resultant intensity will'be'zero,atl(x-y.);7'. (2n+ 1) i


where' n : 0; 1,.2, .
330 . . PRACTICE PROBLEMS

163. (i) 3.46 em


(ii) n = 0, 15 em, 30 em, 45 em, 60 em
(iii) 0

(iu) Y, = 2sin(96llt+ ~;). Y2 = -2~in(961lt- ~;)


,164. 0.12 m
165. 11Hz.

100)
166. ( 133.mls

167. 27N.
168. Diameter = 3.33 em, v.= 163.3 Hz.
169. (i) 95 Hz
(ii) 1.27 x lO"kglm3
170. 1650Hz.
171. 50Hz.
172. 230 mls.

173. (N umberofGoops 'onviire ,of'l'adiusr)~_


~-- 1
(Nuniber.oflloops.on wire of radius 2r) .2

ri4. 45em.

175. 25J2 Hz.

176. 260Hz.
177. (i) 270'm
(ii) 36(hn!s

178..
179. 7:21. :gmicc , ,
180. (a) l00696lHz
(b) 103039I1ifz.

181.
f [2 (u-+'Vm)'IIb'1
v2._.vl .J
PRACTICE PROBLEMS . 331

182. "max.=484 Hz, "min= 403.3 Hz.


183. V max= 442 Hz, V:min'=.255 Hz.
184. (i) 599 Hz

(ii) (~~) km,620 Hz.

185. 1.5 mls


186. 8 beats/sec.
187. Yz= 20<,
188. Coefficient,ofvo!ume,expansion, y=,2 x l'o-'fo,C
189. (i) 400 kg
(ii)2J
190. Y = 1.1 X 1011N/m2
0<=2x 1O-5foC
.
,191. 70mls

il92. [
(W2 -'WI) +'/3 (WO-WI) (t2 - tl)]
, ,two -W~H~2'-tl) .'

193- Loses 10.3:7 .seco!\ds per da,Y.


194. 75.04 em.
195. .Length ofthe,copperrod.=:9.16.em
Length ofthesteeLmd = 14.16,em

""96.
i' ") "F
",~ AT(11
I = ------
"-1 -+ il,2"-2~
(il+~)
''') I 1 ....Fli
III . I +, ,i,,-.I~-"Y2A" 112
, Fl2
+ll~"-iT - Y A
2
UW. :2:31 x 16-~~C.

L
332 PRACTICE PROBLEMS

200. 3 x 1O-4/°C.
201. 25.2N.
202. 0.495 kg
203. 12gm
204. O°C
205. 20.25°C
206. 1.8 x 10- 41°C
207. 100°C, (572.2 gm water + 77.5 gm steam)
208. 178.5°C
209. 25.5°C
210. lkg/hour.
K
211.
[4<JET;L+Kj

212. (a) 595 J/m2.sec (b) 163°C

213. (300 + (12.5)e- 2KAl,/CL)K

214. Rate of heat flow = 41.6 W


Temperature of Glass I and air =26.48°C
Temperature Glass II and air = 0.52°C
215. 166.4 seconds
216. 9000W.
217. TB = 30°C, and Tc = To = 20°C
218. 15.24°C.
219. 93.3°C, 0.257 calls

220. Ratio of radii, rA = ~KB


rB KA

221. 2°C
222. 0.3 callm-sec-oC
PRACTICE PROBLEMS 333

223. 76°C
224. (a) 160 K
(b) 3.312 x 10-21 J
(c) 0.3012 gIll
225. neon = 4.08 gm, argon = 23.92 gIll.
226. 75.4 em of mercury.
227. 83.75 em of mercury.
228.

229. 817 mm ofHg


230. 3 em.
231. 1.1588 g.
232. Length = 36.5 em, pressure = 85.3 em ofHg.
233. 8.02 x 101•

234. --~=-
dTOP

dBOTTOM
75

76

235. 48 em.

236. 400i (f K

mv~_
237.
3R

238. (a) 327°C


(b) QAB= 1500R
QBe = 831.8 R
~D=-900R
QDA= - 831.8 R
(c) 600R
334 PRACTICE PROBLEMS

239. (a) + 1200 R


(b) IlQAB = -2100 R
IlQnc= 1500 R
Il~A = 831.77 R

240. (b) IlW;,ip,VI [1-(~J/:i]


AU=Q-2PIVI
3 [(V V: )2/3]
1-

T -_ ~+
3R
PI VI
2R
(VIV )2/3
2

24L (a)PoVo

(b) ~A (Rejected) = % PoVo


QAB(absorbed) = 3PoVo

(c) Qnc(absorbed) = 21PoVo


25PoVo
(d) Tm",,= 8R

242. Tn = 909 K, To = 791.4 K, '1 = 61.4%


243. (i) TA = 120.28 K, Tn = 240.56K, Tc = 481.12 K, To = 240.56 K
(ii) No.
(iii) QABC = 3.25 x 106 J
QAOC = 2.75 x 106J
244. (i) 189 K
(ii) -2767 J
(iii) 2767 J
246. (a) 2 moles
(b) 401 mls
(c) 0.167%
V
(d) 6" = 4OT'

J
PRACTICE PROBLEMS 335

246. (i) W4=765J


(ii) 10.82%
247. (i) 1870J
(ii) 5300J
(iii) 500K

248. (a) 1153 J


(b) 1153 J
(c) zero
249. (b) W = (0.58) RTA, AQ = (0.58) RTA
250. m 5

(ii) ~PV
4
251. (ii) V = 0.113 m3, P = 4.41 X 104 N/mz
(iii) 12450 J
252. 800 K, 720 J
253. T = 675 K, P = 3.6 x 106 N/mZ

254. (i) (~~7)TO(Left chamber), (~)To(Right chamber)


(ii) 15.58 To
255. (i) 1.94 x lOz7
(ii) Vo = 35.6 mls
256. 972J.

257. ~tPo
211 MYo

q(O", - O"z)a
258.
2../2 &0
qp
259. (a) Z
4" Eo d
qp
(b) 3
2" EO d

L
336 . PRACTICE PROBLEMS

260. -5.824
__q2
(4" eo)a

261. 5.86 mls.


262. va = 3 mis, KE at origin = 2.5 x 10-' J.
4
263. (a) aa,
(b) Height of equilibrium position = :fa.
264. (a)

265. (~)cr
266.
~2eom

3Q2
267. (a)
207t eo R
(b) 15 x H,ll J
Q2
(c)
8" eo R

268. (a) Equation of circle, x2 + y2 - lOa.X + 9a2 = 0


radius = 4a, centre at (Sa, 0)

(b). V = 4"~ ex _13a1 Ix :3al]


'.: ,

(c)
v~
~

.... '.}"
PRACTICE PROBLEMS 337

cr
269. (i) V - -(a-b+c)
A - Eo

Vc= :(:2 _b
C
2
+c)
(ii) c=a+b

270. t= ;J~~
271. 3.16 x 10-9 C
272. 2q and Sq at ends. q at a distance of 3 em from 2 q and 6 em from
Sq.
E=O.

273. OD= 6./2 m

274. (~) seconds.

275. 8.9 x 10-' N, tan-l (~)

276. (i) 60°


mg+ qlq2
(ii)
4,. Eo 12
qlq2
(iii) mgJ3 • mg when chord is cut 4,. Eo 12 =mg

277. (i) move towards


q
(ii) - J3' zero.

278.

279.

L
338 PRACTICE PROBLEMS

280. Horizontal component = 106 ml8


Vertical component = 1.88 x 106 mls (upward)
282. Smaller conductor is placed inside the bigger conductor.
283. K= 2.
284. 10<mls.
Q
285. q = -4 on the middle point of the line joining two charges.

286. The neutral point is ata distance of8cm from + 4Q and 4 cmfrom
+Q.
287. 4.73 cm, No
288. zero.
289. 2 x 10- 12 coulomb.
290. (i) Vp = 0, VQ = 400 volt,
ai) Ep = 4.242 x 103 N/C along -ve x-axis
EQ = 5.33 x 103N/C along +ve x-axis
(iii) 400 J/C
au) No, independent of path
291. 3062.5 volt, 1.96 mls2
292. Same charges.
q q
.293. (i) V= ,E= 2
2" Eo a 3" Eo a
q q
(ii) V= ,E= 2
6" Eo a 5" Eo a
294. (i) 1.5 x 10- 2 volt, . .
'"
.. ", .
(ii) 2.382 x 10- 2 volt

295. 5.88 x 105 N/C


296. (C!) (i) 6 cm from + 100 esu and 24 cm from - 400 esu.
(ii) 10 cm from + 100 esu and 40 cm from - 400 esa.

(b) At many points in the space satisfying the condition',!L = .!.,


. T2 4
where T I ,i.s,distance from +.100 esu an!i T 2 is distance from
-400 es (i.,.,....
. .~. .'
I . "

(e) 30 cm fiom + 100 esu and 60.cui froin - 4QO esiI:


"", ! -,
PRACTICE PROBLEMS ' 339

297. S.79 x J. 10-6


298. (i) QA = 90 fiC, QB = 150 fiC, Q.; = 210 fiC
(ii) Uinitial = 4.74 x 10- 2 J
Ufinal= 1.Sx 10-2J

EOA( K}K2 lin K2


299. d K2-K2 ! K1

300. (i) 2 X 10-9 F, 1.21 X 10-5 J


(ii) 4.84 x 10-5 J
(iii) 1.1 x 10-5 J

301. utnitiaI =~
"Final 5
302. 32 IlF
23
~NlTIAL 1
303.
c"NAL 2
4
304. R= -(T) - T2).
3
304. (a) No. (c) So.
306. (a) c; (1_e- 2t!3RC)

(b) 1= ~ (3- e-2t13RC)


V
Att4<.O,I= 2R'

V1T2 - V2T1) T.T2


307. V=:. ( 7. +72
,7=
7. +72

308. 0.21Jf\.
20
309. 3"y.
310. 4.425 x 10-9 A
311. 1.5 A, 1.44 x 10- 5 J.
312. (i) 2 n (ii) 1.5 A .
340 PRACTICEPROBLEMS

313. 8 X 10-4 J.

21
(ii) P.O. across G is 13 V,

19
P.O. across H is 13 V

315. 0.9A.
316. Reading of ammeter = 4.96 x 10-3 A
Reading of voltmeter = 1.95 V
317. (i) 2V
(ii) Current through E1 is 1 A
Current through Ez is 2 A
Current through E3 is 1 A
Current through R is 2 A
318. (ii) 22.5 V
319. PD. = 2.5 volts.
320. (i) 9 min 42 second.
(ii) Same.

321. VA - VB = 25 V, VB - Vc = 75 V
322. U1
323. 50.
324. 0.2%.
325. 31.25 W.
326. 40V.
327. 790 ohms
328. 50.
10
329 • -0.
.3 .
... 330. 1 cel1:-:-
PRACTICEPROBLEMS 341

331. 0.095 sec.


332. (i) 200n.
(ii) 0.23 volts.

333. (i) . 14 Ininutes


24
(ii) 7 minutes.

334. 1.40 amp, 1.66 amp, 1.94 amp.


335. (i) 7.28 calories
(ii) 5.86 volts.
336. Power in second coil is more.
337. Larger.
.
338. Current through 4.5 n, 3 nand 6 n are respectIvely 0.5 A, "31A ,
1
6"A. Potential difference across E1 and E2 are 4.25 Vand 7.5 V
respectively.
1
339. "3 ohm.

340. 70%.
341. 90 watts.

342. (i) (1~0) ohm.


(ii) Yes.
(iii) E = 2 volt, r = 0.49 n.
rp 1
343.
r. = ./2'
DTo
344.
QR2B

345. (a) From Pto Q

(b) IbBo (3k - 4i)


mg
(c)
6bBo
342 ' PRACTICE PROBLEMS

346. (a) 6.54 x 10-'5 T


(b) (i) Zero
(ii) Zero (on AC)
(iii) 8.11 x lo-GN (on CD)

347. (a) ("o::t) (-k)


(b)
-
F1 = (2ffiR)i
-
.F2 = (2ffiR)i
Force on the loop F= (4mR)i
mvo
348. (a) 2qBo

(b) -vo'
...• ...•
...•
349. v = (COSlDt)VO
-(SinlDt)~+
...• Exv (qt)m E...•
qB
where ,lD=- m

. 350. (a)
...•I
= _0"
T2B("J-'.~)
__
~ J2
(b) h9= (~}At)2
d
351. (i) :J: = :t .J3
(ii) I r;;-
2nd"~
31.2. T = 0.2 seconds.
353. 4.74 x 100"T.
2 2
1'01
-- ) In( 1+-a )
( 21t 'L2 ,zero.

355. (a)

(b)
PRACTICE PROBLEMS 343

356. (i) 1= 4 Amp.


(ii) 1 m from R on RX and 1 m from R on RQ.

357. 10-4 Tupward.


358. (i) 3 Amp.
(ii) 1.3 x 10-6 T
(iii) 2.88 x 10- 6 N/m
359. radius = 12 mm, .pitch = 43.7 mm.
360. (i) EF = 0.1414 m, e = 45°
(ii) 4.71 x 10-6 S.

361. At t = 5 X 10-6 s, coordinates are (6.4 m, 0, 0)


At t = 7.45 X 10-6 s, coordinates are (6.4 m, 0, 2 m)

362. 6.5N .
. 363. (i) Perpendicular to plane containing wires.
(ii) Attractive towards other.
(iii) 4 times.
(iv) Magnetic field is zero.

364. (a) iR+Ldi = -d<l>


dt dt
l102
(b)
2-(1'010
R 2"
Li.)
OJ

L T
(c) R = 2102

365. (12) e-5J v~lt, (6) e-1o, Amp.


366. Final temperature = 35.61°C

367. (a) (B~V) in the direction EHGFE (anticlock wise)

(b) _(B~;2v)J
fC.l _ mgR (l-e-«')
"J v - B'oa 2
B2a' ~R
where (1 = -~ ' v"nninal=B~a'
344 I PRACTICEPROBLEMS

368. (a) I~: I = 10' Als


(b) 1=0
(c) 1m•• = 2A
(d) IQI = 173 fie
mgR'
369. (i)
B2L2
g
(ii)
2
(21fi~M2a'v )
370.
4Rx8
371. 3.47 seconds.
1
372. (a) -BOJ ,2
2

(b) (i) B;~2 (1- e- RtIL)

2
B ,' OJ mg'
(ii) + -- cos OJt
4R 2
373. v = 1 m/s, R, = 0.474 n, R2 = 0.3 n.
7
374. 22 Amp. from E to A,

3
11 Amp. from B to E,
1
22 Amp. from F to E.

375. (a) VA - VB = -5 V, Rate of production of joule heat in R, is 24.5


W.
(b) (i) 0.6A
(ii) 1.386 ms, 4.5 x 10- 4 J.
I 2Aml2
'376. m v = -d (R+2XA), F= IBd+--(R+2xA)
B B2d2
~_ 1
(ii)
W - 2IAm
1 + -3-3 (R + l!XA)
Bd
PRACTICE PROBLEMS 345

377. (i) (-l)n[B;~(j)l


where n = 1, 2, 3, .... is the number of the half revolution.
378. V o = 2 cm/s, clockwise.
379. 1 millivolt.
. 11
380. o = 20 Amp ' <ll= -4 .
!

l'oClo(j)2aln 2
381. (a)
11

CloClo(j)aln 2)C
(b) 1=- - oS(j)t
1t

382. OE=6.06m.
383. 0.09 mis, 0.3/5.
384 . (a) 0
• (b) 1250 A.
1'3R
385.
1'3 - 1'\

386. 4°,0.04°.
387. 1.6
388. Distance of A'B' from pole of mirror 15 em, magnification = - 1.5.
Distance of A' above RS is 0.3 em,
Distance of B' below RS is 1.5 em.

(3/ +4J -5k)


389.
5./2
4
390. m=3
391. (a) "0 = 600 nm,
(b) i = sin-
1
(i)

--_._---------- ..
348 . PRACTICE PROBLEMS

424. 2J.1t; (n +~)A, where n ; 0, 1, 2, 3, .....


tmin = 90 nm.

425. (a) y;4.33 mm


3
(b) I; "4lm
(c) 650 nm, 433.3 nm
1 3
426. (a) y;:l:.,fl5m
15 '
:l:,[7m.
7 '
1 3
(b) _--m -m
y -.,fl5 ',[7
427. 9.3 J.1m
428. Thickness glass plate; 7 J.1m..
Refractive index of glass for green light; 1.6
Change in fringe width; 57.1 J.1m
429. (i) 0.63 mm
(ii) 1.575 J.1m
430. (i) 1mm
(ii) increase
431. (a) 8 x 10-15 m2
(b) 27.
(c) 91.

432. Im_. ; 49
Imin

433. 7 x 10- 6 watts.


434. (i) 1.17 mm
(ii) 4.68mm
435. 5892 A
436. 14 W/m2•
437. (a) 5 x 107
(b) 2000 N/C;
(c) 23 eV
438. 150.8 eV, 0.5 A
PRACTICE PROBLEMS . 349

439. (a) 105


Alight
(b) = 286.2
Aelectron

(c) Sphere gets positively charged (d) 111 seconds.

440. (a) 6 x lOll, 9.6 X 10-8 A


(b) 1.478 X lOll, 2.13 x 10-8 A
(c) 1.07 V in both the cases.
441. 1.1 x 10 '2
442. 0.55 ev.
443. z=42.
444. (a) z= 3 ; (b) Amin = 4060 nm
445. (a) n = 2, z: 4, Emin: 10.58 eV
(b) N = 6.25 lOll, Emin: 0, Emax:
X 5 eV.
446. 3.3 eV, 3.94 eV, 2.64 ev'
447. (i) 3.4 eV
(ii) 6.663 x 10-10 M.

448. (i) --
4nm' ( eh )
. (ii) --
81tm
(ehB)
449. z: 3, n : 6.
450. (i) Neutron 6.36 eV, 0.32 eV; He atom 17.84 eV, 16.32 eV
(ii) 9.87 x 10'5 Hz, 11.68 x 10'5 Hz, 1.82 x 1015 Hz.

451. (a) 2.55 eV,


(b) n: 2 andn: 4,
h
(c)
11

(d) 0.814 rn/s


452. Work function = 2 eV
Stopping potential for longer wavelength: 0.756 V.
453. (iJ 2; (ii) 14.4 eV; (iii) 13.5 eV, 0.7 ev'

£0 h2
454. (i) 6241t me2
(ii) n = 25
(iii) 5.48 x 1O-1lM.
350

455. (i) 114 A,


(ii) Three.
• PRACTICE PROBLEMS
l
456. (i) 300A
(ii) 0.26 A
t
457. 6.6 x 1O-34Joule-sec.
458. Number of lines = 6
Longest wavelength = 1.88 filll

459. (i) z = 5,
(ii) 16.53 eV
(iii) 36.4A
(iv) KE = 340 eY, PE = - 680 eY, L = 1.05 x 10-34 J-sec.
(v) 1.06 x 10-11 m.

Npb
460. N = 0.259
u
i
461. 0.145/sec. I
462. t = [(15) In 3) seconds.
Nx = 1.924 x 1019•
Nz = 2.302 x 1019•

463. (a) N = ~ (1- e-"') + Noe~'"


. 3.
(b) After one half life N " 2' No, At -.. DO N = 2No

464. 1.8684 x 10" years. I'.


r

465. Mean life = 14.43 Seconds, t = 40 seconds.


466. 5954cc.

r
467. 3.96 x 10-"
468. 3.845 x io" kg.
469. KE = 6.5 Mev; Mass = 227.62 a.m.u.
470. 3.32 x 10-" W. II
471. (a)
(b)
A= 232, Z = 90
Energy released = 5.34 MeV., binding energy = 1823 MeV. r
472. 121gm.
~
•• ...
,
,.
,.

1
)

Vous aimerez peut-être aussi